Вы находитесь на странице: 1из 113

I.

Introduction

G.R. No. 154514. July 28, 2005

WHITE GOLD MARINE SERVICES, INC., Petitioners,


vs.
PIONEER INSURANCE AND SURETY CORPORATION AND THE STEAMSHIP MUTUAL UNDERWRITING
ASSOCIATION (BERMUDA) LTD., Respondents.

DECISION

QUISUMBING, J.:

This petition for review assails the Decision1 dated July 30, 2002 of the Court of Appeals in CA-G.R. SP No. 60144, affirming
the Decision2 dated May 3, 2000 of the Insurance Commission in I.C. Adm. Case No. RD-277. Both decisions held that there
was no violation of the Insurance Code and the respondents do not need license as insurer and insurance agent/broker.

The facts are undisputed.

White Gold Marine Services, Inc. (White Gold) procured a protection and indemnity coverage for its vessels from The
Steamship Mutual Underwriting Association (Bermuda) Limited (Steamship Mutual) through Pioneer Insurance and Surety
Corporation (Pioneer). Subsequently, White Gold was issued a Certificate of Entry and Acceptance. 3 Pioneer also issued
receipts evidencing payments for the coverage. When White Gold failed to fully pay its accounts, Steamship Mutual refused
to renew the coverage.

Steamship Mutual thereafter filed a case against White Gold for collection of sum of money to recover the latters unpaid
balance. White Gold on the other hand, filed a complaint before the Insurance Commission claiming that Steamship Mutual
violated Sections 1864 and 1875 of the Insurance Code, while Pioneer violated Sections 299, 63007 and 3018 in relation to
Sections 302 and 303, thereof.

The Insurance Commission dismissed the complaint. It said that there was no need for Steamship Mutual to secure a license
because it was not engaged in the insurance business. It explained that Steamship Mutual was a Protection and Indemnity
Club (P & I Club). Likewise, Pioneer need not obtain another license as insurance agent and/or a broker for Steamship
Mutual because Steamship Mutual was not engaged in the insurance business. Moreover, Pioneer was already licensed,
hence, a separate license solely as agent/broker of Steamship Mutual was already superfluous.

The Court of Appeals affirmed the decision of the Insurance Commissioner. In its decision, the appellate court distinguished
between P & I Clubs vis--vis conventional insurance. The appellate court also held that Pioneer merely acted as a collection
agent of Steamship Mutual.

In this petition, petitioner assigns the following errors allegedly committed by the appellate court,

FIRST ASSIGNMENT OF ERROR

THE COURT A QUO ERRED WHEN IT RULED THAT RESPONDENT STEAMSHIP IS NOT DOING BUSINESS IN THE
PHILIPPINES ON THE GROUND THAT IT COURSED . . . ITS TRANSACTIONS THROUGH ITS AGENT AND/OR BROKER
HENCE AS AN INSURER IT NEED NOT SECURE A LICENSE TO ENGAGE IN INSURANCE BUSINESS IN THE
PHILIPPINES.

SECOND ASSIGNMENT OF ERROR

THE COURT A QUO ERRED WHEN IT RULED THAT THE RECORD IS BEREFT OF ANY EVIDENCE THAT
RESPONDENT STEAMSHIP IS ENGAGED IN INSURANCE BUSINESS.

THIRD ASSIGNMENT OF ERROR


THE COURT A QUO ERRED WHEN IT RULED, THAT RESPONDENT PIONEER NEED NOT SECURE A LICENSE WHEN
CONDUCTING ITS AFFAIR AS AN AGENT/BROKER OF RESPONDENT STEAMSHIP.

FOURTH ASSIGNMENT OF ERROR

THE COURT A QUO ERRED IN NOT REVOKING THE LICENSE OF RESPONDENT PIONEER AND [IN NOT REMOVING]
THE OFFICERS AND DIRECTORS OF RESPONDENT PIONEER.9

Simply, the basic issues before us are (1) Is Steamship Mutual, a P & I Club, engaged in the insurance business in the
Philippines? (2) Does Pioneer need a license as an insurance agent/broker for Steamship Mutual?

The parties admit that Steamship Mutual is a P & I Club. Steamship Mutual admits it does not have a license to do business
in the Philippines although Pioneer is its resident agent. This relationship is reflected in the certifications issued by the
Insurance Commission.

Petitioner insists that Steamship Mutual as a P & I Club is engaged in the insurance business. To buttress its assertion, it
cites the definition of a P & I Club in Hyopsung Maritime Co., Ltd. v. Court of Appeals 10 as "an association composed of
shipowners in general who band together for the specific purpose of providing insurance cover on a mutual basis against
liabilities incidental to shipowning that the members incur in favor of third parties." It stresses that as a P & I Club, Steamship
Mutuals primary purpose is to solicit and provide protection and indemnity coverage and for this purpose, it has engaged the
services of Pioneer to act as its agent.

Respondents contend that although Steamship Mutual is a P & I Club, it is not engaged in the insurance business in the
Philippines. It is merely an association of vessel owners who have come together to provide mutual protection against
liabilities incidental to shipowning.11 Respondents aver Hyopsung is inapplicable in this case because the issue
in Hyopsung was the jurisdiction of the court over Hyopsung.

Is Steamship Mutual engaged in the insurance business?

Section 2(2) of the Insurance Code enumerates what constitutes "doing an insurance business" or "transacting an insurance
business". These are:

(a) making or proposing to make, as insurer, any insurance contract;

(b) making, or proposing to make, as surety, any contract of suretyship as a vocation and not as merely incidental to any
other legitimate business or activity of the surety;

(c) doing any kind of business, including a reinsurance business, specifically recognized as constituting the doing of an
insurance business within the meaning of this Code;

(d) doing or proposing to do any business in substance equivalent to any of the foregoing in a manner designed to evade the
provisions of this Code.

...

The same provision also provides, the fact that no profit is derived from the making of insurance contracts, agreements or
transactions, or that no separate or direct consideration is received therefor, shall not preclude the existence of an insurance
business.12

The test to determine if a contract is an insurance contract or not, depends on the nature of the promise, the act required to
be performed, and the exact nature of the agreement in the light of the occurrence, contingency, or circumstances under
which the performance becomes requisite. It is not by what it is called. 13

Basically, an insurance contract is a contract of indemnity. In it, one undertakes for a consideration to indemnify another
against loss, damage or liability arising from an unknown or contingent event. 14

In particular, a marine insurance undertakes to indemnify the assured against marine losses, such as the losses incident to a
marine adventure.15 Section 9916 of the Insurance Code enumerates the coverage of marine insurance.
Relatedly, a mutual insurance company is a cooperative enterprise where the members are both the insurer and insured. In
it, the members all contribute, by a system of premiums or assessments, to the creation of a fund from which all losses and
liabilities are paid, and where the profits are divided among themselves, in proportion to their interest. 17 Additionally, mutual
insurance associations, or clubs, provide three types of coverage, namely, protection and indemnity, war risks, and defense
costs.18

A P & I Club is "a form of insurance against third party liability, where the third party is anyone other than the P & I Club and
the members."19 By definition then, Steamship Mutual as a P & I Club is a mutual insurance association engaged in the
marine insurance business.

The records reveal Steamship Mutual is doing business in the country albeit without the requisite certificate of authority
mandated by Section 18720 of the Insurance Code. It maintains a resident agent in the Philippines to solicit insurance and to
collect payments in its behalf. We note that Steamship Mutual even renewed its P & I Club cover until it was cancelled due to
non-payment of the calls. Thus, to continue doing business here, Steamship Mutual or through its agent Pioneer, must
secure a license from the Insurance Commission.

Since a contract of insurance involves public interest, regulation by the State is necessary. Thus, no insurer or insurance
company is allowed to engage in the insurance business without a license or a certificate of authority from the Insurance
Commission.21

Does Pioneer, as agent/broker of Steamship Mutual, need a special license?

Pioneer is the resident agent of Steamship Mutual as evidenced by the certificate of registration 22 issued by the Insurance
Commission. It has been licensed to do or transact insurance business by virtue of the certificate of authority 23 issued by the
same agency. However, a Certification from the Commission states that Pioneer does not have a separate license to be an
agent/broker of Steamship Mutual.24

Although Pioneer is already licensed as an insurance company, it needs a separate license to act as insurance agent for
Steamship Mutual. Section 299 of the Insurance Code clearly states:

SEC. 299 . . .

No person shall act as an insurance agent or as an insurance broker in the solicitation or procurement of applications for
insurance, or receive for services in obtaining insurance, any commission or other compensation from any insurance
company doing business in the Philippines or any agent thereof, without first procuring a license so to act from the
Commissioner, which must be renewed annually on the first day of January, or within six months thereafter. . .

Finally, White Gold seeks revocation of Pioneers certificate of authority and removal of its directors and officers. Regrettably,
we are not the forum for these issues.

WHEREFORE, the petition is PARTIALLY GRANTED. The Decision dated July 30, 2002 of the Court of Appeals affirming the
Decision dated May 3, 2000 of the Insurance Commission is hereby REVERSED AND SET ASIDE. The Steamship Mutual
Underwriting Association (Bermuda) Ltd., and Pioneer Insurance and Surety Corporation are ORDERED to obtain licenses
and to secure proper authorizations to do business as insurer and insurance agent, respectively. The petitioners prayer for
the revocation of Pioneers Certificate of Authority and removal of its directors and officers, is DENIED. Costs against
respondents.

SO ORDERED.

Davide, Jr., C.J., (Chairman), Ynares-Santiago, Carpio, and Azcuna, JJ., concur.
G.R. No. 75605 January 22, 1993

RAFAEL (REX) VERENDIA, petitioner,


vs.
COURT OF APPEALS and FIDELITY & SURETY CO. OF THE PHILIPPINES, respondents.

G.R. No. 76399 January 22, 1993

FIDELITY & SURETY CO. OF THE PHILIPPINES, INC., petitioner,


vs.
RAFAEL VERENDIA and THE COURT OF APPEALS, respondents.

B.L. Padilla for petitioner.

Sabino Padilla, Jr. for Fidelity & Surety, Co.

MELO, J.:

The two consolidated cases involved herein stemmed from the issuance by Fidelity and Surety Insurance Company of the
Philippines (Fidelity for short) of its Fire Insurance Policy No. F-18876 effective between June 23, 1980 and June 23, 1981
covering Rafael (Rex) Verendia's residential building located at Tulip Drive, Beverly Hills, Antipolo, Rizal in the amount of
P385,000.00. Designated as beneficiary was the Monte de Piedad & Savings Bank. Verendia also insured the same building
with two other companies, namely, The Country Bankers Insurance for P56,000.00 under Policy No. PDB-80-1913 expiring
on May 12, 1981, and The Development Insurance for P400,000.00 under Policy No. F-48867 expiring on June 30, 198l.

While the three fire insurance policies were in force, the insured property was completely destroyed by fire on the early
morning of December 28, 1980. Fidelity was accordingly informed of the loss and despite demands, refused payment under
its policy, thus prompting Verendia to file a complaint with the then Court of First Instance of Quezon City, praying for
payment of P385,000.00, legal interest thereon, plus attorney's fees and litigation expenses. The complaint was later
amended to include Monte de Piedad as an "unwilling defendant" (P. 16, Record).

Answering the complaint, Fidelity, among other things, averred that the policy was avoided by reason of over-insurance; that
Verendia maliciously represented that the building at the time of the fire was leased under a contract executed on June 25,
1980 to a certain Roberto Garcia, when actually it was a Marcelo Garcia who was the lessee.

On May 24, 1983, the trial court rendered a decision, per Judge Rodolfo A. Ortiz, ruling in favor of Fidelity. In sustaining the
defenses set up by Fidelity, the trial court ruled that Paragraph 3 of the policy was also violated by Verendia in that the
insured failed to inform Fidelity of his other insurance coverages with Country Bankers Insurance and Development
Insurance.

Verendia appealed to the then Intermediate Appellate Court and in a decision promulgated on March 31, 1986, (CA-G.R. No.
CV No. 02895, Coquia, Zosa, Bartolome, and Ejercito (P), JJ.), the appellate court reversed for the following reasons: (a)
there was no misrepresentation concerning the lease for the contract was signed by Marcelo Garcia in the name of Roberto
Garcia; and (b) Paragraph 3 of the policy contract requiring Verendia to give notice to Fidelity of other contracts of insurance
was waived by Fidelity as shown by its conduct in attempting to settle the claim of Verendia (pp. 32-33, Rollo of G.R. No.
76399).

Fidelity received a copy of the appellate court's decision on April 4, 1986, but instead of directly filing a motion for
reconsideration within 15 days therefrom, Fidelity filed on April 21, 1986, a motion for extension of 3 days within which to file
a motion for reconsideration. The motion for extension was not filed on April 19, 1986 which was the 15th day after receipt of
the decision because said 15th day was a Saturday and of course, the following day was a Sunday (p. 14., Rollo of G.R. No.
75605). The motion for extension was granted by the appellate court on April 30, 1986 (p. 15. ibid.), but Fidelity had in the
meantime filed its motion for reconsideration on April 24, 1986 (p. 16, ibid.).

Verendia filed a motion to expunge from the record Fidelity's motion for reconsideration on the ground that the motion for
extension was filed out of time because the 15th day from receipt of the decision which fell on a Saturday was ignored by
Fidelity, for indeed, so Verendia contended, the Intermediate Appellate Court has personnel receiving pleadings even on
Saturdays.
The motion to expunge was denied on June 17, 1986 (p. 27, ibid.) and after a motion for reconsideration was similarly
brushed aside on July 22, 1986 (p. 30, ibid .), the petition herein docketed as G.R. No. 75605 was initiated. Subsequently, or
more specifically on October 21, 1986, the appellate court denied Fidelity's motion for reconsideration and account thereof.
Fidelity filed on March 31, 1986, the petition for review on certiorari now docketed as G.R. No. 76399. The two petitions,
inter-related as they are, were consolidated
(p. 54, Rollo of G.R. No. 76399) and thereafter given due course.

Before we can even begin to look into the merits of the main case which is the petition for review on certiorari, we must first
determine whether the decision of the appellate court may still be reviewed, or whether the same is beyond further judicial
scrutiny. Stated otherwise, before anything else, inquiry must be made into the issue of whether Fidelity could have legally
asked for an extension of the 15-day reglementary period for appealing or for moving for reconsideration.

As early as 1944, this Court through Justice Ozaeta already pronounced the doctrine that the pendency of a motion for
extension of time to perfect an appeal does not suspend the running of the period sought to be extended (Garcia vs.
Buenaventura 74 Phil. 611 [1944]). To the same effect were the rulings in Gibbs vs. CFI of Manila (80 Phil. 160 [1948]) Bello
vs. Fernando (4 SCRA 138 [1962]), and Joe vs. King(20 SCRA 1120 [1967]).

The above cases notwithstanding and because the Rules of Court do not expressly prohibit the filing of a motion for
extension of time to file a motion for reconsideration in regard to a final order or judgment, magistrates, including those in the
Court of Appeals, held sharply divided opinions on whether the period for appealing which also includes the period for
moving to reconsider may be extended. The matter was not definitely settled until this Court issued its Resolution
in Habaluyas Enterprises, Inc. vs. Japson (142 SCRA [1986]), declaring that beginning one month from the promulgation of
the resolution on May 30, 1986

. . . the rule shall be strictly enforced that no motion for extension of time to file a motion for new trial or
reconsideration shall be filed . . . (at p. 212.)

In the instant case, the motion for extension was filed and granted before June 30, 1986, although, of course, Verendia's
motion to expunge the motion for reconsideration was not finally disposed until July 22, 1986, or after the dictum
in Habaluyas had taken effect. Seemingly, therefore, the filing of the motion for extension came before its formal proscription
under Habaluyas, for which reason we now turn our attention to G.R. No. 76399.

Reduced to bare essentials, the issues Fidelity raises therein are: (a) whether or not the contract of lease submitted by
Verendia to support his claim on the fire insurance policy constitutes a false declaration which would forfeit his benefits under
Section 13 of the policy and (b) whether or not, in submitting the subrogation receipt in evidence, Fidelity had in effect agreed
to settle Verendia's claim in the amount stated in said receipt. 1

Verging on the factual, the issue of the veracity or falsity of the lease contract could have been better resolved by the
appellate court for, in a petition for review on certiorari under Rule 45, the jurisdiction of this Court is limited to the review of
errors of law. The appellate court's findings of fact are, therefore, conclusive upon this Court except in the following cases: (1)
when the conclusion is a finding grounded entirely on speculation, surmises, or conjectures; (2) when the inference made is
manifestly absurd, mistaken, or impossible; (3) when there is grave abuse of discretion in the appreciation of facts; (4) when
the judgment is premised on a misapprehension of facts; (5) when the findings of fact are conflicting; and (6) when the Court
of Appeals in making its findings went beyond the issues of the case and the same are contrary to the admissions of both
appellant and appellee (Ronquillo v. Court of Appeals, 195 SCRA 433 [1991]). In view of the conflicting findings of the trial
court and the appellate court on important issues in these consolidated cases and it appearing that the appellate court
judgment is based on a misapprehension of facts, this Court shall review the evidence on record.

The contract of lease upon which Verendia relies to support his claim for insurance benefits, was entered into between him
and one Robert Garcia, married to Helen Cawinian, on June 25, 1980 (Exh. "1"), a couple of days after the effectivity of the
insurance policy. When the rented residential building was razed to the ground on December 28, 1980, it appears that Robert
Garcia (or Roberto Garcia) was still within the premises. However, according to the investigation report prepared by Pat.
Eleuterio M. Buenviaje of the Antipolo police, the building appeared to have "no occupant" and that Mr. Roberto Garcia was
"renting on the otherside (sic) portion of said compound"
(Exh. "E"). These pieces of evidence belie Verendia's uncorroborated testimony that Marcelo Garcia, whom he considered as
the real lessee, was occupying the building when it was burned (TSN, July 27, 1982, p.10).

Robert Garcia disappeared after the fire. It was only on October 9, 1981 that an adjuster was able to locate him. Robert
Garcia then executed an affidavit before the National Intelligence and Security Authority (NISA) to the effect that he was not
the lessee of Verendia's house and that his signature on the contract of lease was a complete forgery. Thus, on the strength
of these facts, the adjuster submitted a report dated December 4, 1981 recommending the denial of Verendia's claim (Exh.
"2").
Ironically, during the trial, Verendia admitted that it was not Robert Garcia who signed the lease contract. According to
Verendia, it was signed by Marcelo Garcia, cousin of Robert, who had been paying the rentals all the while. Verendia,
however, failed to explain why Marcelo had to sign his cousin's name when he in fact was paying for the rent and why he
(Verendia) himself, the lessor, allowed such a ruse. Fidelity's conclusions on these proven facts appear, therefore, to have
sufficient bases; Verendia concocted the lease contract to deflect responsibility for the fire towards an alleged "lessee",
inflated the value of the property by the alleged monthly rental of P6,500 when in fact, the Provincial Assessor of Rizal had
assessed the property's fair market value to be only P40,300.00, insured the same property with two other insurance
companies for a total coverage of around P900,000, and created a dead-end for the adjuster by the disappearance of Robert
Garcia.

Basically a contract of indemnity, an insurance contract is the law between the parties (Pacific Banking Corporation vs. Court
of Appeals 168 SCRA 1 [1988]). Its terms and conditions constitute the measure of the insurer's liability and compliance
therewith is a condition precedent to the insured's right to recovery from the insurer (Oriental Assurance Corporation vs.
Court of Appeals, 200 SCRA 459 [1991], citing Perla Compania de Seguros, Inc. vs. Court of Appeals, 185 SCRA 741
[1991]). As it is also a contract of adhesion, an insurance contract should be liberally construed in favor of the insured and
strictly against the insurer company which usually prepares it (Western Guaranty Corporation vs. Court of Appeals, 187
SCRA 652 [1980]).

Considering, however, the foregoing discussion pointing to the fact that Verendia used a false lease contract to support his
claim under Fire Insurance Policy No. F-18876, the terms of the policy should be strictly construed against the insured.
Verendia failed to live by the terms of the policy, specifically Section 13 thereof which is expressed in terms that are clear and
unambiguous, that all benefits under the policy shall be forfeited "If the claim be in any respect fraudulent, or if any false
declaration be made or used in support thereof, or if any fraudulent means or devises are used by the Insured or anyone
acting in his behalf to obtain any benefit under the policy". Verendia, having presented a false declaration to support his claim
for benefits in the form of a fraudulent lease contract, he forfeited all benefits therein by virtue of Section 13 of the policy in
the absence of proof that Fidelity waived such provision (Pacific Banking Corporation vs. Court of Appeals , supra). Worse
yet, by presenting a false lease contract, Verendia, reprehensibly disregarded the principle that insurance contracts
are uberrimae fidae and demand the most abundant good faith (Velasco vs. Apostol, 173 SCRA 228 [1989]).

There is also no reason to conclude that by submitting the subrogation receipt as evidence in court, Fidelity bound itself to a
"mutual agreement" to settle Verendia's claims in consideration of the amount of P142,685.77. While the said receipt
appears to have been a filled-up form of Fidelity, no representative of Fidelity had signed it. It is even incomplete as the blank
spaces for a witness and his address are not filled up. More significantly, the same receipt states that Verendia had received
the aforesaid amount. However, that Verendia had not received the amount stated therein, is proven by the fact that Verendia
himself filed the complaint for the full amount of P385,000.00 stated in the policy. It might be that there had been efforts to
settle Verendia's claims, but surely, the subrogation receipt by itself does not prove that a settlement had been arrived at and
enforced. Thus, to interpret Fidelity's presentation of the subrogation receipt in evidence as indicative of its accession to its
"terms" is not only wanting in rational basis but would be substituting the will of the Court for that of the parties.

WHEREFORE, the petition in G.R. No. 75605 is DISMISSED. The petition in G.R. No. 76399 is GRANTED and the decision
of the then Intermediate Appellate Court under review is REVERSED and SET ASIDE and that of the trial court is hereby
REINSTATED and UPHELD.

SO ORDERED.

Gutierrez, Jr., Bidin, Davide, Jr. and Romero, JJ., concur.

# Footnotes

1 Fidelity appears to have agreed with the appellate court that it had waived Verendia's failure to abide by
policy condition No. 3 on disclosure of other insurance policies by its failure to assign it as an error in the
petition in G.R. No. 76399. It must have likewise realized the futility of assigning it as an error because on
the first page of the policy the following is typewritten: "Other insurances allowed, the amounts to be
declared in the event of loss or when required."
G.R. No. 112360 July 18, 2000
RIZAL SURETY & INSURANCE COMPANY, petitioner,
vs.
COURT OF APPEALS and TRANSWORLD KNITTING MILLS, INC., respondents.
DECISION
PURISIMA, J.:

At bar is a Petition for Review on Certiorari under Rule 45 of the Rules of Court seeking to annul and set aside the July 15,
1993 Decision1 and October 22, 1993 Resolution2 of the Court of Appeals3 in CA-G.R. CV NO. 28779, which modified the
Ruling4 of the Regional Trial Court of Pasig, Branch 161, in Civil Case No. 46106.

The antecedent facts that matter are as follows:

On March 13, 1980, Rizal Surety & Insurance Company (Rizal Insurance) issued Fire Insurance Policy No. 45727 in favor of
Transworld Knitting Mills, Inc. (Transworld), initially for One Million (P1,000,000.00) Pesos and eventually increased to One
Million Five Hundred Thousand (P1,500,000.00) Pesos, covering the period from August 14, 1980 to March 13, 1981.

Pertinent portions of subject policy on the buildings insured, and location thereof, read:

"On stocks of finished and/or unfinished products, raw materials and supplies of every kind and description, the properties of
the Insureds and/or held by them in trust, on commission or on joint account with others and/or for which they (sic)
responsible in case of loss whilst contained and/or stored during the currency of this Policy in the premises occupied by them
forming part of the buildings situate (sic) within own Compound at MAGDALO STREET, BARRIO UGONG, PASIG, METRO
MANILA, PHILIPPINES, BLOCK NO. 601.
xxx xxx xxx
Said building of four-span lofty one storey in height with mezzanine portions is constructed of reinforced concrete and hollow
blocks and/or concrete under galvanized iron roof and occupied as hosiery mills, garment and lingerie factory, transistor-
stereo assembly plant, offices, warehouse and caretaker's quarters.

'Bounds in front partly by one-storey concrete building under galvanized iron roof occupied as canteen and guardhouse,
partly by building of two and partly one storey constructed of concrete below, timber above undergalvanized iron roof
occupied as garage and quarters and partly by open space and/or tracking/ packing, beyond which is the aforementioned
Magdalo Street; on its right and left by driveway, thence open spaces, and at the rear by open spaces.'" 5

The same pieces of property insured with the petitioner were also insured with New India Assurance Company, Ltd., (New
India).

On January 12, 1981, fire broke out in the compound of Transworld, razing the middle portion of its four-span building and
partly gutting the left and right sections thereof. A two-storey building (behind said four-span building) where fun and
amusement machines and spare parts were stored, was also destroyed by the fire.

Transworld filed its insurance claims with Rizal Surety & Insurance Company and New India Assurance Company but to no
avail.

On May 26, 1982, private respondent brought against the said insurance companies an action for collection of sum of money
and damages, docketed as Civil Case No. 46106 before Branch 161 of the then Court of First Instance of Rizal; praying for
judgment ordering Rizal Insurance and New India to pay the amount of P2,747, 867.00 plus legal interest, P400,000.00 as
attorney's fees, exemplary damages, expenses of litigation ofP50,000.00 and costs of suit.6

Petitioner Rizal Insurance countered that its fire insurance policy sued upon covered only the contents of the four-span
building, which was partly burned, and not the damage caused by the fire on the two-storey annex building. 7

On January 4, 1990, the trial court rendered its decision; disposing as follows:

"ACCORDINGLY, judgment is hereby rendered as follows:

(1)Dismissing the case as against The New India Assurance Co., Ltd.;

(2) Ordering defendant Rizal Surety And Insurance Company to pay Transwrold (sic) Knitting Mills, Inc. the amount
of P826, 500.00 representing the actual value of the losses suffered by it; and
(3) Cost against defendant Rizal Surety and Insurance Company.

SO ORDERED."8

Both the petitioner, Rizal Insurance Company, and private respondent, Transworld Knitting Mills, Inc., went to the Court of
Appeals, which came out with its decision of July 15, 1993 under attack, the decretal portion of which reads:

"WHEREFORE, and upon all the foregoing, the decision of the court below is MODIFIED in that defendant New India
Assurance Company has and is hereby required to pay plaintiff-appellant the amount of P1,818,604.19 while the other Rizal
Surety has to pay the plaintiff-appellant P470,328.67, based on the actual losses sustained by plaintiff Transworld in the fire,
totalling P2,790,376.00 as against the amounts of fire insurance coverages respectively extended by New India in the
amount of P5,800,000.00 and Rizal Surety and Insurance Company in the amount of P1,500,000.00.

No costs.

SO ORDERED."9

On August 20, 1993, from the aforesaid judgment of the Court of Appeals New India appealed to this Court theorizing inter
alia that the private respondent could not be compensated for the loss of the fun and amusement machines and spare parts
stored at the two-storey building because it (Transworld) had no insurable interest in said goods or items.

On February 2, 1994, the Court denied the appeal with finality in G.R. No. L-111118 (New India Assurance Company Ltd. vs.
Court of Appeals).

Petitioner Rizal Insurance and private respondent Transworld, interposed a Motion for Reconsideration before the Court of
Appeals, and on October 22, 1993, the Court of Appeals reconsidered its decision of July 15, 1993, as regards the imposition
of interest, ruling thus:

"WHEREFORE, the Decision of July 15, 1993 is amended but only insofar as the imposition of legal interest is concerned,
that, on the assessment against New India Assurance Company on the amount of P1,818,604.19 and that against Rizal
Surety & Insurance Company on the amount of P470,328.67, from May 26, 1982 when the complaint was filed until payment
is made. The rest of the said decision is retained in all other respects.

SO ORDERED."10

Undaunted, petitioner Rizal Surety & Insurance Company found its way to this Court via the present Petition, contending that:

I.....SAID DECISION (ANNEX A) ERRED IN ASSUMING THAT THE ANNEX BUILDING WHERE THE BULK OF
THE BURNED PROPERTIES WERE STORED, WAS INCLUDED IN THE COVERAGE OF THE INSURANCE
POLICY ISSUED BY RIZAL SURETY TO TRANSWORLD.

II.....SAID DECISION AND RESOLUTION (ANNEXES A AND B) ERRED IN NOT CONSIDERING THE PICTURES
(EXHS. 3 TO 7-C-RIZAL SURETY), TAKEN IMMEDIATELY AFTER THE FIRE, WHICH CLEARLY SHOW THAT
THE PREMISES OCCUPIED BY TRANSWORLD, WHERE THE INSURED PROPERTIES WERE LOCATED,
SUSTAINED PARTIAL DAMAGE ONLY.

III. SAID DECISION (ANNEX A) ERRED IN NOT HOLDING THAT TRANSWORLD HAD ACTED IN PALPABLE BAD
FAITH AND WITH MALICE IN FILING ITS CLEARLY UNFOUNDED CIVIL ACTION, AND IN NOT ORDERING
TRANSWORLD TO PAY TO RIZAL SURETY MORAL AND PUNITIVE DAMAGES (ART. 2205, CIVIL CODE), PLUS
ATTORNEY'S FEES AND EXPENSES OF LITIGATION (ART. 2208 PARS. 4 and 11, CIVIL CODE). 11

The Petition is not impressed with merit.

It is petitioner's submission that the fire insurance policy litigated upon protected only the contents of the main building (four-
span),12 and did not include those stored in the two-storey annex building. On the other hand, the private respondent
theorized that the so called "annex" was not an annex but was actually an integral part of the four-span building 13 and
therefore, the goods and items stored therein were covered by the same fire insurance policy.
Resolution of the issues posited here hinges on the proper interpretation of the stipulation in subject fire insurance policy
regarding its coverage, which reads:

"xxx contained and/or stored during the currency of this Policy in the premises occupied by them forming part of the buildings
situate (sic) within own Compound xxx"

Therefrom, it can be gleaned unerringly that the fire insurance policy in question did not limit its coverage to what were stored
in the four-span building. As opined by the trial court of origin, two requirements must concur in order that the said fun and
amusement machines and spare parts would be deemed protected by the fire insurance policy under scrutiny, to wit:

"First, said properties must be contained and/or stored in the areas occupied by Transworld and second, said areas must
form part of the building described in the policy xxx" 14

'Said building of four-span lofty one storey in height with mezzanine portions is constructed of reinforced concrete and hollow
blocks and/or concrete under galvanized iron roof and occupied as hosiery mills, garment and lingerie factory, transistor-
stereo assembly plant, offices, ware house and caretaker's quarter.'

The Court is mindful of the well-entrenched doctrine that factual findings by the Court of Appeals are conclusive on the
parties and not reviewable by this Court, and the same carry even more weight when the Court of Appeals has affirmed the
findings of fact arrived at by the lower court.15

In the case under consideration, both the trial court and the Court of Appeals found that the so called "annex " was not an
annex building but an integral and inseparable part of the four-span building described in the policy and consequently, the
machines and spare parts stored therein were covered by the fire insurance in dispute. The letter-report of the Manila
Adjusters and Surveyor's Company, which petitioner itself cited and invoked, describes the "annex" building as follows:

"Two-storey building constructed of partly timber and partly concrete hollow blocks under g.i. roof which is adjoining and
intercommunicating with the repair of the first right span of the lofty storey building and thence by property fence wall." 16

Verily, the two-storey building involved, a permanent structure which adjoins and intercommunicates with the "first right span
of the lofty storey building",17 formed part thereof, and meets the requisites for compensability under the fire insurance policy
sued upon.

So also, considering that the two-storey building aforementioned was already existing when subject fire insurance policy
contract was entered into on January 12, 1981, having been constructed sometime in 1978, 18 petitioner should have
specifically excluded the said two-storey building from the coverage of the fire insurance if minded to exclude the same but if
did not, and instead, went on to provide that such fire insurance policy covers the products, raw materials and supplies
stored within the premises of respondent Transworld which was an integral part of the four-span building occupied by
Transworld, knowing fully well the existence of such building adjoining and intercommunicating with the right section of the
four-span building.

After a careful study, the Court does not find any basis for disturbing what the lower courts found and arrived at.

Indeed, the stipulation as to the coverage of the fire insurance policy under controversy has created a doubt regarding the
portions of the building insured thereby. Article 1377 of the New Civil Code provides:

"Art.1377. The interpretation of obscure words or stipulations in a contract shall not favor the party who caused the obscurity"

Conformably, it stands to reason that the doubt should be resolved against the petitioner, Rizal Surety Insurance Company,
whose lawyer or managers drafted the fire insurance policy contract under scrutiny. Citing the aforecited provision of law in
point, the Court in Landicho vs. Government Service Insurance System,19 ruled:

"This is particularly true as regards insurance policies, in respect of which it is settled that the 'terms in an insurance policy,
which are ambiguous, equivocal, or uncertain x x x are to be construed strictly and most strongly against the insurer, and
liberally in favor of the insured so as to effect the dominant purpose of indemnity or payment to the insured, especially where
forfeiture is involved' (29 Am. Jur., 181), and the reason for this is that the 'insured usually has no voice in the selection or
arrangement of the words employed and that the language of the contract is selected with great care and deliberation by
experts and legal advisers employed by, and acting exclusively in the interest of, the insurance company.' (44 C.J.S., p.
1174).""20
Equally relevant is the following disquisition of the Court in Fieldmen's Insurance Company, Inc. vs. Vda. De Songco,21 to wit:

"'This rigid application of the rule on ambiguities has become necessary in view of current business practices. The courts
1wphi1

cannot ignore that nowadays monopolies, cartels and concentration of capital, endowed with overwhelming economic power,
manage to impose upon parties dealing with them cunningly prepared 'agreements' that the weaker party may not change
one whit, his participation in the 'agreement' being reduced to the alternative to 'take it or leave it' labelled since Raymond
Saleilles 'contracts by adherence' (contrats [sic] d'adhesion), in contrast to these entered into by parties bargaining on an
equal footing, such contracts (of which policies of insurance and international bills of lading are prime example) obviously call
for greater strictness and vigilance on the part of courts of justice with a view to protecting the weaker party from abuses and
imposition, and prevent their becoming traps for the unwary (New Civil Code, Article 24; Sent. of Supreme Court of Spain, 13
Dec. 1934, 27 February 1942.)'"22

The issue of whether or not Transworld has an insurable interest in the fun and amusement machines and spare parts, which
entitles it to be indemnified for the loss thereof, had been settled in G.R. No. L-111118, entitled New India Assurance
Company, Ltd., vs. Court of Appeals, where the appeal of New India from the decision of the Court of Appeals under review,
was denied with finality by this Court on February 2, 1994.

The rule on conclusiveness of judgment, which obtains under the premises, precludes the relitigation of a particular fact or
issue in another action between the same parties based on a different claim or cause of action. "xxx the judgment in the prior
action operates as estoppel only as to those matters in issue or points controverted, upon the determination of which the
finding or judgment was rendered. In fine, the previous judgment is conclusive in the second case, only as those matters
actually and directly controverted and determined and not as to matters merely involved therein." 23

Applying the abovecited pronouncement, the Court, in Smith Bell and Company (Phils.), Inc. vs. Court of Appeals, 24 held that
the issue of negligence of the shipping line, which issue had already been passed upon in a case filed by one of the insurers,
is conclusive and can no longer be relitigated in a similar case filed by another insurer against the same shipping line on the
basis of the same factual circumstances. Ratiocinating further, the Court opined:

"In the case at bar, the issue of which vessel ('Don Carlos' or 'Yotai Maru') had been negligent, or so negligent as to have
proximately caused the collision between them, was an issue that was actually, directly and expressly raised, controverted
and litigated in C.A.-G.R. No. 61320-R. Reyes, L.B., J., resolved that issue in his Decision and held the 'Don Carlos' to have
been negligent rather than the 'Yotai Maru' and, as already noted, that Decision was affirmed by this Court in G.R. No. L-
48839 in a Resolution dated 6 December 1987. The Reyes Decision thus became final and executory approximately two (2)
years before the Sison Decision, which is assailed in the case at bar, was promulgated. Applying the rule of conclusiveness
of judgment, the question of which vessel had been negligent in the collision between the two (2) vessels, had long been
settled by this Court and could no longer be relitigated in C.A.-G.R. No. 61206-R. Private respondent Go Thong was
certainly bound by the ruling or judgment of Reyes, L.B., J. and that of this Court. The Court of Appeals fell into clear and
reversible error when it disregarded the Decision of this Court affirming the Reyes Decision." 25

The controversy at bar is on all fours with the aforecited case. Considering that private respondent's insurable interest in, and
compensability for the loss of subject fun and amusement machines and spare parts, had been adjudicated, settled and
sustained by the Court of Appeals in CA-G.R. CV NO. 28779, and by this Court in G.R. No. L-111118, in a Resolution, dated
February 2, 1994, the same can no longer be relitigated and passed upon in the present case. Ineluctably, the petitioner,
Rizal Surety Insurance Company, is bound by the ruling of the Court of Appeals and of this Court that the private respondent
has an insurable interest in the aforesaid fun and amusement machines and spare parts; and should be indemnified for the
loss of the same.

So also, the Court of Appeals correctly adjudged petitioner liable for the amount of P470,328.67, it being the total loss and
damage suffered by Transworld for which petitioner Rizal Insurance is liable. 26

All things studiedly considered and viewed in proper perspective, the Court is of the irresistible conclusion, and so finds, that
the Court of Appeals erred not in holding the petitioner, Rizal Surety Insurance Company, liable for the destruction and loss of
the insured buildings and articles of the private respondent.

WHEREFORE, the Decision, dated July 15, 1993, and the Resolution, dated October 22, 1993, of the Court of Appeals in
CA-G.R. CV NO. 28779 are AFFIRMED in toto. No pronouncement as to costs.

SO ORDERED.

Melo, (Chairman), Vitug, Panganiban, and Gonzaga-Reyes, JJ., concur.


G.R. No. 125678 March 18, 2002

PHILAMCARE HEALTH SYSTEMS, INC., petitioner,


vs.
COURT OF APPEALS and JULITA TRINOS, respondents.

YNARES-SANTIAGO, J.:

Ernani Trinos, deceased husband of respondent Julita Trinos, applied for a health care coverage with petitioner Philamcare
Health Systems, Inc. In the standard application form, he answered no to the following question:

Have you or any of your family members ever consulted or been treated for high blood pressure, heart trouble,
diabetes, cancer, liver disease, asthma or peptic ulcer? (If Yes, give details). 1

The application was approved for a period of one year from March 1, 1988 to March 1, 1989. Accordingly, he was issued
Health Care Agreement No. P010194. Under the agreement, respondents husband was entitled to avail of hospitalization
benefits, whether ordinary or emergency, listed therein. He was also entitled to avail of "out-patient benefits" such as annual
physical examinations, preventive health care and other out-patient services.

Upon the termination of the agreement, the same was extended for another year from March 1, 1989 to March 1, 1990, then
from March 1, 1990 to June 1, 1990. The amount of coverage was increased to a maximum sum of P75,000.00 per
disability.2

During the period of his coverage, Ernani suffered a heart attack and was confined at the Manila Medical Center (MMC) for
one month beginning March 9, 1990. While her husband was in the hospital, respondent tried to claim the benefits under the
health care agreement. However, petitioner denied her claim saying that the Health Care Agreement was void. According to
petitioner, there was a concealment regarding Ernanis medical history. Doctors at the MMC allegedly discovered at the time
of Ernanis confinement that he was hypertensive, diabetic and asthmatic, contrary to his answer in the application form.
Thus, respondent paid the hospitalization expenses herself, amounting to about P76,000.00.

After her husband was discharged from the MMC, he was attended by a physical therapist at home. Later, he was admitted
at the Chinese General Hospital. Due to financial difficulties, however, respondent brought her husband home again. In the
morning of April 13, 1990, Ernani had fever and was feeling very weak. Respondent was constrained to bring him back to the
Chinese General Hospital where he died on the same day.

On July 24, 1990, respondent instituted with the Regional Trial Court of Manila, Branch 44, an action for damages against
petitioner and its president, Dr. Benito Reverente, which was docketed as Civil Case No. 90-53795. She asked for
reimbursement of her expenses plus moral damages and attorneys fees. After trial, the lower court ruled against
petitioners, viz:

WHEREFORE, in view of the forgoing, the Court renders judgment in favor of the plaintiff Julita Trinos, ordering:

1. Defendants to pay and reimburse the medical and hospital coverage of the late Ernani Trinos in the amount of
P76,000.00 plus interest, until the amount is fully paid to plaintiff who paid the same;

2. Defendants to pay the reduced amount of moral damages of P10,000.00 to plaintiff;

3. Defendants to pay the reduced amount of P10,000.00 as exemplary damages to plaintiff;

4. Defendants to pay attorneys fees of P20,000.00, plus costs of suit.

SO ORDERED.3

On appeal, the Court of Appeals affirmed the decision of the trial court but deleted all awards for damages and absolved
petitioner Reverente.4 Petitioners motion for reconsideration was denied.5 Hence, petitioner brought the instant petition for
review, raising the primary argument that a health care agreement is not an insurance contract; hence the "incontestability
clause" under the Insurance Code6 does not apply. 1wphi1.nt
Petitioner argues that the agreement grants "living benefits," such as medical check-ups and hospitalization which a member
may immediately enjoy so long as he is alive upon effectivity of the agreement until its expiration one-year thereafter.
Petitioner also points out that only medical and hospitalization benefits are given under the agreement without any
indemnification, unlike in an insurance contract where the insured is indemnified for his loss. Moreover, since Health Care
Agreements are only for a period of one year, as compared to insurance contracts which last longer, 7 petitioner argues that
the incontestability clause does not apply, as the same requires an effectivity period of at least two years. Petitioner further
argues that it is not an insurance company, which is governed by the Insurance Commission, but a Health Maintenance
Organization under the authority of the Department of Health.

Section 2 (1) of the Insurance Code defines a contract of insurance as an agreement whereby one undertakes for a
consideration to indemnify another against loss, damage or liability arising from an unknown or contingent event. An
insurance contract exists where the following elements concur:

1. The insured has an insurable interest;

2. The insured is subject to a risk of loss by the happening of the designated peril;

3. The insurer assumes the risk;

4. Such assumption of risk is part of a general scheme to distribute actual losses among a large group of persons
bearing a similar risk; and

5. In consideration of the insurers promise, the insured pays a premium. 8

Section 3 of the Insurance Code states that any contingent or unknown event, whether past or future, which may damnify a
person having an insurable interest against him, may be insured against. Every person has an insurable interest in the life
and health of himself. Section 10 provides:

Every person has an insurable interest in the life and health:

(1) of himself, of his spouse and of his children;

(2) of any person on whom he depends wholly or in part for education or support, or in whom he has a pecuniary
interest;

(3) of any person under a legal obligation to him for the payment of money, respecting property or service, of which
death or illness might delay or prevent the performance; and

(4) of any person upon whose life any estate or interest vested in him depends.

In the case at bar, the insurable interest of respondents husband in obtaining the health care agreement was his own health.
The health care agreement was in the nature of non-life insurance, which is primarily a contract of indemnity. 9 Once the
member incurs hospital, medical or any other expense arising from sickness, injury or other stipulated contingent, the health
care provider must pay for the same to the extent agreed upon under the contract.

Petitioner argues that respondents husband concealed a material fact in his application. It appears that in the application for
health coverage, petitioners required respondents husband to sign an express authorization for any person, organization or
entity that has any record or knowledge of his health to furnish any and all information relative to any hospitalization,
consultation, treatment or any other medical advice or examination. 10 Specifically, the Health Care Agreement signed by
respondents husband states:

We hereby declare and agree that all statement and answers contained herein and in any addendum annexed to
this application are full, complete and true and bind all parties in interest under the Agreement herein applied for,
that there shall be no contract of health care coverage unless and until an Agreement is issued on this application
and the full Membership Fee according to the mode of payment applied for is actually paid during the lifetime and
good health of proposed Members; that no information acquired by any Representative of PhilamCare shall be
binding upon PhilamCare unless set out in writing in the application;that any physician is, by these presents,
expressly authorized to disclose or give testimony at anytime relative to any information acquired by him in his
professional capacity upon any question affecting the eligibility for health care coverage of the Proposed
Members and that the acceptance of any Agreement issued on this application shall be a ratification of any
correction in or addition to this application as stated in the space for Home Office Endorsement. 11 (Underscoring
ours)

In addition to the above condition, petitioner additionally required the applicant for authorization to inquire about the
applicants medical history, thus:

I hereby authorize any person, organization, or entity that has any record or knowledge of my health and/or that of
__________ to give to the PhilamCare Health Systems, Inc. any and all information relative to any hospitalization,
consultation, treatment or any other medical advice or examination. This authorization is in connection with the
application for health care coverage only. A photographic copy of this authorization shall be as valid as the
original.12 (Underscoring ours)

Petitioner cannot rely on the stipulation regarding "Invalidation of agreement" which reads:

Failure to disclose or misrepresentation of any material information by the member in the application or medical
examination, whether intentional or unintentional, shall automatically invalidate the Agreement from the very
beginning and liability of Philamcare shall be limited to return of all Membership Fees paid. An undisclosed or
misrepresented information is deemed material if its revelation would have resulted in the declination of the applicant
by Philamcare or the assessment of a higher Membership Fee for the benefit or benefits applied for. 13

The answer assailed by petitioner was in response to the question relating to the medical history of the applicant. This largely
depends on opinion rather than fact, especially coming from respondents husband who was not a medical doctor. Where
matters of opinion or judgment are called for, answers made in good faith and without intent to deceive will not avoid a policy
even though they are untrue.14 Thus,

(A)lthough false, a representation of the expectation, intention, belief, opinion, or judgment of the insured will not
avoid the policy if there is no actual fraud in inducing the acceptance of the risk, or its acceptance at a lower rate of
premium, and this is likewise the rule although the statement is material to the risk, if the statement is obviously of
the foregoing character, since in such case the insurer is not justified in relying upon such statement, but is obligated
to make further inquiry. There is a clear distinction between such a case and one in which the insured is fraudulently
and intentionally states to be true, as a matter of expectation or belief, that which he then knows, to be actually
untrue, or the impossibility of which is shown by the facts within his knowledge, since in such case the intent to
deceive the insurer is obvious and amounts to actual fraud. 15 (Underscoring ours)

The fraudulent intent on the part of the insured must be established to warrant rescission of the insurance
contract.16 Concealment as a defense for the health care provider or insurer to avoid liability is an affirmative defense and the
duty to establish such defense by satisfactory and convincing evidence rests upon the provider or insurer. In any case, with
or without the authority to investigate, petitioner is liable for claims made under the contract. Having assumed a responsibility
under the agreement, petitioner is bound to answer the same to the extent agreed upon. In the end, the liability of the health
care provider attaches once the member is hospitalized for the disease or injury covered by the agreement or whenever he
avails of the covered benefits which he has prepaid.

Under Section 27 of the Insurance Code, "a concealment entitles the injured party to rescind a contract of insurance." The
right to rescind should be exercised previous to the commencement of an action on the contract. 17 In this case, no rescission
was made. Besides, the cancellation of health care agreements as in insurance policies require the concurrence of the
following conditions:

1. Prior notice of cancellation to insured;

2. Notice must be based on the occurrence after effective date of the policy of one or more of the grounds mentioned;

3. Must be in writing, mailed or delivered to the insured at the address shown in the policy;

4. Must state the grounds relied upon provided in Section 64 of the Insurance Code and upon request of insured, to furnish
facts on which cancellation is based.18

None of the above pre-conditions was fulfilled in this case. When the terms of insurance contract contain limitations on
liability, courts should construe them in such a way as to preclude the insurer from non-compliance with his
obligation.19 Being a contract of adhesion, the terms of an insurance contract are to be construed strictly against the party
which prepared the contract the insurer. 20 By reason of the exclusive control of the insurance company over the terms and
phraseology of the insurance contract, ambiguity must be strictly interpreted against the insurer and liberally in favor of the
insured, especially to avoid forfeiture. 21 This is equally applicable to Health Care Agreements. The phraseology used in
medical or hospital service contracts, such as the one at bar, must be liberally construed in favor of the subscriber, and if
doubtful or reasonably susceptible of two interpretations the construction conferring coverage is to be adopted, and
exclusionary clauses of doubtful import should be strictly construed against the provider. 22

Anent the incontestability of the membership of respondents husband, we quote with approval the following findings of the
trial court:

(U)nder the title Claim procedures of expenses, the defendant Philamcare Health Systems Inc. had twelve months
from the date of issuance of the Agreement within which to contest the membership of the patient if he had previous
ailment of asthma, and six months from the issuance of the agreement if the patient was sick of diabetes or
hypertension. The periods having expired, the defense of concealment or misrepresentation no longer lie. 23

Finally, petitioner alleges that respondent was not the legal wife of the deceased member considering that at the time of their
marriage, the deceased was previously married to another woman who was still alive. The health care agreement is in the
nature of a contract of indemnity. Hence, payment should be made to the party who incurred the expenses. It is not
controverted that respondent paid all the hospital and medical expenses. She is therefore entitled to reimbursement. The
records adequately prove the expenses incurred by respondent for the deceaseds hospitalization, medication and the
professional fees of the attending physicians.24

WHEREFORE, in view of the foregoing, the petition is DENIED. The assailed decision of the Court of Appeals dated
December 14, 1995 is AFFIRMED.

SO ORDERED.

Davide, Jr., C.J., Puno, and Kapunan, JJ., concur.


G.R. No. 115278 May 23, 1995

FORTUNE INSURANCE AND SURETY CO., INC., petitioner,


vs.
COURT OF APPEALS and PRODUCERS BANK OF THE PHILIPPINES, respondents.

DAVIDE, JR., J.:

The fundamental legal issue raised in this petition for review on certiorari is whether the petitioner is liable under the Money,
Security, and Payroll Robbery policy it issued to the private respondent or whether recovery thereunder is precluded under
the general exceptions clause thereof. Both the trial court and the Court of Appeals held that there should be recovery. The
petitioner contends otherwise.

This case began with the filing with the Regional Trial Court (RTC) of Makati, Metro Manila, by private respondent Producers
Bank of the Philippines (hereinafter Producers) against petitioner Fortune Insurance and Surety Co., Inc. (hereinafter
Fortune) of a complaint for recovery of the sum of P725,000.00 under the policy issued by Fortune. The sum was allegedly
lost during a robbery of Producer's armored vehicle while it was in transit to transfer the money from its Pasay City Branch to
its head office in Makati. The case was docketed as Civil Case No. 1817 and assigned to Branch 146 thereof.

After joinder of issues, the parties asked the trial court to render judgment based on the following stipulation of facts:

1. The plaintiff was insured by the defendants and an insurance policy was issued, the
duplicate original of which is hereto attached as Exhibit "A";

2. An armored car of the plaintiff, while in the process of transferring cash in the sum of
P725,000.00 under the custody of its teller, Maribeth Alampay, from its Pasay Branch to its
Head Office at 8737 Paseo de Roxas, Makati, Metro Manila on June 29, 1987, was robbed
of the said cash. The robbery took place while the armored car was traveling along Taft
Avenue in Pasay City;

3. The said armored car was driven by Benjamin Magalong Y de Vera, escorted by
Security Guard Saturnino Atiga Y Rosete. Driver Magalong was assigned by PRC
Management Systems with the plaintiff by virtue of an Agreement executed on August 7,
1983, a duplicate original copy of which is hereto attached as Exhibit "B";

4. The Security Guard Atiga was assigned by Unicorn Security Services, Inc. with the
plaintiff by virtue of a contract of Security Service executed on October 25, 1982, a
duplicate original copy of which is hereto attached as Exhibit "C";

5. After an investigation conducted by the Pasay police authorities, the driver Magalong
and guard Atiga were charged, together with Edelmer Bantigue Y Eulalio, Reynaldo Aquino
and John Doe, with violation of P.D. 532 (Anti-Highway Robbery Law) before the Fiscal of
Pasay City. A copy of the complaint is hereto attached as Exhibit "D";

6. The Fiscal of Pasay City then filed an information charging the aforesaid persons with
the said crime before Branch 112 of the Regional Trial Court of Pasay City. A copy of the
said information is hereto attached as Exhibit "E." The case is still being tried as of this
date;

7. Demands were made by the plaintiff upon the defendant to pay the amount of the loss
of P725,000.00, but the latter refused to pay as the loss is excluded from the coverage of
the insurance policy, attached hereto as Exhibit "A," specifically under page 1 thereof,
"General Exceptions" Section (b), which is marked as Exhibit "A-1," and which reads as
follows:

GENERAL EXCEPTIONS

The company shall not be liable under this policy in report of


xxx xxx xxx

(b) any loss caused by any dishonest, fraudulent or criminal act of the
insured or any officer, employee, partner, director, trustee or authorized
representative of the Insured whether acting alone or in conjunction with
others. . . .

8. The plaintiff opposes the contention of the defendant and contends that Atiga and
Magalong are not its "officer, employee, . . . trustee or authorized representative . . . at the
time of the robbery. 1

On 26 April 1990, the trial court rendered its decision in favor of Producers. The dispositive portion thereof reads as follows:

WHEREFORE, premises considered, the Court finds for plaintiff and against defendant, and

(a) orders defendant to pay plaintiff the net amount of P540,000.00 as


liability under Policy No. 0207 (as mitigated by the P40,000.00 special
clause deduction and by the recovered sum of P145,000.00), with
interest thereon at the legal rate, until fully paid;

(b) orders defendant to pay plaintiff the sum of P30,000.00 as and for
attorney's fees; and

(c) orders defendant to pay costs of suit.

All other claims and counterclaims are accordingly dismissed forthwith.

SO ORDERED. 2

The trial court ruled that Magalong and Atiga were not employees or representatives of Producers. It Said:

The Court is satisfied that plaintiff may not be said to have selected and engaged Magalong and Atiga, their
services as armored car driver and as security guard having been merely offered by PRC Management and
by Unicorn Security and which latter firms assigned them to plaintiff. The wages and salaries of both
Magalong and Atiga are presumably paid by their respective firms, which alone wields the power to dismiss
them. Magalong and Atiga are assigned to plaintiff in fulfillment of agreements to provide driving services
and property protection as such in a context which does not impress the Court as translating into
plaintiff's power to control the conduct of any assigned driver or security guard, beyond perhaps entitling
plaintiff to request are replacement for such driver guard. The finding is accordingly compelled that neither
Magalong nor Atiga were plaintiff's "employees" in avoidance of defendant's liability under the policy,
particularly the general exceptions therein embodied.

Neither is the Court prepared to accept the proposition that driver Magalong and guard Atiga were the
"authorized representatives" of plaintiff. They were merely an assigned armored car driver and security
guard, respectively, for the June 29, 1987 money transfer from plaintiff's Pasay Branch to its Makati Head
Office. Quite plainly it was teller Maribeth Alampay who had "custody" of the P725,000.00 cash being
transferred along a specified money route, and hence plaintiff's then designated "messenger" adverted to in
the policy. 3

Fortune appealed this decision to the Court of Appeals which docketed the case as CA-G.R. CV No. 32946. In its
decision 4 promulgated on 3 May 1994, it affirmed in toto the appealed decision.

The Court of Appeals agreed with the conclusion of the trial court that Magalong and Atiga were neither employees nor
authorized representatives of Producers and ratiocinated as follows:

A policy or contract of insurance is to be construed liberally in favor of the insured and strictly against the
insurance company (New Life Enterprises vs. Court of Appeals, 207 SCRA 669; Sun Insurance Office, Ltd.
vs. Court of Appeals, 211 SCRA 554). Contracts of insurance, like other contracts, are to be construed
according to the sense and meaning of the terms which the parties themselves have used. If such terms are
clear and unambiguous, they must be taken and understood in their plain, ordinary and popular sense (New
Life Enterprises Case, supra, p. 676; Sun Insurance Office, Ltd. vs. Court of Appeals, 195 SCRA 193).

The language used by defendant-appellant in the above quoted stipulation is plain, ordinary and simple. No
other interpretation is necessary. The word "employee" must be taken to mean in the ordinary sense.

The Labor Code is a special law specifically dealing with/and specifically designed to protect labor and
therefore its definition as to employer-employee relationships insofar as the application/enforcement of said
Code is concerned must necessarily be inapplicable to an insurance contract which defendant-appellant
itself had formulated. Had it intended to apply the Labor Code in defining what the word "employee" refers
to, it must/should have so stated expressly in the insurance policy.

Said driver and security guard cannot be considered as employees of plaintiff-appellee bank because it has
no power to hire or to dismiss said driver and security guard under the contracts (Exhs. 8 and C) except
only to ask for their replacements from the contractors. 5

On 20 June 1994, Fortune filed this petition for review on certiorari. It alleges that the trial court and the Court of Appeals
erred in holding it liable under the insurance policy because the loss falls within the general exceptions clause considering
that driver Magalong and security guard Atiga were Producers' authorized representatives or employees in the transfer of the
money and payroll from its branch office in Pasay City to its head office in Makati.

According to Fortune, when Producers commissioned a guard and a driver to transfer its funds from one branch to another,
they effectively and necessarily became its authorized representatives in the care and custody of the money. Assuming that
they could not be considered authorized representatives, they were, nevertheless, employees of Producers. It asserts that
the existence of an employer-employee relationship "is determined by law and being such, it cannot be the subject of
agreement." Thus, if there was in reality an employer-employee relationship between Producers, on the one hand, and
Magalong and Atiga, on the other, the provisions in the contracts of Producers with PRC Management System for Magalong
and with Unicorn Security Services for Atiga which state that Producers is not their employer and that it is absolved from any
liability as an employer, would not obliterate the relationship.

Fortune points out that an employer-employee relationship depends upon four standards: (1) the manner of selection and
engagement of the putative employee; (2) the mode of payment of wages; (3) the presence or absence of a power to
dismiss; and (4) the presence and absence of a power to control the putative employee's conduct. Of the four, the right-of-
control test has been held to be the decisive factor. 6 It asserts that the power of control over Magalong and Atiga was
vested in and exercised by Producers. Fortune further insists that PRC Management System and Unicorn Security
Services are but "labor-only" contractors under Article 106 of the Labor Code which provides:

Art. 106. Contractor or subcontractor. There is "labor-only" contracting where the person supplying
workers to an employer does not have substantial capital or investment in the form of tools, equipment,
machineries, work premises, among others, and the workers recruited and placed by such persons are
performing activities which are directly related to the principal business of such employer. In such cases, the
person or intermediary shall be considered merely as an agent of the employer who shall be responsible to
the workers in the same manner and extent as if the latter were directly employed by him.

Fortune thus contends that Magalong and Atiga were employees of Producers, following the ruling in International Timber
Corp. vs. NLRC 7 that a finding that a contractor is a "labor-only" contractor is equivalent to a finding that there is an
employer-employee relationship between the owner of the project and the employees of the "labor-only" contractor.

On the other hand, Producers contends that Magalong and Atiga were not its employees since it had nothing to do with their
selection and engagement, the payment of their wages, their dismissal, and the control of their conduct. Producers argued
that the rule in International Timber Corp. is not applicable to all cases but only when it becomes necessary to prevent any
violation or circumvention of the Labor Code, a social legislation whose provisions may set aside contracts entered into by
parties in order to give protection to the working man.

Producers further asseverates that what should be applied is the rule in American President Lines vs. Clave, 8 to wit:

In determining the existence of employer-employee relationship, the following elements are generally
considered, namely: (1) the selection and engagement of the employee; (2) the payment of wages; (3) the
power of dismissal; and (4) the power to control the employee's conduct.
Since under Producers' contract with PRC Management Systems it is the latter which assigned Magalong as the driver of
Producers' armored car and was responsible for his faithful discharge of his duties and responsibilities, and since Producers
paid the monthly compensation of P1,400.00 per driver to PRC Management Systems and not to Magalong, it is clear that
Magalong was not Producers' employee. As to Atiga, Producers relies on the provision of its contract with Unicorn Security
Services which provides that the guards of the latter "are in no sense employees of the CLIENT."

There is merit in this petition.

It should be noted that the insurance policy entered into by the parties is a theft or robbery insurance policy which is a form of
casualty insurance. Section 174 of the Insurance Code provides:

Sec. 174. Casualty insurance is insurance covering loss or liability arising from accident or mishap,
excluding certain types of loss which by law or custom are considered as falling exclusively within the scope
of insurance such as fire or marine. It includes, but is not limited to, employer's liability insurance, public
liability insurance, motor vehicle liability insurance, plate glass insurance, burglary and theft insurance,
personal accident and health insurance as written by non-life insurance companies, and other substantially
similar kinds of insurance. (emphases supplied)

Except with respect to compulsory motor vehicle liability insurance, the Insurance Code contains no other provisions
applicable to casualty insurance or to robbery insurance in particular. These contracts are, therefore, governed by the
general provisions applicable to all types of insurance. Outside of these, the rights and obligations of the parties must be
determined by the terms of their contract, taking into consideration its purpose and always in accordance with the general
principles of insurance law. 9

It has been aptly observed that in burglary, robbery, and theft insurance, "the opportunity to defraud the insurer the moral
hazard is so great that insurers have found it necessary to fill up their policies with countless restrictions, many designed
to reduce this hazard. Seldom does the insurer assume the risk of all losses due to the hazards insured against." 10 Persons
frequently excluded under such provisions are those in the insured's service and employment. 11 The purpose of the
exception is to guard against liability should the theft be committed by one having unrestricted access to the
property. 12 In such cases, the terms specifying the excluded classes are to be given their meaning as understood in
common speech. 13 The terms "service" and "employment" are generally associated with the idea of selection, control,
and compensation. 14

A contract of insurance is a contract of adhesion, thus any ambiguity therein should be resolved against the insurer, or it
15

should be construed liberally in favor of the insured and strictly against the insurer. Limitations of liability should be
16

regarded with extreme jealousy and must be construed


in such a way, as to preclude the insurer from non-compliance with its obligation. 17 It goes without saying then that if
the terms of the contract are clear and unambiguous, there is no room for construction and such terms cannot be
enlarged or diminished by judicial construction. 18

An insurance contract is a contract of indemnity upon the terms and conditions specified therein. It is settled that the
19

terms of the policy constitute the measure of the insurer's liability. In the absence of statutory prohibition to the
20

contrary, insurance companies have the same rights as individuals to limit their liability and to impose whatever
conditions they deem best upon their obligations not inconsistent with public policy.

With the foregoing principles in mind, it may now be asked whether Magalong and Atiga qualify as employees or authorized
representatives of Producers under paragraph (b) of the general exceptions clause of the policy which, for easy reference, is
again quoted:

GENERAL EXCEPTIONS

The company shall not be liable under this policy in respect of

xxx xxx xxx

(b) any loss caused by any dishonest, fraudulent or criminal act of the insured or any
officer, employee, partner, director, trustee or authorized representative of the Insured
whether acting alone or in conjunction with others. . . . (emphases supplied)
There is marked disagreement between the parties on the correct meaning of the terms "employee" and "authorized
representatives."

It is clear to us that insofar as Fortune is concerned, it was its intention to exclude and exempt from protection and coverage
losses arising from dishonest, fraudulent, or criminal acts of persons granted or having unrestricted access to Producers'
money or payroll. When it used then the term "employee," it must have had in mind any person who qualifies as such as
generally and universally understood, or jurisprudentially established in the light of the four standards in the determination of
the employer-employee relationship, 21 or as statutorily declared even in a limited sense as in the case of Article 106 of
the Labor Code which considers the employees under a "labor-only" contract as employees of the party employing
them and not of the party who supplied them to the employer. 22

Fortune claims that Producers' contracts with PRC Management Systems and Unicorn Security Services are "labor-only"
contracts.

Producers, however, insists that by the express terms thereof, it is not the employer of Magalong. Notwithstanding
such express assumption of PRC Management Systems and Unicorn Security Services that the drivers and the
security guards each shall supply to Producers are not the latter's employees, it may, in fact, be that it is because
the contracts are, indeed, "labor-only" contracts. Whether they are is, in the light of the criteria provided for in Article
106 of the Labor Code, a question of fact. Since the parties opted to submit the case for judgment on the basis of
their stipulation of facts which are strictly limited to the insurance policy, the contracts with PRC Management
Systems and Unicorn Security Services, the complaint for violation of P.D. No. 532, and the information therefor filed
by the City Fiscal of Pasay City, there is a paucity of evidence as to whether the contracts between Producers and
PRC Management Systems and Unicorn Security Services are "labor-only" contracts.

But even granting for the sake of argument that these contracts were not "labor-only" contracts, and PRC Management
Systems and Unicorn Security Services were truly independent contractors, we are satisfied that Magalong and Atiga were,
in respect of the transfer of Producer's money from its Pasay City branch to its head office in Makati, its "authorized
representatives" who served as such with its teller Maribeth Alampay. Howsoever viewed, Producers entrusted the three with
the specific duty to safely transfer the money to its head office, with Alampay to be responsible for its custody in transit;
Magalong to drive the armored vehicle which would carry the money; and Atiga to provide the needed security for the money,
the vehicle, and his two other companions. In short, for these particular tasks, the three acted as agents of Producers. A
"representative" is defined as one who represents or stands in the place of another; one who represents others or another in
a special capacity, as an agent, and is interchangeable with "agent." 23

In view of the foregoing, Fortune is exempt from liability under the general exceptions clause of the insurance policy.

WHEREFORE , the instant petition is hereby GRANTED. The decision of the Court of Appeals in CA-G.R. CV No. 32946
dated 3 May 1994 as well as that of Branch 146 of the Regional Trial Court of Makati in Civil Case No. 1817 are REVERSED
and SET ASIDE. The complaint in Civil Case No. 1817 is DISMISSED.

No pronouncement as to costs.

SO ORDERED.

Bellosillo and Kapunan, JJ., concur.

Padilla, J., took no part.

Quiason, J., is on leave.


G.R. No. 156167 May 16, 2005

GULF RESORTS, INC., petitioner,


vs.
PHILIPPINE CHARTER INSURANCE CORPORATION, respondent.

DECISION

PUNO, J.:

Before the Court is the petition for certiorari under Rule 45 of the Revised Rules of Court by petitioner GULF RESORTS,
INC., against respondent PHILIPPINE CHARTER INSURANCE CORPORATION. Petitioner assails the appellate court
decision1 which dismissed its two appeals and affirmed the judgment of the trial court.

For review are the warring interpretations of petitioner and respondent on the scope of the insurance companys liability for
earthquake damage to petitioners properties. Petitioner avers that, pursuant to its earthquake shock endorsement rider,
Insurance Policy No. 31944 covers all damages to the properties within its resort caused by earthquake. Respondent
contends that the rider limits its liability for loss to the two swimming pools of petitioner.

The facts as established by the court a quo, and affirmed by the appellate court are as follows:

[P]laintiff is the owner of the Plaza Resort situated at Agoo, La Union and had its properties in said resort insured
originally with the American Home Assurance Company (AHAC-AIU). In the first four insurance policies issued by
AHAC-AIU from 1984-85; 1985-86; 1986-1987; and 1987-88 (Exhs. "C", "D", "E" and "F"; also Exhs. "1", "2", "3" and
"4" respectively), the risk of loss from earthquake shock was extended only to plaintiffs two swimming pools, thus,
"earthquake shock endt." (Item 5 only) (Exhs. "C-1"; "D-1," and "E" and two (2) swimming pools only (Exhs. "C-1";
D-1", "E" and "F-1"). "Item 5" in those policies referred to the two (2) swimming pools only (Exhs. "1-B", "2-B", "3-B"
and "F-2"); that subsequently AHAC(AIU) issued in plaintiffs favor Policy No. 206-4182383-0 covering the period
March 14, 1988 to March 14, 1989 (Exhs. "G" also "G-1") and in said policy the earthquake endorsement clause as
indicated in Exhibits "C-1", "D-1", Exhibits "E" and "F-1" was deleted and the entry under Endorsements/Warranties
at the time of issue read that plaintiff renewed its policy with AHAC (AIU) for the period of March 14, 1989 to March
14, 1990 under Policy No. 206-4568061-9 (Exh. "H") which carried the entry under "Endorsement/Warranties at
Time of Issue", which read "Endorsement to Include Earthquake Shock (Exh. "6-B-1") in the amount of P10,700.00
and paid P42,658.14 (Exhs. "6-A" and "6-B") as premium thereof, computed as follows:

Item - P7,691,000.00 - on the Clubhouse only

@ .392%;

- 1,500,000.00 - on the furniture, etc. contained in the building above-


mentioned@ .490%;

- 393,000.00 - on the two swimming pools, only (against the peril of


earthquake shock only) @ 0.100%

- 116,600.00 other buildings include as follows:

a) Tilter House - P19,800.00 - 0.551%

b) Power House - P41,000.00 - 0.551%


c) House Shed - P55,000.00 - 0.540%

P100,000.00 - for furniture, fixtures, lines air-con and operating


equipment

that plaintiff agreed to insure with defendant the properties covered by AHAC (AIU) Policy No. 206-4568061-9 (Exh.
"H") provided that the policy wording and rates in said policy be copied in the policy to be issued by defendant; that
defendant issued Policy No. 31944 to plaintiff covering the period of March 14, 1990 to March 14, 1991
for P10,700,600.00 for a total premium of P45,159.92 (Exh. "I"); that in the computation of the premium, defendants
Policy No. 31944 (Exh. "I"), which is the policy in question, contained on the right-hand upper portion of page 7
thereof, the following:

Rate-Various

Premium P37,420.60 F/L

2,061.52 Typhoon

1,030.76 EC

393.00 ES

Doc. Stamps 3,068.10

F.S.T. 776.89

Prem. Tax 409.05

TOTAL 45,159.92;

that the above break-down of premiums shows that plaintiff paid only P393.00 as premium against earthquake
shock (ES); that in all the six insurance policies (Exhs. "C", "D", "E", "F", "G" and "H"), the premium against the peril
of earthquake shock is the same, that is P393.00 (Exhs. "C" and "1-B"; "2-B" and "3-B-1" and "3-B-2"; "F-02" and "4-
A-1"; "G-2" and "5-C-1"; "6-C-1"; issued by AHAC (Exhs. "C", "D", "E", "F", "G" and "H") and in Policy No. 31944
issued by defendant, the shock endorsement provide(sic):
In consideration of the payment by the insured to the company of the sum included additional premium the
Company agrees, notwithstanding what is stated in the printed conditions of this policy due to the contrary,
that this insurance covers loss or damage to shock to any of the property insured by this Policy occasioned
by or through or in consequence of earthquake (Exhs. "1-D", "2-D", "3-A", "4-B", "5-A", "6-D" and "7-C");

that in Exhibit "7-C" the word "included" above the underlined portion was deleted; that on July 16, 1990 an
earthquake struck Central Luzon and Northern Luzon and plaintiffs properties covered by Policy No. 31944 issued
by defendant, including the two swimming pools in its Agoo Playa Resort were damaged. 2

After the earthquake, petitioner advised respondent that it would be making a claim under its Insurance Policy No. 31944 for
damages on its properties. Respondent instructed petitioner to file a formal claim, then assigned the investigation of the claim
to an independent claims adjuster, Bayne Adjusters and Surveyors, Inc. 3 On July 30, 1990, respondent, through its adjuster,
requested petitioner to submit various documents in support of its claim. On August 7, 1990, Bayne Adjusters and Surveyors,
Inc., through its Vice-President A.R. de Leon,4 rendered a preliminary report5 finding extensive damage caused by the
earthquake to the clubhouse and to the two swimming pools. Mr. de Leon stated that "except for the swimming pools, all
affected items have no coverage for earthquake shocks." 6 On August 11, 1990, petitioner filed its formal demand7 for
settlement of the damage to all its properties in the Agoo Playa Resort. On August 23, 1990, respondent denied petitioners
claim on the ground that its insurance policy only afforded earthquake shock coverage to the two swimming pools of the
resort.8Petitioner and respondent failed to arrive at a settlement. 9 Thus, on January 24, 1991, petitioner filed a
complaint10 with the regional trial court of Pasig praying for the payment of the following:

1.) The sum of P5,427,779.00, representing losses sustained by the insured properties, with interest thereon, as
computed under par. 29 of the policy (Annex "B") until fully paid;

2.) The sum of P428,842.00 per month, representing continuing losses sustained by plaintiff on account of
defendants refusal to pay the claims;

3.) The sum of P500,000.00, by way of exemplary damages;

4.) The sum of P500,000.00 by way of attorneys fees and expenses of litigation;

5.) Costs.11

Respondent filed its Answer with Special and Affirmative Defenses with Compulsory Counterclaims. 12

On February 21, 1994, the lower court after trial ruled in favor of the respondent, viz:

The above schedule clearly shows that plaintiff paid only a premium of P393.00 against the peril of earthquake
shock, the same premium it paid against earthquake shock only on the two swimming pools in all the policies issued
by AHAC(AIU) (Exhibits "C", "D", "E", "F" and "G"). From this fact the Court must consequently agree with the
position of defendant that the endorsement rider (Exhibit "7-C") means that only the two swimming pools were
insured against earthquake shock.

Plaintiff correctly points out that a policy of insurance is a contract of adhesion hence, where the language used in
an insurance contract or application is such as to create ambiguity the same should be resolved against the party
responsible therefor, i.e., the insurance company which prepared the contract. To the mind of [the] Court, the
language used in the policy in litigation is clear and unambiguous hence there is no need for interpretation or
construction but only application of the provisions therein.

From the above observations the Court finds that only the two (2) swimming pools had earthquake shock coverage
and were heavily damaged by the earthquake which struck on July 16, 1990. Defendant having admitted that the
damage to the swimming pools was appraised by defendants adjuster at P386,000.00, defendant must, by virtue of
the contract of insurance, pay plaintiff said amount.

Because it is the finding of the Court as stated in the immediately preceding paragraph that defendant is liable only
for the damage caused to the two (2) swimming pools and that defendant has made known to plaintiff its willingness
and readiness to settle said liability, there is no basis for the grant of the other damages prayed for by plaintiff. As to
the counterclaims of defendant, the Court does not agree that the action filed by plaintiff is baseless and highly
speculative since such action is a lawful exercise of the plaintiffs right to come to Court in the honest belief that their
Complaint is meritorious. The prayer, therefore, of defendant for damages is likewise denied.
WHEREFORE, premises considered, defendant is ordered to pay plaintiffs the sum of THREE HUNDRED EIGHTY
SIX THOUSAND PESOS (P386,000.00) representing damage to the two (2) swimming pools, with interest at 6% per
annum from the date of the filing of the Complaint until defendants obligation to plaintiff is fully paid.

No pronouncement as to costs.13

Petitioners Motion for Reconsideration was denied. Thus, petitioner filed an appeal with the Court of Appeals based on the
following assigned errors:14

A. THE TRIAL COURT ERRED IN FINDING THAT PLAINTIFF-APPELLANT CAN ONLY RECOVER FOR THE
DAMAGE TO ITS TWO SWIMMING POOLS UNDER ITS FIRE POLICY NO. 31944, CONSIDERING ITS
PROVISIONS, THE CIRCUMSTANCES SURROUNDING THE ISSUANCE OF SAID POLICY AND THE
ACTUATIONS OF THE PARTIES SUBSEQUENT TO THE EARTHQUAKE OF JULY 16, 1990.

B. THE TRIAL COURT ERRED IN DETERMINING PLAINTIFF-APPELLANTS RIGHT TO RECOVER UNDER


DEFENDANT-APPELLEES POLICY (NO. 31944; EXH "I") BY LIMITING ITSELF TO A CONSIDERATION OF THE
SAID POLICY ISOLATED FROM THE CIRCUMSTANCES SURROUNDING ITS ISSUANCE AND THE
ACTUATIONS OF THE PARTIES AFTER THE EARTHQUAKE OF JULY 16, 1990.

C. THE TRIAL COURT ERRED IN NOT HOLDING THAT PLAINTIFF-APPELLANT IS ENTITLED TO THE
DAMAGES CLAIMED, WITH INTEREST COMPUTED AT 24% PER ANNUM ON CLAIMS ON PROCEEDS OF
POLICY.

On the other hand, respondent filed a partial appeal, assailing the lower courts failure to award it attorneys fees and
damages on its compulsory counterclaim.

After review, the appellate court affirmed the decision of the trial court and ruled, thus:

However, after carefully perusing the documentary evidence of both parties, We are not convinced that the last two
(2) insurance contracts (Exhs. "G" and "H"), which the plaintiff-appellant had with AHAC (AIU) and upon which the
subject insurance contract with Philippine Charter Insurance Corporation is said to have been based and copied
(Exh. "I"), covered an extended earthquake shock insurance on all the insured properties.

xxx

We also find that the Court a quo was correct in not granting the plaintiff-appellants prayer for the imposition of
interest 24% on the insurance claim and 6% on loss of income allegedly amounting toP4,280,000.00. Since the
defendant-appellant has expressed its willingness to pay the damage caused on the two (2) swimming pools, as the
Court a quo and this Court correctly found it to be liable only, it then cannot be said that it was in default and
therefore liable for interest.

Coming to the defendant-appellants prayer for an attorneys fees, long-standing is the rule that the award thereof is
subject to the sound discretion of the court. Thus, if such discretion is well-exercised, it will not be disturbed on
appeal (Castro et al. v. CA, et al., G.R. No. 115838, July 18, 2002). Moreover, being the award thereof an exception
rather than a rule, it is necessary for the court to make findings of facts and law that would bring the case within the
exception and justify the grant of such award (Country Bankers Insurance Corp. v. Lianga Bay and Community Multi-
Purpose Coop., Inc., G.R. No. 136914, January 25, 2002). Therefore, holding that the plaintiff-appellants action is
not baseless and highly speculative, We find that the Court a quo did not err in granting the same.

WHEREFORE, in view of all the foregoing, both appeals are hereby DISMISSED and judgment of the Trial Court
hereby AFFIRMED in toto. No costs.15

Petitioner filed the present petition raising the following issues: 16

A. WHETHER THE COURT OF APPEALS CORRECTLY HELD THAT UNDER RESPONDENTS INSURANCE
POLICY NO. 31944, ONLY THE TWO (2) SWIMMING POOLS, RATHER THAN ALL THE PROPERTIES COVERED
THEREUNDER, ARE INSURED AGAINST THE RISK OF EARTHQUAKE SHOCK.

B. WHETHER THE COURT OF APPEALS CORRECTLY DENIED PETITIONERS PRAYER FOR DAMAGES WITH
INTEREST THEREON AT THE RATE CLAIMED, ATTORNEYS FEES AND EXPENSES OF LITIGATION.
Petitioner contends:

First, that the policys earthquake shock endorsement clearly covers all of the properties insured and not only the swimming
pools. It used the words "any property insured by this policy," and it should be interpreted as all inclusive.

Second, the unqualified and unrestricted nature of the earthquake shock endorsement is confirmed in the body of the
insurance policy itself, which states that it is "[s]ubject to: Other Insurance Clause, Typhoon Endorsement,Earthquake
Shock Endt., Extended Coverage Endt., FEA Warranty & Annual Payment Agreement On Long Term Policies." 17

Third, that the qualification referring to the two swimming pools had already been deleted in the earthquake shock
endorsement.

Fourth, it is unbelievable for respondent to claim that it only made an inadvertent omission when it deleted the said
qualification.

Fifth, that the earthquake shock endorsement rider should be given precedence over the wording of the insurance policy,
because the rider is the more deliberate expression of the agreement of the contracting parties.

Sixth, that in their previous insurance policies, limits were placed on the endorsements/warranties enumerated at the time of
issue.

Seventh, any ambiguity in the earthquake shock endorsement should be resolved in favor of petitioner and against
respondent. It was respondent which caused the ambiguity when it made the policy in issue.

Eighth, the qualification of the endorsement limiting the earthquake shock endorsement should be interpreted as a caveat on
the standard fire insurance policy, such as to remove the two swimming pools from the coverage for the risk of fire. It should
not be used to limit the respondents liability for earthquake shock to the two swimming pools only.

Ninth, there is no basis for the appellate court to hold that the additional premium was not paid under the extended
coverage. The premium for the earthquake shock coverage was already included in the premium paid for the policy.

Tenth, the parties contemporaneous and subsequent acts show that they intended to extend earthquake shock coverage to
all insured properties. When it secured an insurance policy from respondent, petitioner told respondent that it wanted an
exact replica of its latest insurance policy from American Home Assurance Company (AHAC-AIU), which covered all the
resorts properties for earthquake shock damage and respondent agreed. After the July 16, 1990 earthquake, respondent
assured petitioner that it was covered for earthquake shock. Respondents insurance adjuster, Bayne Adjusters and
Surveyors, Inc., likewise requested petitioner to submit the necessary documents for its building claims and other repair
costs. Thus, under the doctrine of equitable estoppel, it cannot deny that the insurance policy it issued to petitioner covered
all of the properties within the resort.

Eleventh, that it is proper for it to avail of a petition for review by certiorari under Rule 45 of the Revised Rules of Court as its
remedy, and there is no need for calibration of the evidence in order to establish the facts upon which this petition is based.

On the other hand, respondent made the following counter arguments: 18

First, none of the previous policies issued by AHAC-AIU from 1983 to 1990 explicitly extended coverage against earthquake
shock to petitioners insured properties other than on the two swimming pools. Petitioner admitted that from 1984 to 1988,
only the two swimming pools were insured against earthquake shock. From 1988 until 1990, the provisions in its policy were
practically identical to its earlier policies, and there was no increase in the premium paid. AHAC-AIU, in a letter 19 by its
representative Manuel C. Quijano, categorically stated that its previous policy, from which respondents policy was copied,
covered only earthquake shock for the two swimming pools.

Second, petitioners payment of additional premium in the amount of P393.00 shows that the policy only covered earthquake
shock damage on the two swimming pools. The amount was the same amount paid by petitioner for earthquake shock
coverage on the two swimming pools from 1990-1991. No additional premium was paid to warrant coverage of the other
properties in the resort.

Third, the deletion of the phrase pertaining to the limitation of the earthquake shock endorsement to the two swimming pools
in the policy schedule did not expand the earthquake shock coverage to all of petitioners properties. As per its agreement
with petitioner, respondent copied its policy from the AHAC-AIU policy provided by petitioner. Although the first five policies
contained the said qualification in their riders title, in the last two policies, this qualification in the title was deleted. AHAC-
AIU, through Mr. J. Baranda III, stated that such deletion was a mere inadvertence. This inadvertence did not make the policy
incomplete, nor did it broaden the scope of the endorsement whose descriptive title was merely enumerated. Any ambiguity
in the policy can be easily resolved by looking at the other provisions, specially the enumeration of the items insured, where
only the two swimming pools were noted as covered for earthquake shock damage.

Fourth, in its Complaint, petitioner alleged that in its policies from 1984 through 1988, the phrase "Item 5 P393,000.00 on
the two swimming pools only (against the peril of earthquake shock only)" meant that only the swimming pools were insured
for earthquake damage. The same phrase is used in toto in the policies from 1989 to 1990, the only difference being the
designation of the two swimming pools as "Item 3."

Fifth, in order for the earthquake shock endorsement to be effective, premiums must be paid for all the properties covered. In
all of its seven insurance policies, petitioner only paid P393.00 as premium for coverage of the swimming pools against
earthquake shock. No other premium was paid for earthquake shock coverage on the other properties. In addition, the use of
the qualifier "ANY" instead of "ALL" to describe the property covered was done deliberately to enable the parties to specify
the properties included for earthquake coverage.

Sixth, petitioner did not inform respondent of its requirement that all of its properties must be included in the earthquake
shock coverage. Petitioners own evidence shows that it only required respondent to follow the exact provisions of its
previous policy from AHAC-AIU. Respondent complied with this requirement. Respondents only deviation from the
agreement was when it modified the provisions regarding the replacement cost endorsement. With regard to the issue under
litigation, the riders of the old policy and the policy in issue are identical.

Seventh, respondent did not do any act or give any assurance to petitioner as would estop it from maintaining that only the
two swimming pools were covered for earthquake shock. The adjusters letter notifying petitioner to present certain
documents for its building claims and repair costs was given to petitioner before the adjuster knew the full coverage of its
policy.

Petitioner anchors its claims on AHAC-AIUs inadvertent deletion of the phrase "Item 5 Only" after the descriptive name or
title of the Earthquake Shock Endorsement. However, the words of the policy reflect the parties clear intention to limit
earthquake shock coverage to the two swimming pools.

Before petitioner accepted the policy, it had the opportunity to read its conditions. It did not object to any deficiency nor did it
institute any action to reform the policy. The policy binds the petitioner.

Eighth, there is no basis for petitioner to claim damages, attorneys fees and litigation expenses. Since respondent was
willing and able to pay for the damage caused on the two swimming pools, it cannot be considered to be in default, and
therefore, it is not liable for interest.

We hold that the petition is devoid of merit.

In Insurance Policy No. 31944, four key items are important in the resolution of the case at bar.

First, in the designation of location of risk, only the two swimming pools were specified as included, viz:

ITEM 3 393,000.00 On the two (2) swimming pools only (against the peril of earthquake shock only) 20

Second, under the breakdown for premium payments, 21 it was stated that:

PREMIUM RECAPITULATION

ITEM NOS. AMOUNT RATES PREMIUM

xxx
3 393,000.00 0.100%-E/S 393.0022]

Third, Policy Condition No. 6 stated:

6. This insurance does not cover any loss or damage occasioned by or through or in consequence, directly or
indirectly of any of the following occurrences, namely:--

(a) Earthquake, volcanic eruption or other convulsion of nature. 23

Fourth, the rider attached to the policy, titled "Extended Coverage Endorsement (To Include the Perils of Explosion, Aircraft,
Vehicle and Smoke)," stated, viz:

ANNUAL PAYMENT AGREEMENT ON


LONG TERM POLICIES

THE INSURED UNDER THIS POLICY HAVING ESTABLISHED AGGREGATE SUMS INSURED IN EXCESS OF
FIVE MILLION PESOS, IN CONSIDERATION OF A DISCOUNT OF 5% OR 7 % OF THE NET PREMIUM x x x
POLICY HEREBY UNDERTAKES TO CONTINUE THE INSURANCE UNDER THE ABOVE NAMED x x x AND TO
PAY THE PREMIUM.

Earthquake Endorsement

In consideration of the payment by the Insured to the Company of the sum of P. . . . . . . . . . . . . . . . . additional
premium the Company agrees, notwithstanding what is stated in the printed conditions of this Policy to the contrary,
that this insurance covers loss or damage (including loss or damage by fire) to any of the property insured by this
Policy occasioned by or through or in consequence of Earthquake.

Provided always that all the conditions of this Policy shall apply (except in so far as they may be hereby expressly
varied) and that any reference therein to loss or damage by fire should be deemed to apply also to loss or damage
occasioned by or through or in consequence of Earthquake. 24

Petitioner contends that pursuant to this rider, no qualifications were placed on the scope of the earthquake shock coverage.
Thus, the policy extended earthquake shock coverage to all of the insured properties.

It is basic that all the provisions of the insurance policy should be examined and interpreted in consonance with each
other.25 All its parts are reflective of the true intent of the parties. The policy cannot be construed piecemeal. Certain
stipulations cannot be segregated and then made to control; neither do particular words or phrases necessarily determine its
character. Petitioner cannot focus on the earthquake shock endorsement to the exclusion of the other provisions. All the
provisions and riders, taken and interpreted together, indubitably show the intention of the parties to extend earthquake
shock coverage to the two swimming pools only.

A careful examination of the premium recapitulation will show that it is the clear intent of the parties to extend earthquake
shock coverage only to the two swimming pools. Section 2(1) of the Insurance Code defines a contract of insurance as an
agreement whereby one undertakes for a consideration to indemnify another against loss, damage or liability arising from an
unknown or contingent event. Thus, an insurance contract exists where the following elements concur:

1. The insured has an insurable interest;

2. The insured is subject to a risk of loss by the happening of the designated peril;

3. The insurer assumes the risk;

4. Such assumption of risk is part of a general scheme to distribute actual losses among a large group of persons
bearing a similar risk; and

5. In consideration of the insurer's promise, the insured pays a premium.26 (Emphasis ours)
An insurance premium is the consideration paid an insurer for undertaking to indemnify the insured against a specified
peril.27 In fire, casualty, and marine insurance, the premium payable becomes a debt as soon as the risk attaches. 28 In the
subject policy, no premium payments were made with regard to earthquake shock coverage, except on the two swimming
pools. There is no mention of any premium payable for the other resort properties with regard to earthquake shock. This is
consistent with the history of petitioners previous insurance policies from AHAC-AIU. As borne out by petitioners witnesses:

CROSS EXAMINATION OF LEOPOLDO MANTOHAC TSN, November 25, 1991


pp. 12-13

Q. Now Mr. Mantohac, will it be correct to state also that insofar as your insurance policy during the period from
March 4, 1984 to March 4, 1985 the coverage on earthquake shock was limited to the two swimming pools only?

A. Yes, sir. It is limited to the two swimming pools, specifically shown in the warranty, there is a provision here that it
was only for item 5.

Q. More specifically Item 5 states the amount of P393,000.00 corresponding to the two swimming pools only?

A. Yes, sir.

CROSS EXAMINATION OF LEOPOLDO MANTOHAC TSN, November 25, 1991

pp. 23-26

Q. For the period from March 14, 1988 up to March 14, 1989, did you personally arrange for the procurement of this
policy?

A. Yes, sir.

Q. Did you also do this through your insurance agency?

A. If you are referring to Forte Insurance Agency, yes.

Q. Is Forte Insurance Agency a department or division of your company?

A. No, sir. They are our insurance agency.

Q. And they are independent of your company insofar as operations are concerned?

A. Yes, sir, they are separate entity.

Q. But insofar as the procurement of the insurance policy is concerned they are of course subject to your instruction,
is that not correct?

A. Yes, sir. The final action is still with us although they can recommend what insurance to take.

Q. In the procurement of the insurance police (sic) from March 14, 1988 to March 14, 1989, did you give written
instruction to Forte Insurance Agency advising it that the earthquake shock coverage must extend to all properties of
Agoo Playa Resort in La Union?

A. No, sir. We did not make any written instruction, although we made an oral instruction to that effect of extending
the coverage on (sic) the other properties of the company.

Q. And that instruction, according to you, was very important because in April 1987 there was an earthquake tremor
in La Union?

A. Yes, sir.

Q. And you wanted to protect all your properties against similar tremors in the [future], is that correct?
A. Yes, sir.

Q. Now, after this policy was delivered to you did you bother to check the provisions with respect to your instructions
that all properties must be covered again by earthquake shock endorsement?

A. Are you referring to the insurance policy issued by American Home Assurance Company marked Exhibit "G"?

Atty. Mejia: Yes.

Witness:

A. I examined the policy and seeing that the warranty on the earthquake shock endorsement has no more limitation
referring to the two swimming pools only, I was contented already that the previous limitation pertaining to the two
swimming pools was already removed.

Petitioner also cited and relies on the attachment of the phrase "Subject to: Other Insurance Clause, Typhoon
Endorsement, Earthquake Shock Endorsement, Extended Coverage Endorsement, FEA Warranty & Annual Payment
Agreement on Long Term Policies"29 to the insurance policy as proof of the intent of the parties to extend the coverage for
earthquake shock. However, this phrase is merely an enumeration of the descriptive titles of the riders, clauses, warranties or
endorsements to which the policy is subject, as required under Section 50, paragraph 2 of the Insurance Code.

We also hold that no significance can be placed on the deletion of the qualification limiting the coverage to the two swimming
pools. The earthquake shock endorsement cannot stand alone. As explained by the testimony of Juan Baranda III,
underwriter for AHAC-AIU:

DIRECT EXAMINATION OF JUAN BARANDA III30


TSN, August 11, 1992
pp. 9-12

Atty. Mejia:

We respectfully manifest that the same exhibits C to H inclusive have been previously marked by counsel
for defendant as Exhibit[s] 1-6 inclusive. Did you have occasion to review of (sic) these six (6) policies
issued by your company [in favor] of Agoo Playa Resort?

WITNESS:

Yes[,] I remember having gone over these policies at one point of time, sir.

Q. Now, wach (sic) of these six (6) policies marked in evidence as Exhibits C to H respectively carries an earthquake
shock endorsement[?] My question to you is, on the basis on (sic) the wordings indicated in Exhibits C to H
respectively what was the extent of the coverage [against] the peril of earthquake shock as provided for in each of
the six (6) policies?

xxx

WITNESS:

The extent of the coverage is only up to the two (2) swimming pools, sir.

Q. Is that for each of the six (6) policies namely: Exhibits C, D, E, F, G and H?

A. Yes, sir.

ATTY. MEJIA:

What is your basis for stating that the coverage against earthquake shock as provided for in each of the six
(6) policies extend to the two (2) swimming pools only?
WITNESS:

Because it says here in the policies, in the enumeration "Earthquake Shock Endorsement, in the Clauses
and Warranties: Item 5 only (Earthquake Shock Endorsement)," sir.

ATTY. MEJIA:

Witness referring to Exhibit C-1, your Honor.

WITNESS:

We do not normally cover earthquake shock endorsement on stand alone basis. For swimming pools we do
cover earthquake shock. For building we covered it for full earthquake coverage which includes earthquake
shock

COURT:

As far as earthquake shock endorsement you do not have a specific coverage for other things other than
swimming pool? You are covering building? They are covered by a general insurance?

WITNESS:

Earthquake shock coverage could not stand alone. If we are covering building or another we can issue
earthquake shock solely but that the moment I see this, the thing that comes to my mind is either insuring a
swimming pool, foundations, they are normally affected by earthquake but not by fire, sir.

DIRECT EXAMINATION OF JUAN BARANDA III


TSN, August 11, 1992
pp. 23-25

Q. Plaintiffs witness, Mr. Mantohac testified and he alleged that only Exhibits C, D, E and F inclusive [remained] its
coverage against earthquake shock to two (2) swimming pools only but that Exhibits G and H respectively entend
the coverage against earthquake shock to all the properties indicated in the respective schedules attached to said
policies, what can you say about that testimony of plaintiffs witness?

WITNESS:

As I have mentioned earlier, earthquake shock cannot stand alone without the other half of it. I assure you
that this one covers the two swimming pools with respect to earthquake shock endorsement. Based on it, if
we are going to look at the premium there has been no change with respect to the rates. Everytime (sic)
there is a renewal if the intention of the insurer was to include the earthquake shock, I think there is a
substantial increase in the premium. We are not only going to consider the two (2) swimming pools of the
other as stated in the policy. As I see, there is no increase in the amount of the premium. I must say that the
coverage was not broaden (sic) to include the other items.

COURT:

They are the same, the premium rates?

WITNESS:

They are the same in the sence (sic), in the amount of the coverage. If you are going to do some
computation based on the rates you will arrive at the same premiums, your Honor.

CROSS-EXAMINATION OF JUAN BARANDA III


TSN, September 7, 1992
pp. 4-6

ATTY. ANDRES:
Would you as a matter of practice [insure] swimming pools for fire insurance?

WITNESS:

No, we dont, sir.

Q. That is why the phrase "earthquake shock to the two (2) swimming pools only" was placed, is it not?

A. Yes, sir.

ATTY. ANDRES:

Will you not also agree with me that these exhibits, Exhibits G and H which you have pointed to during your
direct-examination, the phrase "Item no. 5 only" meaning to (sic) the two (2) swimming pools was deleted
from the policies issued by AIU, is it not?

xxx

ATTY. ANDRES:

As an insurance executive will you not attach any significance to the deletion of the qualifying phrase for the
policies?

WITNESS:

My answer to that would be, the deletion of that particular phrase is inadvertent. Being a company
underwriter, we do not cover. . it was inadvertent because of the previous policies that we have issued with
no specific attachments, premium rates and so on. It was inadvertent, sir.

The Court also rejects petitioners contention that respondents contemporaneous and subsequent acts to the issuance of the
insurance policy falsely gave the petitioner assurance that the coverage of the earthquake shock endorsement included all its
properties in the resort. Respondent only insured the properties as intended by the petitioner. Petitioners own witness
testified to this agreement, viz:

CROSS EXAMINATION OF LEOPOLDO MANTOHAC


TSN, January 14, 1992
pp. 4-5

Q. Just to be clear about this particular answer of yours Mr. Witness, what exactly did you tell Atty. Omlas (sic) to
copy from Exhibit "H" for purposes of procuring the policy from Philippine Charter Insurance Corporation?

A. I told him that the insurance that they will have to get will have the same provisions as this American Home
Insurance Policy No. 206-4568061-9.

Q. You are referring to Exhibit "H" of course?

A. Yes, sir, to Exhibit "H".

Q. So, all the provisions here will be the same except that of the premium rates?

A. Yes, sir. He assured me that with regards to the insurance premium rates that they will be charging will be limited
to this one. I (sic) can even be lesser.

CROSS EXAMINATION OF LEOPOLDO MANTOHAC


TSN, January 14, 1992
pp. 12-14

Atty. Mejia:
Q. Will it be correct to state[,] Mr. Witness, that you made a comparison of the provisions and scope of coverage of
Exhibits "I" and "H" sometime in the third week of March, 1990 or thereabout?

A. Yes, sir, about that time.

Q. And at that time did you notice any discrepancy or difference between the policy wordings as well as scope of
coverage of Exhibits "I" and "H" respectively?

A. No, sir, I did not discover any difference inasmuch (sic) as I was assured already that the policy wordings and
rates were copied from the insurance policy I sent them but it was only when this case erupted that we discovered
some discrepancies.

Q. With respect to the items declared for insurance coverage did you notice any discrepancy at any time between
those indicated in Exhibit "I" and those indicated in Exhibit "H" respectively?

A. With regard to the wordings I did not notice any difference because it was exactly the same P393,000.00 on the
two (2) swimming pools only against the peril of earthquake shock which I understood before that this provision will
have to be placed here because this particular provision under the peril of earthquake shock only is requested
because this is an insurance policy and therefore cannot be insured against fire, so this has to be placed.

The verbal assurances allegedly given by respondents representative Atty. Umlas were not proved. Atty. Umlas categorically
denied having given such assurances.

Finally, petitioner puts much stress on the letter of respondents independent claims adjuster, Bayne Adjusters and
Surveyors, Inc. But as testified to by the representative of Bayne Adjusters and Surveyors, Inc., respondent never meant to
lead petitioner to believe that the endorsement for earthquake shock covered properties other than the two swimming
pools, viz:

DIRECT EXAMINATION OF ALBERTO DE LEON (Bayne Adjusters and Surveyors, Inc.)


TSN, January 26, 1993
pp. 22-26

Q. Do you recall the circumstances that led to your discussion regarding the extent of coverage of the policy issued
by Philippine Charter Insurance Corporation?

A. I remember that when I returned to the office after the inspection, I got a photocopy of the insurance coverage
policy and it was indicated under Item 3 specifically that the coverage is only for earthquake shock. Then, I
remember I had a talk with Atty. Umlas (sic), and I relayed to him what I had found out in the policy and he confirmed
to me indeed only Item 3 which were the two swimming pools have coverage for earthquake shock.

xxx

Q. Now, may we know from you Engr. de Leon your basis, if any, for stating that except for the swimming pools all
affected items have no coverage for earthquake shock?

xxx

A. I based my statement on my findings, because upon my examination of the policy I found out that under Item 3 it
was specific on the wordings that on the two swimming pools only, then enclosed in parenthesis (against the peril[s]
of earthquake shock only), and secondly, when I examined the summary of premium payment only Item 3 which
refers to the swimming pools have a computation for premium payment for earthquake shock and all the other items
have no computation for payment of premiums.

In sum, there is no ambiguity in the terms of the contract and its riders. Petitioner cannot rely on the general rule that
insurance contracts are contracts of adhesion which should be liberally construed in favor of the insured and strictly against
the insurer company which usually prepares it.31 A contract of adhesion is one wherein a party, usually a corporation,
prepares the stipulations in the contract, while the other party merely affixes his signature or his "adhesion" thereto. Through
the years, the courts have held that in these type of contracts, the parties do not bargain on equal footing, the weaker party's
participation being reduced to the alternative to take it or leave it. Thus, these contracts are viewed as traps for the weaker
party whom the courts of justice must protect.32Consequently, any ambiguity therein is resolved against the insurer, or
construed liberally in favor of the insured. 33

The case law will show that this Court will only rule out blind adherence to terms where facts and circumstances will show
that they are basically one-sided.34 Thus, we have called on lower courts to remain careful in scrutinizing the factual
circumstances behind each case to determine the efficacy of the claims of contending parties. In Development Bank of the
Philippines v. National Merchandising Corporation, et al.,35 the parties, who were acute businessmen of experience, were
presumed to have assented to the assailed documents with full knowledge.

We cannot apply the general rule on contracts of adhesion to the case at bar. Petitioner cannot claim it did not know the
provisions of the policy. From the inception of the policy, petitioner had required the respondent to copyverbatim the
provisions and terms of its latest insurance policy from AHAC-AIU. The testimony of Mr. Leopoldo Mantohac, a direct
participant in securing the insurance policy of petitioner, is reflective of petitioners knowledge,viz:

DIRECT EXAMINATION OF LEOPOLDO MANTOHAC36


TSN, September 23, 1991
pp. 20-21

Q. Did you indicate to Atty. Omlas (sic) what kind of policy you would want for those facilities in Agoo Playa?

A. Yes, sir. I told him that I will agree to that renewal of this policy under Philippine Charter Insurance Corporation as
long as it will follow the same or exact provisions of the previous insurance policy we had with American Home
Assurance Corporation.

Q. Did you take any step Mr. Witness to ensure that the provisions which you wanted in the American Home
Insurance policy are to be incorporated in the PCIC policy?

A. Yes, sir.

Q. What steps did you take?

A. When I examined the policy of the Philippine Charter Insurance Corporation I specifically told him that the policy
and wordings shall be copied from the AIU Policy No. 206-4568061-9.

Respondent, in compliance with the condition set by the petitioner, copied AIU Policy No. 206-4568061-9 in drafting its
Insurance Policy No. 31944. It is true that there was variance in some terms, specifically in the replacement cost
endorsement, but the principal provisions of the policy remained essentially similar to AHAC-AIUs policy. Consequently, we
cannot apply the "fine print" or "contract of adhesion" rule in this case as the parties intent to limit the coverage of the policy
to the two swimming pools only is not ambiguous.37

IN VIEW WHEREOF, the judgment of the Court of Appeals is affirmed. The petition for certiorari is dismissed. No costs.

SO ORDERED.

Austria-Martinez, Callejo, Sr., Tinga, and Chico-Nazario, JJ., concur.

G.R. No. L-52756 October 12, 1987

MANILA MAHOGANY MANUFACTURING CORPORATION, petitioner,


vs.
COURT OF APPEALS AND ZENITH INSURANCE CORPORATION, respondents.

PADILLA, J:
Petition to review the decision * of the Court of Appeals, in CA-G.R. No. SP-08642, dated 21 March 1979, ordering petitioner Manila Mahogany
Manufacturing Corporation to pay private respondent Zenith Insurance Corporation the sum of Five Thousand Pesos (P5,000.00) with 6% annual interest from
18 January 1973, attorney's fees in the sum of five hundred pesos (P500.00), and costs of suit, and the resolution of the same Court, dated 8 February 1980,
denying petitioner's motion for reconsideration of it's decision.

From 6 March 1970 to 6 March 1971, petitioner insured its Mercedes Benz 4-door sedan with respondent insurance
company. On 4 May 1970 the insured vehicle was bumped and damaged by a truck owned by San Miguel Corporation. For
the damage caused, respondent company paid petitioner five thousand pesos (P5,000.00) in amicable settlement.
Petitioner's general manager executed a Release of Claim, subrogating respondent company to all its right to action against
San Miguel Corporation.

On 11 December 1972, respondent company wrote Insurance Adjusters, Inc. to demand reimbursement from San Miguel
Corporation of the amount it had paid petitioner. Insurance Adjusters, Inc. refused reimbursement, alleging that San Miguel
Corporation had already paid petitioner P4,500.00 for the damages to petitioner's motor vehicle, as evidenced by a cash
voucher and a Release of Claim executed by the General Manager of petitioner discharging San Miguel Corporation from "all
actions, claims, demands the rights of action that now exist or hereafter [sic] develop arising out of or as a consequence of
the accident."

Respondent insurance company thus demanded from petitioner reimbursement of the sum of P4,500.00 paid by San Miguel
Corporation. Petitioner refused; hence, respondent company filed suit in the City Court of Manila for the recovery of
P4,500.00. The City Court ordered petitioner to pay respondent P4,500.00. On appeal the Court of First Instance of Manila
affirmed the City Court's decision in toto, which CFI decision was affirmed by the Court of Appeals, with the modification that
petitioner was to pay respondent the total amount of P5,000.00 that it had earlier received from the respondent insurance
company.

Petitioner now contends it is not bound to pay P4,500.00, and much more, P5,000.00 to respondent company as the
subrogation in the Release of Claim it executed in favor of respondent was conditioned on recovery of the total amount of
damages petitioner had sustained. Since total damages were valued by petitioner at P9,486.43 and only P5,000.00 was
received by petitioner from respondent, petitioner argues that it was entitled to go after San Miguel Corporation to claim the
additional P4,500.00 eventually paid to it by the latter, without having to turn over said amount to respondent. Respondent of
course disputes this allegation and states that there was no qualification to its right of subrogation under the Release of
Claim executed by petitioner, the contents of said deed having expressed all the intents and purposes of the parties.

To support its alleged right not to return the P4,500.00 paid by San Miguel Corporation, petitioner cites Art. 2207 of the Civil
Code, which states:

If the plaintiff's property has been insured, and he has received indemnity from the insurance company for
the injury or loss arising out of the wrong or breach of contract complained of the insurance company shall
be subrogated to the rights of the insured against the wrongdoer or the person who has violated the
contract. If the amount paid by the insurance company does not fully cover the injury or loss the aggrieved
party shall be entitled to recover the deficiency from the person causing the loss or injury.

Petitioner also invokes Art. 1304 of the Civil Code, stating.

A creditor, to whom partial payment has been made, may exercise his right for the remainder, and he shall
be preferred to the person who has been subrogated in his place in virtue of the partial payment of the
same credit.

We find petitioners arguments to be untenable and without merit. In the absence of any other evidence to support its
allegation that a gentlemen's agreement existed between it and respondent, not embodied in the Release of Claim, such
ease of Claim must be taken as the best evidence of the intent and purpose of the parties. Thus, the Court of Appeals rightly
stated:

Petitioner argues that the release claim it executed subrogating Private respondent to any right of action it
had against San Miguel Corporation did not preclude Manila Mahogany from filing a deficiency claim
against the wrongdoer. Citing Article 2207, New Civil Code, to the effect that if the amount paid by an
insurance company does not fully cover the loss, the aggrieved party shall be entitled to recover the
deficiency from the person causing the loss, petitioner claims a preferred right to retain the amount coming
from San Miguel Corporation, despite the subrogation in favor of Private respondent.

Although petitioners right to file a deficiency claim against San Miguel Corporation is with legal basis,
without prejudice to the insurer's right of subrogation, nevertheless when Manila Mahogany executed
another release claim (Exhibit K) discharging San Miguel Corporation from "all actions, claims, demands
and rights of action that now exist or hereafter arising out of or as a consequence of the accident" after the
insurer had paid the proceeds of the policy- the compromise agreement of P5,000.00 being based on the
insurance policy-the insurer is entitled to recover from the insured the amount of insurance money paid
(Metropolitan Casualty Insurance Company of New York vs. Badler, 229 N.Y.S. 61, 132 Misc. 132 cited in
Insurance Code and Insolvency Law with comments and annotations, H.B. Perez 1976, p. 151). Since
petitioner by its own acts released San Miguel Corporation, thereby defeating private respondents, the right
of subrogation, the right of action of petitioner against the insurer was also nullified. (Sy Keng & Co. vs.
Queensland Insurance Co., Ltd., 54 O.G. 391) Otherwise stated: private respondent may recover the sum
of P5,000.00 it had earlier paid to petitioner. 1

As held in Phil. Air Lines v. Heald Lumber Co., 2

If a property is insured and the owner receives the indemnity from the insurer, it is provided in [Article 2207
of the New Civil Code] that the insurer is deemed subrogated to the rights of the insured against the
wrongdoer and if the amount paid by the insurer does not fully cover the loss, then the aggrieved party is
the one entitled to recover the deficiency. ... Under this legal provision, the real party in interest with regard
to the portion of the indemnity paid is the insurer and not the insured 3(Emphasis supplied)

The decision of the respondent court ordering petitioner to pay respondent company, not the P4,500.00 as originally asked
for, but P5,000.00, the amount respondent company paid petitioner as insurance, is also in accord with law and
jurisprudence. In disposing of this issue, the Court of Appeals held:

... petitioner is entitled to keep the sum of P4,500.00 paid by San Miguel Corporation under its clear right to
file a deficiency claim for damages incurred, against the wrongdoer, should the insurance company not fully
pay for the injury caused (Article 2207, New Civil Code). However, when petitioner released San Miguel
Corporation from any liability, petitioner's right to retain the sum of P5,000.00 no longer existed, thereby
entitling private respondent to recover the same. (Emphasis supplied)

As has been observed:

... The right of subrogation can only exist after the insurer has paid the otherwise the insured will be
deprived of his right to full indemnity. If the insurance proceeds are not sufficient to cover the damages
suffered by the insured, then he may sue the party responsible for the damage for the the [sic] remainder.
To the extent of the amount he has already received from the insurer enjoy's [sic] the right of subrogation.

Since the insurer can be subrogated to only such rights as the insured may have, should the insured, after
receiving payment from the insurer, release the wrongdoer who caused the loss, the insurer loses his rights
against the latter. But in such a case, the insurer will be entitled to recover from the insured whatever it has
paid to the latter, unless the release was made with the consent of the insurer. 4 (Emphasis supplied.)

And even if the specific amount asked for in the complaint is P4,500.00 only and not P5,000.00, still, the respondent Court
acted well within its discretion in awarding P5,000.00, the total amount paid by the insurer. The Court of Appeals rightly
reasoned as follows:

It is to be noted that private respondent, in its companies, prays for the recovery, not of P5,000.00 it had
paid under the insurance policy but P4,500.00 San Miguel Corporation had paid to petitioner. On this score,
We believe the City Court and Court of First Instance erred in not awarding the proper relief. Although
private respondent prays for the reimbursement of P4,500.00 paid by San Miguel Corporation, instead of
P5,000.00 paid under the insurance policy, the trial court should have awarded the latter, although not
prayed for, under the general prayer in the complaint "for such further or other relief as may be deemed just
or equitable, (Rule 6, Sec. 3, Revised Rules of Court; Rosales vs. Reyes Ordoveza, 25 Phil. 495 ; Cabigao
vs. Lim, 50 Phil. 844; Baguiro vs. Barrios Tupas, 77 Phil 120).

WHEREFORE, premises considered, the petition is DENIED. The judgment appealed from is hereby AFFIRMED with costs
against petitioner.

SO ORDERED.

Yap (Chairman), Melencio-Herrera, Paras and Sarmiento, JJ., concur.


G.R. No. 150094 August 18, 2004

FEDERAL EXPRESS CORPORATION, petitioner,


vs.
AMERICAN HOME ASSURANCE COMPANY and PHILAM INSURANCE COMPANY, INC., respondents.

DECISION
PANGANIBAN, J.:

Basic is the requirement that before suing to recover loss of or damage to transported goods, the plaintiff must give the
carrier notice of the loss or damage, within the period prescribed by the Warsaw Convention and/or the airway bill.

The Case

Before us is a Petition for Review1 under Rule 45 of the Rules of Court, challenging the June 4, 2001 Decision 2and the
September 21, 2001 Resolution3 of the Court of Appeals (CA) in CA-GR CV No. 58208. The assailed Decision disposed as
follows:

"WHEREFORE, premises considered, the present appeal is hereby DISMISSED for lack of merit. The appealed
Decision of Branch 149 of the Regional Trial Court of Makati City in Civil Case No. 95-1219,entitled 'American Home
Assurance Co. and PHILAM Insurance Co., Inc. v. FEDERAL EXPRESS CORPORATION and/or CARGOHAUS,
INC. (formerly U-WAREHOUSE, INC.),' is hereby AFFIRMED andREITERATED.

"Costs against the [petitioner and Cargohaus, Inc.]." 4

The assailed Resolution denied petitioner's Motion for Reconsideration.

The Facts

The antecedent facts are summarized by the appellate court as follows:

"On January 26, 1994, SMITHKLINE Beecham (SMITHKLINE for brevity) of Nebraska, USA delivered to Burlington
Air Express (BURLINGTON), an agent of [Petitioner] Federal Express Corporation, a shipment of 109 cartons of
veterinary biologicals for delivery to consignee SMITHKLINE and French Overseas Company in Makati City, Metro
Manila. The shipment was covered by Burlington Airway Bill No. 11263825 with the words, 'REFRIGERATE WHEN
NOT IN TRANSIT' and 'PERISHABLE' stamp marked on its face. That same day, Burlington insured the cargoes in
the amount of $39,339.00 with American Home Assurance Company (AHAC). The following day, Burlington turned
over the custody of said cargoes to Federal Express which transported the same to Manila. The first shipment,
consisting of 92 cartons arrived in Manila on January 29, 1994 in Flight No. 0071-28NRT and was immediately
stored at [Cargohaus Inc.'s] warehouse. While the second, consisting of 17 cartons, came in two (2) days later, or on
January 31, 1994, in Flight No. 0071-30NRT which was likewise immediately stored at Cargohaus' warehouse. Prior
to the arrival of the cargoes, Federal Express informed GETC Cargo International Corporation, the customs broker
hired by the consignee to facilitate the release of its cargoes from the Bureau of Customs, of the impending arrival of
its client's cargoes.

"On February 10, 1994, DARIO C. DIONEDA ('DIONEDA'), twelve (12) days after the cargoes arrived in Manila, a
non-licensed custom's broker who was assigned by GETC to facilitate the release of the subject cargoes, found out,
while he was about to cause the release of the said cargoes, that the same [were] stored only in a room with two (2)
air conditioners running, to cool the place instead of a refrigerator. When he asked an employee of Cargohaus why
the cargoes were stored in the 'cool room' only, the latter told him that the cartons where the vaccines were
contained specifically indicated therein that it should not be subjected to hot or cold temperature. Thereafter,
DIONEDA, upon instructions from GETC, did not proceed with the withdrawal of the vaccines and instead, samples
of the same were taken and brought to the Bureau of Animal Industry of the Department of Agriculture in the
Philippines by SMITHKLINE for examination wherein it was discovered that the 'ELISA reading of vaccinates sera
are below the positive reference serum.'

"As a consequence of the foregoing result of the veterinary biologics test, SMITHKLINE abandoned the shipment
and, declaring 'total loss' for the unusable shipment, filed a claim with AHAC through its representative in the
Philippines, the Philam Insurance Co., Inc. ('PHILAM') which recompensed SMITHKLINE for the whole insured
amount of THIRTY NINE THOUSAND THREE HUNDRED THIRTY NINE DOLLARS ($39,339.00). Thereafter,
[respondents] filed an action for damages against the [petitioner] imputing negligence on either or both of them in
the handling of the cargo.
"Trial ensued and ultimately concluded on March 18, 1997 with the [petitioner] being held solidarily liable for the loss
as follows:

'WHEREFORE, judgment is hereby rendered in favor of [respondents] and [petitioner and its Co-Defendant
Cargohaus] are directed to pay [respondents], jointly and severally, the following:

1. Actual damages in the amount of the peso equivalent of US$39,339.00 with interest from the
time of the filing of the complaint to the time the same is fully paid.

2. Attorney's fees in the amount of P50,000.00 and

3. Costs of suit.

'SO ORDERED.'

"Aggrieved, [petitioner] appealed to [the CA]."5

Ruling of the Court of Appeals

The Test Report issued by the United States Department of Agriculture (Animal and Plant Health Inspection Service) was
found by the CA to be inadmissible in evidence. Despite this ruling, the appellate court held that the shipping Receipts were a
prima facie proof that the goods had indeed been delivered to the carrier in good condition. We quote from the ruling as
follows:

"Where the plaintiff introduces evidence which shows prima facie that the goods were delivered to the carrier in good
condition [i.e., the shipping receipts], and that the carrier delivered the goods in a damaged condition, a presumption
is raised that the damage occurred through the fault or negligence of the carrier,and this casts upon the carrier the
burden of showing that the goods were not in good condition when delivered to the carrier, or that the damage was
occasioned by some cause excepting the carrier from absolute liability. This the [petitioner] failed to discharge. x x
x."6

Found devoid of merit was petitioner's claim that respondents had no personality to sue. This argument was supposedly not
raised in the Answer or during trial.

Hence, this Petition.7

The Issues

In its Memorandum, petitioner raises the following issues for our consideration:

"I.

Are the decision and resolution of the Honorable Court of Appeals proper subject for review by the Honorable Court
under Rule 45 of the 1997 Rules of Civil Procedure?

"II.

Is the conclusion of the Honorable Court of Appeals petitioner's claim that respondents have no personality to sue
because the payment was made by the respondents to Smithkline when the insured under the policy is Burlington
Air Express is devoid of merit correct or not?

"III.

Is the conclusion of the Honorable Court of Appeals that the goods were received in good condition, correct or not?

"IV.

Are Exhibits 'F' and 'G' hearsay evidence, and therefore, not admissible?
"V.

Is the Honorable Court of Appeals correct in ignoring and disregarding respondents' own admission that petitioner is
not liable? and

"VI.

Is the Honorable Court of Appeals correct in ignoring the Warsaw Convention?" 8

Simply stated, the issues are as follows: (1) Is the Petition proper for review by the Supreme Court? (2) Is Federal Express
liable for damage to or loss of the insured goods?

This Court's Ruling

The Petition has merit.

Preliminary Issue:
Propriety of Review

The correctness of legal conclusions drawn by the Court of Appeals from undisputed facts is a question of law cognizable by
the Supreme Court.9

In the present case, the facts are undisputed. As will be shown shortly, petitioner is questioning the conclusions drawn from
such facts. Hence, this case is a proper subject for review by this Court.

Main Issue:
Liability for Damages

Petitioner contends that respondents have no personality to sue -- thus, no cause of action against it -- because the payment
made to Smithkline was erroneous.

Pertinent to this issue is the Certificate of Insurance10 ("Certificate") that both opposing parties cite in support of their
respective positions. They differ only in their interpretation of what their rights are under its terms. The determination of those
rights involves a question of law, not a question of fact. "As distinguished from a question of law which exists 'when the doubt
or difference arises as to what the law is on a certain state of facts' -- 'there is a question of fact when the doubt or difference
arises as to the truth or the falsehood of alleged facts'; or when the 'query necessarily invites calibration of the whole
evidence considering mainly the credibility of witnesses, existence and relevancy of specific surrounding circumstance, their
relation to each other and to the whole and the probabilities of the situation.'" 11

Proper Payee

The Certificate specifies that loss of or damage to the insured cargo is "payable to order x x x upon surrender of this
Certificate." Such wording conveys the right of collecting on any such damage or loss, as fully as if the property were
covered by a special policy in the name of the holder itself. At the back of the Certificate appears the signature of the
representative of Burlington. This document has thus been duly indorsed in blank and is deemed a bearer instrument.

Since the Certificate was in the possession of Smithkline, the latter had the right of collecting or of being indemnified for loss
of or damage to the insured shipment, as fully as if the property were covered by a special policy in the name of the holder.
Hence, being the holder of the Certificate and having an insurable interest in the goods, Smithkline was the proper payee of
the insurance proceeds.

Subrogation

Upon receipt of the insurance proceeds, the consignee (Smithkline) executed a subrogation Receipt 12 in favor of
respondents. The latter were thus authorized "to file claims and begin suit against any such carrier, vessel, person,
corporation or government." Undeniably, the consignee had a legal right to receive the goods in the same condition it was
delivered for transport to petitioner. If that right was violated, the consignee would have a cause of action against the person
responsible therefor.
Upon payment to the consignee of an indemnity for the loss of or damage to the insured goods, the insurer's entitlement to
subrogation pro tanto -- being of the highest equity -- equips it with a cause of action in case of a contractual breach or
negligence.13 "Further, the insurer's subrogatory right to sue for recovery under the bill of lading in case of loss of or damage
to the cargo is jurisprudentially upheld."14

In the exercise of its subrogatory right, an insurer may proceed against an erring carrier. To all intents and purposes, it stands
in the place and in substitution of the consignee. A fortiori, both the insurer and the consignee are bound by the contractual
stipulations under the bill of lading.15

Prescription of Claim

From the initial proceedings in the trial court up to the present, petitioner has tirelessly pointed out that respondents' claim
and right of action are already barred. The latter, and even the consignee, never filed with the carrier any written notice or
complaint regarding its claim for damage of or loss to the subject cargo within the period required by the Warsaw Convention
and/or in the airway bill. Indeed, this fact has never been denied by respondents and is plainly evident from the records.

Airway Bill No. 11263825, issued by Burlington as agent of petitioner, states:

"6. No action shall be maintained in the case of damage to or partial loss of the shipment unless a written notice,
sufficiently describing the goods concerned, the approximate date of the damage or loss, and the details of the
claim, is presented by shipper or consignee to an office of Burlington within (14) days from the date the goods are
placed at the disposal of the person entitled to delivery, or in the case of total loss (including non-delivery) unless
presented within (120) days from the date of issue of the [Airway Bill]." 16

Relevantly, petitioner's airway bill states:

"12./12.1 The person entitled to delivery must make a complaint to the carrier in writing in the case:

12.1.1 of visible damage to the goods, immediately after discovery of the damage and at the latest within fourteen
(14) days from receipt of the goods;

12.1.2 of other damage to the goods, within fourteen (14) days from the date of receipt of the goods;

12.1.3 delay, within twenty-one (21) days of the date the goods are placed at his disposal; and

12.1.4 of non-delivery of the goods, within one hundred and twenty (120) days from the date of the issue of the air
waybill.

12.2 For the purpose of 12.1 complaint in writing may be made to the carrier whose air waybill was used, or to the
first carrier or to the last carrier or to the carrier who performed the transportation during which the loss, damage or
delay took place."17

Article 26 of the Warsaw Convention, on the other hand, provides:

"ART. 26. (1) Receipt by the person entitled to the delivery of baggage or goods without complaint shall be prima
facie evidence that the same have been delivered in good condition and in accordance with the document of
transportation.

(2) In case of damage, the person entitled to delivery must complain to the carrier forthwith after the discovery of the
damage, and, at the latest, within 3 days from the date of receipt in the case of baggage and 7 days from the date of
receipt in the case of goods. In case of delay the complaint must be made at the latest within 14 days from the date
on which the baggage or goods have been placed at his disposal.

(3) Every complaint must be made in writing upon the document of transportation or by separate notice in writing
dispatched within the times aforesaid.

(4) Failing complaint within the times aforesaid, no action shall lie against the carrier, save in the case of fraud on his
part."18
Condition Precedent

In this jurisdiction, the filing of a claim with the carrier within the time limitation therefor actually constitutes a condition
precedent to the accrual of a right of action against a carrier for loss of or damage to the goods. 19 The shipper or consignee
must allege and prove the fulfillment of the condition. If it fails to do so, no right of action against the carrier can accrue in
favor of the former. The aforementioned requirement is a reasonable condition precedent; it does not constitute a limitation of
action.20

The requirement of giving notice of loss of or injury to the goods is not an empty formalism. The fundamental reasons for
such a stipulation are (1) to inform the carrier that the cargo has been damaged, and that it is being charged with liability
therefor; and (2) to give it an opportunity to examine the nature and extent of the injury. "This protects the carrier by affording
it an opportunity to make an investigation of a claim while the matter is fresh and easily investigated so as to safeguard itself
from false and fraudulent claims."21

When an airway bill -- or any contract of carriage for that matter -- has a stipulation that requires a notice of claim for loss of
or damage to goods shipped and the stipulation is not complied with, its enforcement can be prevented and the liability
cannot be imposed on the carrier. To stress, notice is a condition precedent, and the carrier is not liable if notice is not given
in accordance with the stipulation.22 Failure to comply with such a stipulation bars recovery for the loss or damage suffered. 23

Being a condition precedent, the notice must precede a suit for enforcement. 24 In the present case, there is neither an
allegation nor a showing of respondents' compliance with this requirement within the prescribed period. While respondents
may have had a cause of action then, they cannot now enforce it for their failure to comply with the aforesaid condition
precedent.

In view of the foregoing, we find no more necessity to pass upon the other issues raised by petitioner.

We note that respondents are not without recourse. Cargohaus, Inc. -- petitioner's co-defendant in respondents' Complaint
below -- has been adjudged by the trial court as liable for, inter alia, "actual damages in the amount of the peso equivalent of
US $39,339."25 This judgment was affirmed by the Court of Appeals and is already final and executory. 26

WHEREFORE, the Petition is GRANTED, and the assailed Decision REVERSED insofar as it pertains to Petitioner Federal
Express Corporation. No pronouncement as to costs.

SO ORDERED.

Corona, and Carpio-Morales, JJ., concur.


Sandoval-Gutierrez, J., on leave.

G.R. No. 166245 April 9, 2008

ETERNAL GARDENS MEMORIAL PARK CORPORATION, petitioner,


vs.
THE PHILIPPINE AMERICAN LIFE INSURANCE COMPANY, respondent.

DECISION

VELASCO, JR., J.:

The Case

Central to this Petition for Review on Certiorari under Rule 45 which seeks to reverse and set aside the November 26, 2004
Decision1 of the Court of Appeals (CA) in CA-G.R. CV No. 57810 is the query: May the inaction of the insurer on the
insurance application be considered as approval of the application?
The Facts

On December 10, 1980, respondent Philippine American Life Insurance Company (Philamlife) entered into an agreement
denominated as Creditor Group Life Policy No. P-19202 with petitioner Eternal Gardens Memorial Park Corporation (Eternal).
Under the policy, the clients of Eternal who purchased burial lots from it on installment basis would be insured by Philamlife.
The amount of insurance coverage depended upon the existing balance of the purchased burial lots. The policy was to be
effective for a period of one year, renewable on a yearly basis.

The relevant provisions of the policy are:

ELIGIBILITY.

Any Lot Purchaser of the Assured who is at least 18 but not more than 65 years of age, is indebted to the Assured
for the unpaid balance of his loan with the Assured, and is accepted for Life Insurance coverage by the Company on
its effective date is eligible for insurance under the Policy.

EVIDENCE OF INSURABILITY.

No medical examination shall be required for amounts of insurance up to P50,000.00. However, a declaration of
good health shall be required for all Lot Purchasers as part of the application. The Company reserves the right to
require further evidence of insurability satisfactory to the Company in respect of the following:

1. Any amount of insurance in excess of P50,000.00.

2. Any lot purchaser who is more than 55 years of age.

LIFE INSURANCE BENEFIT.

The Life Insurance coverage of any Lot Purchaser at any time shall be the amount of the unpaid balance of his loan
(including arrears up to but not exceeding 2 months) as reported by the Assured to the Company or the sum of
P100,000.00, whichever is smaller. Such benefit shall be paid to the Assured if the Lot Purchaser dies while insured
under the Policy.

EFFECTIVE DATE OF BENEFIT.

The insurance of any eligible Lot Purchaser shall be effective on the date he contracts a loan with the Assured.
However, there shall be no insurance if the application of the Lot Purchaser is not approved by the Company. 3

Eternal was required under the policy to submit to Philamlife a list of all new lot purchasers, together with a copy of the
application of each purchaser, and the amounts of the respective unpaid balances of all insured lot purchasers. In relation to
the instant petition, Eternal complied by submitting a letter dated December 29, 1982, 4containing a list of insurable balances
of its lot buyers for October 1982. One of those included in the list as "new business" was a certain John Chuang. His
balance of payments was PhP 100,000. On August 2, 1984, Chuang died.

Eternal sent a letter dated August 20, 19845 to Philamlife, which served as an insurance claim for Chuangs death. Attached
to the claim were the following documents: (1) Chuangs Certificate of Death; (2) Identification Certificate stating that Chuang
is a naturalized Filipino Citizen; (3) Certificate of Claimant; (4) Certificate of Attending Physician; and (5) Assureds
Certificate.

In reply, Philamlife wrote Eternal a letter on November 12, 1984, 6 requiring Eternal to submit the following documents relative
to its insurance claim for Chuangs death: (1) Certificate of Claimant (with form attached); (2) Assureds Certificate (with form
attached); (3) Application for Insurance accomplished and signed by the insured, Chuang, while still living; and (4) Statement
of Account showing the unpaid balance of Chuang before his death.

Eternal transmitted the required documents through a letter dated November 14, 1984, 7 which was received by Philamlife on
November 15, 1984.

After more than a year, Philamlife had not furnished Eternal with any reply to the latters insurance claim. This prompted
Eternal to demand from Philamlife the payment of the claim for PhP 100,000 on April 25, 1986. 8
In response to Eternals demand, Philamlife denied Eternals insurance claim in a letter dated May 20, 1986, 9 a portion of
which reads:

The deceased was 59 years old when he entered into Contract #9558 and 9529 with Eternal Gardens Memorial
Park in October 1982 for the total maximum insurable amount of P100,000.00 each. No application for Group
Insurance was submitted in our office prior to his death on August 2, 1984.

In accordance with our Creditors Group Life Policy No. P-1920, under Evidence of Insurability provision, "a
declaration of good health shall be required for all Lot Purchasers as party of the application." We cite further the
provision on Effective Date of Coverage under the policy which states that "there shall be no insurance if the
application is not approved by the Company." Since no application had been submitted by the Insured/Assured, prior
to his death, for our approval but was submitted instead on November 15, 1984, after his death, Mr. John Uy
Chuang was not covered under the Policy. We wish to point out that Eternal Gardens being the Assured was a party
to the Contract and was therefore aware of these pertinent provisions.

With regard to our acceptance of premiums, these do not connote our approval per se of the insurance coverage but
are held by us in trust for the payor until the prerequisites for insurance coverage shall have been met. We will
however, return all the premiums which have been paid in behalf of John Uy Chuang.

Consequently, Eternal filed a case before the Makati City Regional Trial Court (RTC) for a sum of money against Philamlife,
docketed as Civil Case No. 14736. The trial court decided in favor of Eternal, the dispositive portion of which reads:

WHEREFORE, premises considered, judgment is hereby rendered in favor of Plaintiff ETERNAL, against Defendant
PHILAMLIFE, ordering the Defendant PHILAMLIFE, to pay the sum of P100,000.00, representing the proceeds of
the Policy of John Uy Chuang, plus legal rate of interest, until fully paid; and, to pay the sum of P10,000.00 as
attorneys fees.

SO ORDERED.

The RTC found that Eternal submitted Chuangs application for insurance which he accomplished before his death, as
testified to by Eternals witness and evidenced by the letter dated December 29, 1982, stating, among others: "Encl: Phil-Am
Life Insurance Application Forms & Cert."10 It further ruled that due to Philamlifes inaction from the submission of the
requirements of the group insurance on December 29, 1982 to Chuangs death on August 2, 1984, as well as Philamlifes
acceptance of the premiums during the same period, Philamlife was deemed to have approved Chuangs application. The
RTC said that since the contract is a group life insurance, once proof of death is submitted, payment must follow.

Philamlife appealed to the CA, which ruled, thus:

WHEREFORE, the decision of the Regional Trial Court of Makati in Civil Case No. 57810 is REVERSED and SET
ASIDE, and the complaint is DISMISSED. No costs.

SO ORDERED.11

The CA based its Decision on the factual finding that Chuangs application was not enclosed in Eternals letter dated
December 29, 1982. It further ruled that the non-accomplishment of the submitted application form violated Section 26 of the
Insurance Code. Thus, the CA concluded, there being no application form, Chuang was not covered by Philamlifes
insurance.

Hence, we have this petition with the following grounds:

The Honorable Court of Appeals has decided a question of substance, not therefore determined by this Honorable
Court, or has decided it in a way not in accord with law or with the applicable jurisprudence, in holding that:

I. The application for insurance was not duly submitted to respondent PhilamLife before the death of John
Chuang;

II. There was no valid insurance coverage; and

III. Reversing and setting aside the Decision of the Regional Trial Court dated May 29, 1996.
The Courts Ruling

As a general rule, this Court is not a trier of facts and will not re-examine factual issues raised before the CA and first level
courts, considering their findings of facts are conclusive and binding on this Court. However, such rule is subject to
exceptions, as enunciated in Sampayan v. Court of Appeals:

(1) when the findings are grounded entirely on speculation, surmises or conjectures; (2) when the inference made is
manifestly mistaken, absurd or impossible; (3) when there is grave abuse of discretion; (4) when the judgment is
based on a misapprehension of facts; (5) when the findings of facts are conflicting; (6) when in making its findings
the [CA] went beyond the issues of the case, or its findings are contrary to the admissions of both the appellant and
the appellee; (7) when the findings [of the CA] are contrary to the trial court; (8) when the findings are
conclusions without citation of specific evidence on which they are based; (9) when the facts set forth in the petition
as well as in the petitioners main and reply briefs are not disputed by the respondent; (10) when the findings of fact
are premised on the supposed absence of evidence and contradicted by the evidence on record; and (11) when the
Court of Appeals manifestly overlooked certain relevant facts not disputed by the parties, which, if properly
considered, would justify a different conclusion.12 (Emphasis supplied.)

In the instant case, the factual findings of the RTC were reversed by the CA; thus, this Court may review them.

Eternal claims that the evidence that it presented before the trial court supports its contention that it submitted a copy of the
insurance application of Chuang before his death. In Eternals letter dated December 29, 1982, a list of insurable interests of
buyers for October 1982 was attached, including Chuang in the list of new businesses. Eternal added it was noted at the
bottom of said letter that the corresponding "Phil-Am Life Insurance Application Forms & Cert." were enclosed in the letter
that was apparently received by Philamlife on January 15, 1983. Finally, Eternal alleged that it provided a copy of the
insurance application which was signed by Chuang himself and executed before his death.

On the other hand, Philamlife claims that the evidence presented by Eternal is insufficient, arguing that Eternal must present
evidence showing that Philamlife received a copy of Chuangs insurance application.

The evidence on record supports Eternals position.

The fact of the matter is, the letter dated December 29, 1982, which Philamlife stamped as received, states that the
insurance forms for the attached list of burial lot buyers were attached to the letter. Such stamp of receipt has the effect of
acknowledging receipt of the letter together with the attachments. Such receipt is an admission by Philamlife against its own
interest.13 The burden of evidence has shifted to Philamlife, which must prove that the letter did not contain Chuangs
insurance application. However, Philamlife failed to do so; thus, Philamlife is deemed to have received Chuangs insurance
application.

To reiterate, it was Philamlifes bounden duty to make sure that before a transmittal letter is stamped as received, the
contents of the letter are correct and accounted for.

Philamlifes allegation that Eternals witnesses ran out of credibility and reliability due to inconsistencies is groundless. The
trial court is in the best position to determine the reliability and credibility of the witnesses, because it has the opportunity to
observe firsthand the witnesses demeanor, conduct, and attitude. Findings of the trial court on such matters are binding and
conclusive on the appellate court, unless some facts or circumstances of weight and substance have been overlooked,
misapprehended, or misinterpreted,14 that, if considered, might affect the result of the case.15

An examination of the testimonies of the witnesses mentioned by Philamlife, however, reveals no overlooked facts of
substance and value.

Philamlife primarily claims that Eternal did not even know where the original insurance application of Chuang was, as shown
by the testimony of Edilberto Mendoza:

Atty. Arevalo:

Q Where is the original of the application form which is required in case of new coverage?

[Mendoza:]
A It is [a] standard operating procedure for the new client to fill up two copies of this form and the original of this is
submitted to Philamlife together with the monthly remittances and the second copy is remained or retained with the
marketing department of Eternal Gardens.

Atty. Miranda:

We move to strike out the answer as it is not responsive as counsel is merely asking for the location and does not
[ask] for the number of copy.

Atty. Arevalo:

Q Where is the original?

[Mendoza:]

A As far as I remember I do not know where the original but when I submitted with that payment together with the
new clients all the originals I see to it before I sign the transmittal letter the originals are attached therein. 16

In other words, the witness admitted not knowing where the original insurance application was, but believed that the
application was transmitted to Philamlife as an attachment to a transmittal letter.

As to the seeming inconsistencies between the testimony of Manuel Cortez on whether one or two insurance application
forms were accomplished and the testimony of Mendoza on who actually filled out the application form, these are minor
inconsistencies that do not affect the credibility of the witnesses. Thus, we ruled in People v. Paredes that minor
inconsistencies are too trivial to affect the credibility of witnesses, and these may even serve to strengthen their credibility as
these negate any suspicion that the testimonies have been rehearsed. 17

We reiterated the above ruling in Merencillo v. People:

Minor discrepancies or inconsistencies do not impair the essential integrity of the prosecutions evidence as a whole
or reflect on the witnesses honesty. The test is whether the testimonies agree on essential facts and whether the
respective versions corroborate and substantially coincide with each other so as to make a consistent and coherent
whole.18

In the present case, the number of copies of the insurance application that Chuang executed is not at issue, neither is
whether the insurance application presented by Eternal has been falsified. Thus, the inconsistencies pointed out by
Philamlife are minor and do not affect the credibility of Eternals witnesses.

However, the question arises as to whether Philamlife assumed the risk of loss without approving the application.

This question must be answered in the affirmative.

As earlier stated, Philamlife and Eternal entered into an agreement denominated as Creditor Group Life Policy No. P-1920
dated December 10, 1980. In the policy, it is provided that:

EFFECTIVE DATE OF BENEFIT.

The insurance of any eligible Lot Purchaser shall be effective on the date he contracts a loan with the Assured.
However, there shall be no insurance if the application of the Lot Purchaser is not approved by the Company.

An examination of the above provision would show ambiguity between its two sentences. The first sentence appears to state
that the insurance coverage of the clients of Eternal already became effective upon contracting a loan with Eternal while the
second sentence appears to require Philamlife to approve the insurance contract before the same can become effective.

It must be remembered that an insurance contract is a contract of adhesion which must be construed liberally in favor of the
insured and strictly against the insurer in order to safeguard the latters interest. Thus, in Malayan Insurance Corporation v.
Court of Appeals, this Court held that:
Indemnity and liability insurance policies are construed in accordance with the general rule of resolving any
ambiguity therein in favor of the insured, where the contract or policy is prepared by the insurer. A contract of
insurance, being a contract of adhesion, par excellence, any ambiguity therein should be resolved against
the insurer; in other words, it should be construed liberally in favor of the insured and strictly against the insurer.
Limitations of liability should be regarded with extreme jealousy and must be construed in such a way as to preclude
the insurer from noncompliance with its obligations. 19 (Emphasis supplied.)

In the more recent case of Philamcare Health Systems, Inc. v. Court of Appeals, we reiterated the above ruling, stating that:

When the terms of insurance contract contain limitations on liability, courts should construe them in such a way as to
preclude the insurer from non-compliance with his obligation. Being a contract of adhesion, the terms of an
insurance contract are to be construed strictly against the party which prepared the contract, the insurer. By reason
of the exclusive control of the insurance company over the terms and phraseology of the insurance contract,
ambiguity must be strictly interpreted against the insurer and liberally in favor of the insured, especially to avoid
forfeiture.20

Clearly, the vague contractual provision, in Creditor Group Life Policy No. P-1920 dated December 10, 1980, must be
construed in favor of the insured and in favor of the effectivity of the insurance contract.

On the other hand, the seemingly conflicting provisions must be harmonized to mean that upon a partys purchase of a
memorial lot on installment from Eternal, an insurance contract covering the lot purchaser is created and the same is
effective, valid, and binding until terminated by Philamlife by disapproving the insurance application. The second sentence of
Creditor Group Life Policy No. P-1920 on the Effective Date of Benefit is in the nature of a resolutory condition which would
lead to the cessation of the insurance contract. Moreover, the mere inaction of the insurer on the insurance application must
not work to prejudice the insured; it cannot be interpreted as a termination of the insurance contract. The termination of the
insurance contract by the insurer must be explicit and unambiguous.

As a final note, to characterize the insurer and the insured as contracting parties on equal footing is inaccurate at best.
Insurance contracts are wholly prepared by the insurer with vast amounts of experience in the industry purposefully used to
its advantage. More often than not, insurance contracts are contracts of adhesion containing technical terms and conditions
of the industry, confusing if at all understandable to laypersons, that are imposed on those who wish to avail of insurance. As
such, insurance contracts are imbued with public interest that must be considered whenever the rights and obligations of the
insurer and the insured are to be delineated. Hence, in order to protect the interest of insurance applicants, insurance
companies must be obligated to act with haste upon insurance applications, to either deny or approve the same, or otherwise
be bound to honor the application as a valid, binding, and effective insurance contract. 21

WHEREFORE, we GRANT the petition. The November 26, 2004 CA Decision in CA-G.R. CV No. 57810
isREVERSED and SET ASIDE. The May 29, 1996 Decision of the Makati City RTC, Branch 138 is MODIFIED. Philamlife is
hereby ORDERED:

(1) To pay Eternal the amount of PhP 100,000 representing the proceeds of the Life Insurance Policy of Chuang;

(2) To pay Eternal legal interest at the rate of six percent (6%) per annum of PhP 100,000 from the time of extra-
judicial demand by Eternal until Philamlifes receipt of the May 29, 1996 RTC Decision on June 17, 1996;

(3) To pay Eternal legal interest at the rate of twelve percent (12%) per annum of PhP 100,000 from June 17, 1996
until full payment of this award; and

(4) To pay Eternal attorneys fees in the amount of PhP 10,000.

No costs.

SO ORDERED.

Carpio-Morales, Acting Chairperson, Tinga, Brion, Chico-Nazario *, JJ., concur.


II. Insurable Interest
G.R. No. L-15895 November 29, 1920
RAFAEL ENRIQUEZ, as administrator of the estate of the late Joaquin Ma. Herrer, plaintiff-appellant,
vs.
SUN LIFE ASSURANCE COMPANY OF CANADA, defendant-appellee.
Jose A. Espiritu for appellant.
Cohn, Fisher and DeWitt for appellee.

MALCOLM, J.:

This is an action brought by the plaintiff ad administrator of the estate of the late Joaquin Ma. Herrer to recover from the
defendant life insurance company the sum of pesos 6,000 paid by the deceased for a life annuity. The trial court gave
judgment for the defendant. Plaintiff appeals.
The undisputed facts are these: On September 24, 1917, Joaquin Herrer made application to the Sun Life Assurance
Company of Canada through its office in Manila for a life annuity. Two days later he paid the sum of P6,000 to the manager
of the company's Manila office and was given a receipt reading as follows:

MANILA, I. F., 26 de septiembre, 1917.

PROVISIONAL RECEIPT Pesos 6,000

Recibi la suma de seis mil pesos de Don Joaquin Herrer de Manila como prima dela Renta Vitalicia solicitada por dicho Don
Joaquin Herrer hoy, sujeta al examen medico y aprobacion de la Oficina Central de la Compaia.

The application was immediately forwarded to the head office of the company at Montreal, Canada. On November 26, 1917,
the head office gave notice of acceptance by cable to Manila. (Whether on the same day the cable was received notice was
sent by the Manila office of Herrer that the application had been accepted, is a disputed point, which will be discussed later.)
On December 4, 1917, the policy was issued at Montreal. On December 18, 1917, attorney Aurelio A. Torres wrote to the
Manila office of the company stating that Herrer desired to withdraw his application. The following day the local office replied
to Mr. Torres, stating that the policy had been issued, and called attention to the notification of November 26, 1917. This letter
was received by Mr. Torres on the morning of December 21, 1917. Mr. Herrer died on December 20, 1917.

As above suggested, the issue of fact raised by the evidence is whether Herrer received notice of acceptance of his
application. To resolve this question, we propose to go directly to the evidence of record.

The chief clerk of the Manila office of the Sun Life Assurance Company of Canada at the time of the trial testified that he
prepared the letter introduced in evidence as Exhibit 3, of date November 26, 1917, and handed it to the local manager, Mr.
E. E. White, for signature. The witness admitted on cross-examination that after preparing the letter and giving it to he
manager, he new nothing of what became of it. The local manager, Mr. White, testified to having received the cablegram
accepting the application of Mr. Herrer from the home office on November 26, 1917. He said that on the same day he signed
a letter notifying Mr. Herrer of this acceptance. The witness further said that letters, after being signed, were sent to the chief
clerk and placed on the mailing desk for transmission. The witness could not tell if the letter had every actually been placed
in the mails. Mr. Tuason, who was the chief clerk, on November 26, 1917, was not called as a witness. For the defense,
attorney Manuel Torres testified to having prepared the will of Joaquin Ma. Herrer, that on this occasion, Mr. Herrer
mentioned his application for a life annuity, and that he said that the only document relating to the transaction in his
possession was the provisional receipt. Rafael Enriquez, the administrator of the estate, testified that he had gone through
the effects of the deceased and had found no letter of notification from the insurance company to Mr. Herrer.

Our deduction from the evidence on this issue must be that the letter of November 26, 1917, notifying Mr. Herrer that his
application had been accepted, was prepared and signed in the local office of the insurance company, was placed in the
ordinary channels for transmission, but as far as we know, was never actually mailed and thus was never received by the
applicant.

Not forgetting our conclusion of fact, it next becomes necessary to determine the law which should be applied to the facts. In
order to reach our legal goal, the obvious signposts along the way must be noticed.

Until quite recently, all of the provisions concerning life insurance in the Philippines were found in the Code of Commerce and
the Civil Code. In the Code of the Commerce, there formerly existed Title VIII of Book III and Section III of Title III of Book III,
which dealt with insurance contracts. In the Civil Code there formerly existed and presumably still exist, Chapters II and IV,
entitled insurance contracts and life annuities, respectively, of Title XII of Book IV. On the after July 1, 1915, there was,
however, in force the Insurance Act. No. 2427. Chapter IV of this Act concerns life and health insurance. The Act expressly
repealed Title VIII of Book II and Section III of Title III of Book III of the code of Commerce. The law of insurance is
consequently now found in the Insurance Act and the Civil Code.

While, as just noticed, the Insurance Act deals with life insurance, it is silent as to the methods to be followed in order that
there may be a contract of insurance. On the other hand, the Civil Code, in article 1802, not only describes a contact of life
annuity markedly similar to the one we are considering, but in two other articles, gives strong clues as to the proper
disposition of the case. For instance, article 16 of the Civil Code provides that "In matters which are governed by special
laws, any deficiency of the latter shall be supplied by the provisions of this Code." On the supposition, therefore, which is
incontestable, that the special law on the subject of insurance is deficient in enunciating the principles governing acceptance,
the subject-matter of the Civil code, if there be any, would be controlling. In the Civil Code is found article 1262 providing that
"Consent is shown by the concurrence of offer and acceptance with respect to the thing and the consideration which are to
constitute the contract. An acceptance made by letter shall not bind the person making the offer except from the time it came
to his knowledge. The contract, in such case, is presumed to have been entered into at the place where the offer was made."
This latter article is in opposition to the provisions of article 54 of the Code of Commerce.

If no mistake has been made in announcing the successive steps by which we reach a conclusion, then the only duty
remaining is for the court to apply the law as it is found. The legislature in its wisdom having enacted a new law on insurance,
and expressly repealed the provisions in the Code of Commerce on the same subject, and having thus left a void in the
commercial law, it would seem logical to make use of the only pertinent provision of law found in the Civil code, closely
related to the chapter concerning life annuities.

The Civil Code rule, that an acceptance made by letter shall bind the person making the offer only from the date it came to
his knowledge, may not be the best expression of modern commercial usage. Still it must be admitted that its enforcement
avoids uncertainty and tends to security. Not only this, but in order that the principle may not be taken too lightly, let it be
noticed that it is identical with the principles announced by a considerable number of respectable courts in the United States.
The courts who take this view have expressly held that an acceptance of an offer of insurance not actually or constructively
communicated to the proposer does not make a contract. Only the mailing of acceptance, it has been said, completes the
contract of insurance, as the locus poenitentiae is ended when the acceptance has passed beyond the control of the party. (I
Joyce, The Law of Insurance, pp. 235, 244.)

In resume, therefore, the law applicable to the case is found to be the second paragraph of article 1262 of the Civil Code
providing that an acceptance made by letter shall not bind the person making the offer except from the time it came to his
knowledge. The pertinent fact is, that according to the provisional receipt, three things had to be accomplished by the
insurance company before there was a contract: (1) There had to be a medical examination of the applicant; (2) there had to
be approval of the application by the head office of the company; and (3) this approval had in some way to be communicated
by the company to the applicant. The further admitted facts are that the head office in Montreal did accept the application, did
cable the Manila office to that effect, did actually issue the policy and did, through its agent in Manila, actually write the letter
of notification and place it in the usual channels for transmission to the addressee. The fact as to the letter of notification thus
fails to concur with the essential elements of the general rule pertaining to the mailing and delivery of mail matter as
announced by the American courts, namely, when a letter or other mail matter is addressed and mailed with postage prepaid
there is a rebuttable presumption of fact that it was received by the addressee as soon as it could have been transmitted to
him in the ordinary course of the mails. But if any one of these elemental facts fails to appear, it is fatal to the presumption.
For instance, a letter will not be presumed to have been received by the addressee unless it is shown that it was deposited in
the post-office, properly addressed and stamped. (See 22 C.J., 96, and 49 L. R. A. [N. S.], pp. 458, et seq., notes.)

We hold that the contract for a life annuity in the case at bar was not perfected because it has not been proved satisfactorily
that the acceptance of the application ever came to the knowledge of the applicant. lawph!l.net

Judgment is reversed, and the plaintiff shall have and recover from the defendant the sum of P6,000 with legal interest from
November 20, 1918, until paid, without special finding as to costs in either instance. So ordered.

Mapa, C.J., Araullo, Avancea and Villamor, JJ., concur.


Johnson, J., dissents.

G.R. No. L-109937 March 21, 1994

DEVELOPMENT BANK OF THE PHILIPPINES, petitioner,


vs.
COURT OF APPEALS and the ESTATE OF THE LATE JUAN B. DANS, represented by CANDIDA G. DANS, and the
DBP MORTGAGE REDEMPTION INSURANCE POOL, respondents.

Office of the Legal Counsel for petitioner.

Reyes, Santayana, Molo & Alegre for DBP Mortgage Redemption Insurance Pool.

QUIASON, J.:
This is a petition for review on certiorari under Rule 45 of the Revised Rules of Court to reverse and set aside the decision of
the Court of Appeals in CA-G.R CV No. 26434 and its resolution denying reconsideration thereof.

We affirm the decision of the Court of Appeals with modification.

In May 1987, Juan B. Dans, together with his wife Candida, his son and daughter-in-law, applied for a loan of P500,000.00
with the Development Bank of the Philippines (DBP), Basilan Branch. As the principal mortgagor, Dans, then 76 years of age,
was advised by DBP to obtain a mortgage redemption insurance (MRI) with the DBP Mortgage Redemption Insurance Pool
(DBP MRI Pool).

A loan, in the reduced amount of P300,000.00, was approved by DBP on August 4, 1987 and released on August 11, 1987.
From the proceeds of the loan, DBP deducted the amount of P1,476.00 as payment for the MRI premium. On August 15,
1987, Dans accomplished and submitted the "MRI Application for Insurance" and the "Health Statement for DBP MRI Pool."

On August 20, 1987, the MRI premium of Dans, less the DBP service fee of 10 percent, was credited by DBP to the savings
account of the DBP MRI Pool. Accordingly, the DBP MRI Pool was advised of the credit.

On September 3, 1987, Dans died of cardiac arrest. The DBP, upon notice, relayed this information to the DBP MRI Pool. On
September 23, 1987, the DBP MRI Pool notified DBP that Dans was not eligible for MRI coverage, being over the
acceptance age limit of 60 years at the time of application.

On October 21, 1987, DBP apprised Candida Dans of the disapproval of her late husband's MRI application. The DBP
offered to refund the premium of P1,476.00 which the deceased had paid, but Candida Dans refused to accept the same,
demanding payment of the face value of the MRI or an amount equivalent to the loan. She, likewise, refused to accept an ex
gratia settlement of P30,000.00, which the DBP later offered.

On February 10, 1989, respondent Estate, through Candida Dans as administratrix, filed a complaint with the Regional Trial
Court, Branch I, Basilan, against DBP and the insurance pool for "Collection of Sum of Money with Damages." Respondent
Estate alleged that Dans became insured by the DBP MRI Pool when DBP, with full knowledge of Dans' age at the time of
application, required him to apply for MRI, and later collected the insurance premium thereon. Respondent Estate therefore
prayed: (1) that the sum of P139,500.00, which it paid under protest for the loan, be reimbursed; (2) that the mortgage debt
of the deceased be declared fully paid; and (3) that damages be awarded.

The DBP and the DBP MRI Pool separately filed their answers, with the former asserting a cross-claim against the latter.

At the pre-trial, DBP and the DBP MRI Pool admitted all the documents and exhibits submitted by respondent Estate. As a
result of these admissions, the trial court narrowed down the issues and, without opposition from the parties, found the case
ripe for summary judgment. Consequently, the trial court ordered the parties to submit their respective position papers and
documentary evidence, which may serve as basis for the judgment.

On March 10, 1990, the trial court rendered a decision in favor of respondent Estate and against DBP. The DBP MRI Pool,
however, was absolved from liability, after the trial court found no privity of contract between it and the deceased. The trial
court declared DBP in estoppel for having led Dans into applying for MRI and actually collecting the premium and the service
fee, despite knowledge of his age ineligibility. The dispositive portion of the decision read as follows:

WHEREFORE, in view of the foregoing consideration and in the furtherance of justice and equity, the Court
finds judgment for the plaintiff and against Defendant DBP, ordering the latter:

1. To return and reimburse plaintiff the amount of P139,500.00 plus legal rate of interest as amortization
payment paid under protest;

2. To consider the mortgage loan of P300,000.00 including all interest accumulated or otherwise to have
been settled, satisfied or set-off by virtue of the insurance coverage of the late Juan B. Dans;

3. To pay plaintiff the amount of P10,000.00 as attorney's fees;

4. To pay plaintiff in the amount of P10,000.00 as costs of litigation and other expenses, and other relief just
and equitable.
The Counterclaims of Defendants DBP and DBP MRI POOL are hereby dismissed. The Cross-claim of
Defendant DBP is likewise dismissed (Rollo, p. 79)

The DBP appealed to the Court of Appeals. In a decision dated September 7, 1992, the appellate court affirmedin toto the
decision of the trial court. The DBP's motion for reconsideration was denied in a resolution dated April 20, 1993.

Hence, this recourse.

II

When Dans applied for MRI, he filled up and personally signed a "Health Statement for DBP MRI Pool" (Exh. "5-Bank") with
the following declaration:

I hereby declare and agree that all the statements and answers contained herein are true, complete and
correct to the best of my knowledge and belief and form part of my application for insurance. It is
understood and agreed that no insurance coverage shall be effected unless and until this application is
approved and the full premium is paid during my continued good health (Records, p. 40).

Under the aforementioned provisions, the MRI coverage shall take effect: (1) when the application shall be approved by the
insurance pool; and (2) when the full premium is paid during the continued good health of the applicant. These two
conditions, being joined conjunctively, must concur.

Undisputably, the power to approve MRI applications is lodged with the DBP MRI Pool. The pool, however, did not approve
the application of Dans. There is also no showing that it accepted the sum of P1,476.00, which DBP credited to its account
with full knowledge that it was payment for Dan's premium. There was, as a result, no perfected contract of insurance; hence,
the DBP MRI Pool cannot be held liable on a contract that does not exist.

The liability of DBP is another matter.

It was DBP, as a matter of policy and practice, that required Dans, the borrower, to secure MRI coverage. Instead of allowing
Dans to look for his own insurance carrier or some other form of insurance policy, DBP compelled him to apply with the DBP
MRI Pool for MRI coverage. When Dan's loan was released on August 11, 1987, DBP already deducted from the proceeds
thereof the MRI premium. Four days latter, DBP made Dans fill up and sign his application for MRI, as well as his health
statement. The DBP later submitted both the application form and health statement to the DBP MRI Pool at the DBP Main
Building, Makati Metro Manila. As service fee, DBP deducted 10 percent of the premium collected by it from Dans.

In dealing with Dans, DBP was wearing two legal hats: the first as a lender, and the second as an insurance agent.

As an insurance agent, DBP made Dans go through the motion of applying for said insurance, thereby leading him and his
family to believe that they had already fulfilled all the requirements for the MRI and that the issuance of their policy was
forthcoming. Apparently, DBP had full knowledge that Dan's application was never going to be approved. The maximum age
for MRI acceptance is 60 years as clearly and specifically provided in Article 1 of the Group Mortgage Redemption Insurance
Policy signed in 1984 by all the insurance companies concerned (Exh. "1-Pool").

Under Article 1987 of the Civil Code of the Philippines, "the agent who acts as such is not personally liable to the party with
whom he contracts, unless he expressly binds himself or exceeds the limits of his authority without giving such party
sufficient notice of his powers."

The DBP is not authorized to accept applications for MRI when its clients are more than 60 years of age (Exh. "1-Pool").
Knowing all the while that Dans was ineligible for MRI coverage because of his advanced age, DBP exceeded the scope of
its authority when it accepted Dan's application for MRI by collecting the insurance premium, and deducting its agent's
commission and service fee.

The liability of an agent who exceeds the scope of his authority depends upon whether the third person is aware of the limits
of the agent's powers. There is no showing that Dans knew of the limitation on DBP's authority to solicit applications for MRI.

If the third person dealing with an agent is unaware of the limits of the authority conferred by the principal on the agent and
he (third person) has been deceived by the non-disclosure thereof by the agent, then the latter is liable for damages to him
(V Tolentino, Commentaries and Jurisprudence on the Civil Code of the Philippines, p. 422 [1992], citing Sentencia [Cuba] of
September 25, 1907). The rule that the agent is liable when he acts without authority is founded upon the supposition that
there has been some wrong or omission on his part either in misrepresenting, or in affirming, or concealing the authority
under which he assumes to act (Francisco, V., Agency 307 [1952], citing Hall v. Lauderdale, 46 N.Y. 70, 75). Inasmuch as the
non-disclosure of the limits of the agency carries with it the implication that a deception was perpetrated on the unsuspecting
client, the provisions of Articles 19, 20 and 21 of the Civil Code of the Philippines come into play.

Article 19 provides:

Every person must, in the exercise of his rights and in the performance of his duties, act with justice give
everyone his due and observe honesty and good faith.

Article 20 provides:

Every person who, contrary to law, willfully or negligently causes damage to another, shall indemnify the
latter for the same.

Article 21 provides:

Any person, who willfully causes loss or injury to another in a manner that is contrary to morals, good
customs or public policy shall compensate the latter for the damage.

The DBP's liability, however, cannot be for the entire value of the insurance policy. To assume that were it not for DBP's
concealment of the limits of its authority, Dans would have secured an MRI from another insurance company, and therefore
would have been fully insured by the time he died, is highly speculative. Considering his advanced age, there is no absolute
certainty that Dans could obtain an insurance coverage from another company. It must also be noted that Dans died almost
immediately, i.e., on the nineteenth day after applying for the MRI, and on the twenty-third day from the date of release of his
loan.

One is entitled to an adequate compensation only for such pecuniary loss suffered by him as he has duly proved (Civil Code
of the Philippines, Art. 2199). Damages, to be recoverable, must not only be capable of proof, but must be actually proved
with a reasonable degree of certainty (Refractories Corporation v. Intermediate Appellate Court, 176 SCRA 539 [1989]; Choa
Tek Hee v. Philippine Publishing Co., 34 Phil. 447 [1916]). Speculative damages are too remote to be included in an accurate
estimate of damages (Sun Life Assurance v. Rueda Hermanos, 37 Phil. 844 [1918]).

While Dans is not entitled to compensatory damages, he is entitled to moral damages. No proof of pecuniary loss is required
in the assessment of said kind of damages (Civil Code of Philippines, Art. 2216). The same may be recovered in acts
referred to in Article 2219 of the Civil Code.

The assessment of moral damages is left to the discretion of the court according to the circumstances of each case (Civil
Code of the Philippines, Art. 2216). Considering that DBP had offered to pay P30,000.00 to respondent Estate in ex
gratia settlement of its claim and that DBP's non-disclosure of the limits of its authority amounted to a deception to its client,
an award of moral damages in the amount of P50,000.00 would be reasonable.

The award of attorney's fees is also just and equitable under the circumstances (Civil Code of the Philippines, Article 2208
[11]).

WHEREFORE, the decision of the Court of Appeals in CA G.R.-CV


No. 26434 is MODIFIED and petitioner DBP is ORDERED: (1) to REIMBURSE respondent Estate of Juan B. Dans the
amount of P1,476.00 with legal interest from the date of the filing of the complaint until fully paid; and (2) to PAY said Estate
the amount of Fifty Thousand Pesos (P50,000.00) as moral damages and the amount of Ten Thousand Pesos (P10,000.00)
as attorney's fees. With costs against petitioner.

SO ORDERED.

Cruz, Davide, Jr., Bellosillo and Kapunan, JJ., concur.


G.R. No. L-31845 April 30, 1979

GREAT PACIFIC LIFE ASSURANCE COMPANY, petitioner,


vs.
HONORABLE COURT OF APPEALS, respondents.

G.R. No. L-31878 April 30, 1979

LAPULAPU D. MONDRAGON, petitioner,


vs.
HON. COURT OF APPEALS and NGO HING, respondents.

Siguion Reyna, Montecillo & Ongsiako and Sycip, Salazar, Luna & Manalo for petitioner Company.

Voltaire Garcia for petitioner Mondragon.

Pelaez, Pelaez & Pelaez for respondent Ngo Hing.


DE CASTRO, J.:

The two above-entitled cases were ordered consolidated by the Resolution of this Court dated April 29, 1970, (Rollo, No. L-
31878, p. 58), because the petitioners in both cases seek similar relief, through these petitions for certiorari by way of appeal,
from the amended decision of respondent Court of Appeals which affirmed in toto the decision of the Court of First Instance
of Cebu, ordering "the defendants (herein petitioners Great Pacific Ligfe Assurance Company and Mondragon) jointly and
severally to pay plaintiff (herein private respondent Ngo Hing) the amount of P50,000.00 with interest at 6% from the date of
the filing of the complaint, and the sum of P1,077.75, without interest.

It appears that on March 14, 1957, private respondent Ngo Hing filed an application with the Great Pacific Life Assurance
Company (hereinafter referred to as Pacific Life) for a twenty-year endownment policy in the amount of P50,000.00 on the
life of his one-year old daughter Helen Go. Said respondent supplied the essential data which petitioner Lapulapu D.
Mondragon, Branch Manager of the Pacific Life in Cebu City wrote on the corresponding form in his own handwriting (Exhibit
I-M). Mondragon finally type-wrote the data on the application form which was signed by private respondent Ngo Hing. The
latter paid the annual premuim the sum of P1,077.75 going over to the Company, but he reatined the amount of P1,317.00 as
his commission for being a duly authorized agebt of Pacific Life. Upon the payment of the insurance premuim, the binding
deposit receipt (Exhibit E) was issued to private respondent Ngo Hing. Likewise, petitioner Mondragon handwrote at the
bottom of the back page of the application form his strong recommendation for the approval of the insurance application.
Then on April 30, 1957, Mondragon received a letter from Pacific Life disapproving the insurance application (Exhibit 3-M).
The letter stated that the said life insurance application for 20-year endowment plan is not available for minors below seven
years old, but Pacific Life can consider the same under the Juvenile Triple Action Plan, and advised that if the offer is
acceptable, the Juvenile Non-Medical Declaration be sent to the company.

The non-acceptance of the insurance plan by Pacific Life was allegedly not communicated by petitioner Mondragon to private
respondent Ngo Hing. Instead, on May 6, 1957, Mondragon wrote back Pacific Life again strongly recommending the
approval of the 20-year endowment insurance plan to children, pointing out that since 1954 the customers, especially the
Chinese, were asking for such coverage (Exhibit 4-M).

It was when things were in such state that on May 28, 1957 Helen Go died of influenza with complication of
bronchopneumonia. Thereupon, private respondent sought the payment of the proceeds of the insurance, but having failed in
his effort, he filed the action for the recovery of the same before the Court of First Instance of Cebu, which rendered the
adverse decision as earlier refered to against both petitioners.

The decisive issues in these cases are: (1) whether the binding deposit receipt (Exhibit E) constituted a temporary contract of
the life insurance in question; and (2) whether private respondent Ngo Hing concealed the state of health and physical
condition of Helen Go, which rendered void the aforesaid Exhibit E.

1. At the back of Exhibit E are condition precedents required before a deposit is considered a BINDING RECEIPT. These
conditions state that:

A. If the Company or its agent, shan have received the premium deposit ... and the insurance application,
ON or PRIOR to the date of medical examination ... said insurance shan be in force and in effect from the
date of such medical examination, for such period as is covered by the deposit ...,PROVIDED the company
shall be satisfied that on said date the applicant was insurable on standard rates under its rule for the
amount of insurance and the kind of policy requested in the application.

D. If the Company does not accept the application on standard rate for the amount of insurance and/or the
kind of policy requested in the application but issue, or offers to issue a policy for a different plan and/or
amount ..., the insurance shall not be in force and in effect until the applicant shall have accepted the
policy as issued or offered by the Company and shall have paid the full premium thereof. If the applicant
does not accept the policy, the deposit shall be refunded.

E. If the applicant shall not have been insurable under Condition A above, and the Company declines to
approve the application the insurance applied for shall not have been in force at any time and the sum paid
be returned to the applicant upon the surrender of this receipt. (Emphasis Ours).

The aforequoted provisions printed on Exhibit E show that the binding deposit receipt is intended to be merely a provisional
or temporary insurance contract and only upon compliance of the following conditions: (1) that the company shall be satisfied
that the applicant was insurable on standard rates; (2) that if the company does not accept the application and offers to issue
a policy for a different plan, the insurance contract shall not be binding until the applicant accepts the policy offered;
otherwise, the deposit shall be reftmded; and (3) that if the applicant is not ble according to the standard rates, and the
company disapproves the application, the insurance applied for shall not be in force at any time, and the premium paid shall
be returned to the applicant.

Clearly implied from the aforesaid conditions is that the binding deposit receipt in question is merely an acknowledgment, on
behalf of the company, that the latter's branch office had received from the applicant the insurance premium and had
accepted the application subject for processing by the insurance company; and that the latter will either approve or reject the
same on the basis of whether or not the applicant is "insurable on standard rates." Since petitioner Pacific Life disapproved
the insurance application of respondent Ngo Hing, the binding deposit receipt in question had never become in force at any
time.

Upon this premise, the binding deposit receipt (Exhibit E) is, manifestly, merely conditional and does not insure outright. As
held by this Court, where an agreement is made between the applicant and the agent, no liability shall attach until the
principal approves the risk and a receipt is given by the agent. The acceptance is merely conditional and is subordinated to
the act of the company in approving or rejecting the application. Thus, in life insurance, a "binding slip" or "binding receipt"
does not insure by itself (De Lim vs. Sun Life Assurance Company of Canada, 41 Phil. 264).

It bears repeating that through the intra-company communication of April 30, 1957 (Exhibit 3-M), Pacific Life disapproved the
insurance application in question on the ground that it is not offering the twenty-year endowment insurance policy to children
less than seven years of age. What it offered instead is another plan known as the Juvenile Triple Action, which private
respondent failed to accept. In the absence of a meeting of the minds between petitioner Pacific Life and private respondent
Ngo Hing over the 20-year endowment life insurance in the amount of P50,000.00 in favor of the latter's one-year old
daughter, and with the non-compliance of the abovequoted conditions stated in the disputed binding deposit receipt, there
could have been no insurance contract duly perfected between thenl Accordingly, the deposit paid by private respondent
shall have to be refunded by Pacific Life.

As held in De Lim vs. Sun Life Assurance Company of Canada, supra, "a contract of insurance, like other contracts, must be
assented to by both parties either in person or by their agents ... The contract, to be binding from the date of the application,
must have been a completed contract, one that leaves nothing to be dione, nothing to be completed, nothing to be passed
upon, or determined, before it shall take effect. There can be no contract of insurance unless the minds of the parties have
met in agreement."

We are not impressed with private respondent's contention that failure of petitioner Mondragon to communicate to him the
rejection of the insurance application would not have any adverse effect on the allegedly perfected temporary contract
(Respondent's Brief, pp. 13-14). In this first place, there was no contract perfected between the parties who had no meeting
of their minds. Private respondet, being an authorized insurance agent of Pacific Life at Cebu branch office, is indubitably
aware that said company does not offer the life insurance applied for. When he filed the insurance application in dispute,
private respondent was, therefore, only taking the chance that Pacific Life will approve the recommendation of Mondragon for
the acceptance and approval of the application in question along with his proposal that the insurance company starts to offer
the 20-year endowment insurance plan for children less than seven years. Nonetheless, the record discloses that Pacific Life
had rejected the proposal and recommendation. Secondly, having an insurable interest on the life of his one-year old
daughter, aside from being an insurance agent and an offense associate of petitioner Mondragon, private respondent Ngo
Hing must have known and followed the progress on the processing of such application and could not pretend ignorance of
the Company's rejection of the 20-year endowment life insurance application.

At this juncture, We find it fit to quote with approval, the very apt observation of then Appellate Associate Justice Ruperto G.
Martin who later came up to this Court, from his dissenting opinion to the amended decision of the respondent court which
completely reversed the original decision, the following:

Of course, there is the insinuation that neither the memorandum of rejection (Exhibit 3-M) nor the reply
thereto of appellant Mondragon reiterating the desire for applicant's father to have the application
considered as one for a 20-year endowment plan was ever duly communicated to Ngo; Hing, father of the
minor applicant. I am not quite conninced that this was so. Ngo Hing, as father of the applicant herself, was
precisely the "underwriter who wrote this case" (Exhibit H-1). The unchallenged statement of appellant
Mondragon in his letter of May 6, 1957) (Exhibit 4-M), specifically admits that said Ngo Hing was "our
associate" and that it was the latter who "insisted that the plan be placed on the 20-year endowment plan."
Under these circumstances, it is inconceivable that the progress in the processing of the application was not
brought home to his knowledge. He must have been duly apprised of the rejection of the application for a
20-year endowment plan otherwise Mondragon would not have asserted that it was Ngo Hing himself who
insisted on the application as originally filed, thereby implictly declining the offer to consider the application
under the Juvenile Triple Action Plan. Besides, the associate of Mondragon that he was, Ngo Hing should
only be presumed to know what kind of policies are available in the company for minors below 7 years old.
What he and Mondragon were apparently trying to do in the premises was merely to prod the company into
going into the business of issuing endowment policies for minors just as other insurance companies
allegedly do. Until such a definite policy is however, adopted by the company, it can hardly be said that it
could have been bound at all under the binding slip for a plan of insurance that it could not have, by then
issued at all. (Amended Decision, Rollo, pp- 52-53).

2. Relative to the second issue of alleged concealment. this Court is of the firm belief that private respondent had deliberately
concealed the state of health and piysical condition of his daughter Helen Go. Wher private regpondeit supplied the required
essential data for the insurance application form, he was fully aware that his one-year old daughter is typically a mongoloid
child. Such a congenital physical defect could never be ensconced nor disguished. Nonetheless, private respondent, in
apparent bad faith, withheld the fact materal to the risk to be assumed by the insurance compary. As an insurance agent of
Pacific Life, he ought to know, as he surely must have known. his duty and responsibility to such a material fact. Had he
diamond said significant fact in the insurance application fom Pacific Life would have verified the same and would have had
no choice but to disapprove the application outright.

The contract of insurance is one of perfect good faith uberrima fides meaning good faith, absolute and perfect candor or
openness and honesty; the absence of any concealment or demotion, however slight [Black's Law Dictionary, 2nd Edition],
not for the alone but equally so for the insurer (Field man's Insurance Co., Inc. vs. Vda de Songco, 25 SCRA 70).
Concealment is a neglect to communicate that which a partY knows aDd Ought to communicate (Section 25, Act No. 2427).
Whether intentional or unintentional the concealment entitles the insurer to rescind the contract of insurance (Section 26, Id.:
Yu Pang Cheng vs. Court of Appeals, et al, 105 Phil 930; Satumino vs. Philippine American Life Insurance Company, 7
SCRA 316). Private respondent appears guilty thereof.

We are thus constrained to hold that no insurance contract was perfected between the parties with the noncompliance of the
conditions provided in the binding receipt, and concealment, as legally defined, having been comraitted by herein private
respondent.

WHEREFORE, the decision appealed from is hereby set aside, and in lieu thereof, one is hereby entered absolving
petitioners Lapulapu D. Mondragon and Great Pacific Life Assurance Company from their civil liabilities as found by
respondent Court and ordering the aforesaid insurance company to reimburse the amount of P1,077.75, without interest, to
private respondent, Ngo Hing. Costs against private respondent.

SO ORDERED.

Teehankee (Chairman), Makasiar, Guerrero and Melencio-Herrera, JJ., concur.

Fernandez, J., took no part.

III. Insurable interest


G.R. No. 124520 August 18, 1997

Spouses NILO CHA and STELLA UY CHA, and UNITED INSURANCE CO., INC., petitioners,
vs.
COURT OF APPEALS and CKS DEVELOPMENT CORPORATION, respondents.

PADILLA, J.:

This petition for review on certiorari under Rule 45 of the Rules of Court seeks to set aside a decision of respondent Court of
Appeals.

The undisputed facts of the case are as follows:


1. Petitioner-spouses Nilo Cha and Stella Uy-Cha, as lessees, entered into a lease contract with private respondent CKS
Development Corporation (hereinafter CKS), as lessor, on 5 October 1988.

2. One of the stipulations of the one (1) year lease contract states:

18. . . . The LESSEE shall not insure against fire the chattels, merchandise, textiles, goods and effects placed at any
stall or store or space in the leased premises without first obtaining the written consent and approval of the
LESSOR. If the LESSEE obtain(s) the insurance thereof without the consent of the LESSOR then the policy is
deemed assigned and transferred to the LESSOR for its own benefit; . . . 1

3. Notwithstanding the above stipulation in the lease contract, the Cha spouses insured against loss by fire the merchandise
inside the leased premises for Five Hundred Thousand (P500,000.00) with the United Insurance Co., Inc. (hereinafter
United) without the written consent of private respondent CKS.

4. On the day that the lease contract was to expire, fire broke out inside the leased premises.

5. When CKS learned of the insurance earlier procured by the Cha spouses (without its consent), it wrote the insurer (United)
a demand letter asking that the proceeds of the insurance contract (between the Cha spouses and United) be paid directly to
CKS, based on its lease contract with the Cha spouses.

6. United refused to pay CKS. Hence, the latter filed a complaint against the Cha spouses and United.

7. On 2 June 1992, the Regional Trial Court, Branch 6, Manila, rendered a decision * ordering therein defendant United to
pay CKS the amount of P335,063.11 and defendant Cha spouses to pay P50,000.00 as exemplary damages, P20,000.00 as
attorney's fees and costs of suit.

8. On appeal, respondent Court of Appeals in CA GR CV No. 39328 rendered a decision ** dated 11 January 1996, affirming
the trial court decision, deleting however the awards for exemplary damages and attorney's fees. A motion for
reconsideration by United was denied on 29 March 1996.

In the present petition, the following errors are assigned by petitioners to the Court of Appeals:

THE HONORABLE COURT OF APPEALS ERRED IN FAILING TO DECLARE THAT THE STIPULATION IN THE
CONTRACT OF LEASE TRANSFERRING THE PROCEEDS OF THE INSURANCE TO RESPONDENT IS NULL
AND VOID FOR BEING CONTRARY TO LAW, MORALS AND PUBLIC POLICY
II
THE HONORABLE COURT OF APPEALS ERRED IN FAILING TO DECLARE THE CONTRACT OF LEASE
ENTERED INTO AS A CONTRACT OF ADHESION AND THEREFORE THE QUESTIONABLE PROVISION
THEREIN TRANSFERRING THE PROCEEDS OF THE INSURANCE TO RESPONDENT MUST BE RULED OUT
IN FAVOR OF PETITIONER
III
THE HONORABLE COURT OF APPEALS ERRED IN AWARDING PROCEEDS OF AN INSURANCE POLICY TO
APPELLEE WHICH IS NOT PRIVY TO THE SAID POLICY IN CONTRAVENTION OF THE INSURANCE LAW
IV
THE HONORABLE COURT OF APPEALS ERRED IN AWARDING PROCEEDS OF AN INSURANCE POLICY ON
THE BASIS OF A STIPULATION WHICH IS VOID FOR BEING WITHOUT CONSIDERATION AND FOR BEING
TOTALLY DEPENDENT ON THE WILL OF THE RESPONDENT CORPORATION. 2

The core issue to be resolved in this case is whether or not the aforequoted paragraph 18 of the lease contract entered into
between CKS and the Cha spouses is valid insofar as it provides that any fire insurance policy obtained by the lessee (Cha
spouses) over their merchandise inside the leased premises is deemed assigned or transferred to the lessor (CKS) if said
policy is obtained without the prior written consent of the latter.

It is, of course, basic in the law on contracts that the stipulations contained in a contract cannot be contrary to law, morals,
good customs, public order or public policy. 3

Sec. 18 of the Insurance Code provides:


Sec. 18. No contract or policy of insurance on property shall be enforceable except for the benefit of some person
having an insurable interest in the property insured.

A non-life insurance policy such as the fire insurance policy taken by petitioner-spouses over their merchandise is primarily a
contract of indemnity. Insurable interest in the property insured must exist at the time the insurance takes effect and at the
time the loss occurs. 4 The basis of such requirement of insurable interest in property insured is based on sound public
policy: to prevent a person from taking out an insurance policy on property upon which he has no insurable interest
and collecting the proceeds of said policy in case of loss of the property. In such a case, the contract of insurance is a
mere wager which is void under Section 25 of the Insurance Code, which provides:

Sec. 25. Every stipulation in a policy of Insurance for the payment of loss, whether the person insured has or has not
any interest in the property insured, or that the policy shall be received as proof of such interest, and every policy
executed by way of gaming or wagering, is void.

In the present case, it cannot be denied that CKS has no insurable interest in the goods and merchandise inside the leased
premises under the provisions of Section 17 of the Insurance Code which provide:

Sec. 17. The measure of an insurable interest in property is the extent to which the insured might be damnified by
loss of injury thereof.

Therefore, respondent CKS cannot, under the Insurance Code a special law be validly a beneficiary of the fire
insurance policy taken by the petitioner-spouses over their merchandise. This insurable interest over said merchandise
remains with the insured, the Cha spouses. The automatic assignment of the policy to CKS under the provision of the lease
contract previously quoted is void for being contrary to law and/or public policy. The proceeds of the fire insurance policy thus
rightfully belong to the spouses Nilo Cha and Stella Uy-Cha (herein co-petitioners). The insurer (United) cannot be compelled
to pay the proceeds of the fire insurance policy to a person (CKS) who has no insurable interest in the property insured.

The liability of the Cha spouses to CKS for violating their lease contract in that the Cha spouses obtained a fire insurance
policy over their own merchandise, without the consent of CKS, is a separate and distinct issue which we do not resolve in
this case.

WHEREFORE, the decision of the Court of Appeals in CA-G.R. CV No. 39328 is SET ASIDE and a new decision is hereby
entered, awarding the proceeds of the fire insurance policy to petitioners Nilo Cha and Stella Uy-Cha.

SO ORDERED.

Bellosillo, Vitug, Kapunan and Hermosisima, Jr., JJ., concur.

G.R. No. 128833 April 20, 1998

RIZAL COMMERCIAL BANKING CORPORATION, UY CHUN BING AND ELI D. LAO, petitioners,
vs.
COURT OF APPEALS and GOYU & SONS, INC., respondents.

G.R. No. 128834 April 20, 1998

RIZAL COMMERCIAL BANKING CORPORATION, petitioners,


vs.
COURT OF APPEALS, ALFREDO C. SEBASTIAN, GOYU & SONS, INC., GO SONG HIAP, SPOUSES GO TENG KOK
and BETTY CHIU SUK YING alias BETTY GO, respondents.

G.R. No. 128866 April 20, 1998


MALAYAN INSURANCE INC., petitioners,
vs.
GOYU & SONS, INC. respondent.

MELO, J.:

The issue relevant to the herein three consolidated petitions revolve around the fire loss claims of respondent Goyu & Sons,
Inc. (GOYU) with petitioner Malayan Insurance Company, Inc. (MICO) in connection with the mortgage contracts entered into
by and between Rizal Commercial Banking Corporation (RCBC) and GOYU.

The Court of Appeals ordered MICO to pay GOYU its claims in the total amount of P74,040,518.58, plus 37% interest per
annum commending July 27, 1992. RCBC was ordered to pay actual and compensatory damages in the amount of
P5,000,000.00. MICO and RCBC were held solidarily liable to pay GOYU P1,500,000.00 as exemplary damages and
P1,500,000.00 for attorney's fees. GOYU's obligation to RCBC was fixed at P68,785,069.04 as of April 1992, without any
interest, surcharges, and penalties. RCBC and MICO appealed separately but, in view of the common facts and issues
involved, their individual petitions were consolidated.

The undisputed facts may be summarized as follows:

GOYU applied for credit facilities and accommodations with RCBC at its Binondo Branch. After due evaluation, RCBC
Binondo Branch, through its key officers, petitioners Uy Chun Bing and Eli D. Lao, recommended GOYU's application for
approval by RCBC's executive committee. A credit facility in the amount of P30 million was initially granted. Upon GOYU's
application and Uy's and Lao's recommendation, RCBC's executive committee increased GOYU's credit facility to P50
million, then to P90 million, and finally to P117 million.

As security for its credit facilities with RCBC, GOYU executed two real estate mortgages and two chattel mortgages in favor
of RCBC, which were registered with the Registry of Deeds at Valenzuela, Metro Manila. Under each of these four mortgage
contracts, GOYU committed itself to insure the mortgaged property with an insurance company approved by RCBC, and
subsequently, to endorse and deliver the insurance polices to RCBC.

GOYU obtained in its name a total of ten insurance policies from MICO. In February 1992, Alchester Insurance Agency, Inc.,
the insurance agent where GOYU obtained the Malayan insurance policies, issued nine endorsements in favor of RCBC
seemingly upon instructions of GOYU (Exhibits "1-Malayan" to "9-Malayan").

On April 27, 1992, one of GOYU's factory buildings in Valenzuela was gutted by fire. Consequently, GOYU submitted its
claim for indemnity on account of the loss insured against. MICO denied the claim on the ground that the insurance policies
were either attached pursuant to writs of attachments/garnishments issued by various courts or that the insurance proceeds
were also claimed by other creditors of GOYU alleging better rights to the proceeds than the insured. GOYU filed a complaint
for specific performance and damages which was docketed at the Regional Trial Court of the National Capital Judicial Region
(Manila, Branch 3) as Civil Case No. 93-65442, now subject of the present G.R. No. 128833 and 128866.

RCBC, one of GOYU's creditors, also filed with MICO its formal claim over the proceeds of the insurance policies, but said
claims were also denied for the same reasons that MICO denied GOYU's claims.

In an interlocutory order dated October 12, 1993 (Record, pp. 311-312), the Regional Trial Court of Manila (Branch 3),
confirmed that GOYU's other creditors, namely, Urban Bank, Alfredo Sebastian, and Philippine Trust Company obtained their
respective writs of attachments from various courts, covering an aggregate amount of P14,938,080.23, and ordered that the
proceeds of the ten insurance policies be deposited with the said court minus the aforementioned P14,938,080.23.
Accordingly, on January 7, 1994, MICO deposited the amount of P50,505,594.60 with Branch 3 of the Manila RTC.

In the meantime, another notice of garnishment was handed down by another Manila RTC sala (Branch 28) for the amount of
P8,696,838.75 (Exhibit "22-Malayan").

After trial, Branch 3 of the Manila RTC rendered judgment in favor of GOYU, disposing:

WHEREFORE, judgment is hereby rendered in favor of the plaintiff and against the defendant, Malayan
Insurance Company, Inc. and Rizal Commercial Banking Corporation, ordering the latter as follows:

1. For defendant Malayan Insurance Co., Inc.:


a. To pay the plaintiff its fire loss claims in the total amount of
P74,040,518.58 less the amount of P50,000,000.00 which is deposited
with this Court;

b. To pay the plaintiff damages by was of interest for the duration of the
delay since July 27, 1992 (ninety days after defendant insurer's receipt of
the required proof of loss and notice of loss) at the rate of twice the
ceiling prescribed by the Monetary Board, on the following amounts:

1) P50,000,000.00 from July 27, 1992 up to the time


said amount was deposited with this Court on January
7, 1994;

2) P24,040,518.58 from July 27, 1992 up to the time


when the writs of attachments were received by
defendant Malayan;

2. For defendant Rizal Commercial Banking Corporation:

a. To pay the plaintiff actual and compensatory damages in the amount of


P2,000,000.00;

3. For both defendants Malayan and RCBC:

a. To pay the plaintiff, jointly and severally, the following amounts:

1) P1,000,000.00 as exemplary damages;

2) P1,000,000.00 as, and for, attorney's fees;

3) Costs of suit.

and on the Counterclaim of defendant RCBC, ordering the plaintiff to pay its loan
obligations with defendant RCBC in the amount of P68,785,069.04, as of April 27, 1992,
with interest thereon at the rate stipulated in the respective promissory notes (without
surcharges and penalties) per computation, pp. 14-A, 14-B & 14-C.

FURTHER, the Clerk of Court of the Regional Trial Court of Manila is hereby ordered to release immediately
to the plaintiff the amount of P50,000,000.00 deposited with the Court by defendant Malayan, together with
all the interest earned thereon.

(Record, pp. 478-479.)

From this judgment, all parties interposed their respective appeals. GOYU was unsatisfied with the amount awarded in its
favor. MICO and RCBC disputed the trial court's findings of liability on their part. The Court of Appeals party granted GOYU's
appeal, but sustained the findings of the trial court with respect to MICO and RCBC's liabilities, thusly:

WHEREFORE, the decision of the lower court dated June 29, 1994 is hereby modified as follows:

1. FOR DEFENDANT MALAYAN INSURANCE CO., INC:

a) To pay the plaintiff its fire loss claim in the total amount of
P74,040,518.58 less the amount of P50,505,594.60 (per O.R. No.
3649285) plus deposited in court and damages by way of interest
commencing July 27, 1992 until the time Goyu receives the said amount
at the rate of thirty-seven (37%) percent per annum which is twice the
ceiling prescribed by the Monetary Board.

2. FOR DEFENDANT RIZAL COMMERCIAL BANKING CORPORATION;


a) To pay the plaintiff actual and compensatory damages in the amount of
P5,000,000.00.

3. FOR DEFENDANTS MALAYAN INSURANCE CO., INC., RIZAL COMMERCIAL


BANKING CORPORATION, UY CHUN BING AND ELI D. LAO:

a) To pay the plaintiff jointly and severally the following amounts:

1. P1,500,000.00 as exemplary damages;

2. P1,500,000.00 as and for attorney's fees.

4. And on RCBC's Counterclaim, ordering the plaintiff Goyu & Sons, Inc. to pay its loan
obligation with RCBC in the amount of P68,785,069.04 as of April 27, 1992 without any
interest, surcharges and penalties.

The Clerk of the Court of the Regional Trial Court of Manila is hereby ordered to immediately release to
Goyu & Sons, Inc. the amount of P50,505,594.60 (per O.R. No. 3649285) deposited with it by Malayan
Insurance Co., Inc., together with all the interests thereon.

(Rollo, p. 200.)

RCBC and MICO are now before us in G.R. No. 128833 and 128866, respectively, seeking review and consequent reversal
of the above dispositions of the Court of Appeals.

In G.R. No. 128834, RCBC likewise appeals from the decision in C.A. G.R. No. CV-48376, which case, by virtue of the Court
of Appeals' resolution dated August 7, 1996, was consolidated with C.A. G.R. No. CV-46162 (subject of herein G.R. No.
128833). At issue in said petition is RCBC's right to intervene in the action between Alfredo C. Sebastian (the creditor) and
GOYU (the debtor), where the subject insurance policies were attached in favor of Sebastian.

After a careful reviews of the material facts as found by the two courts below in relation to the pertinent and applicable laws,
we find merit in the submission of RCBC and MICO.

The several causes of action pursued below by GOYU gave rise to several related issues which are now submitted in the
petitions before us. This Court, however, discerns one primary and central issue, and this is, whether or not RCBC, as
mortgagee, has any right over the insurance policies taken by GOYU, the mortgagor, in case of the occurrence of loss.

As earlier mentioned, accordant with the credit facilities extended by RCBC to GOYU, the latter executed several mortgage
contracts in favor of RCBC. It was expressly stipulated in these mortgage contracts that GOYU shall insure the mortgaged
property with any of the insurance companies acceptable to RCBC. GOYU indeed insured the mortgaged property with
MICO, an insurance company acceptable to RCBC. Bases on their stipulations in the mortgage contracts, GOYU was
supposed to endorse these insurance policies in favor of, and deliver them, to RCBC. Alchester Insurance Agency, Inc.,
MICO's underwriter from whom GOYU obtained the subject insurance policies, prepared the nine endorsements (see Exh.
"1-Malayan" to "9-Malayan"; also Exh. "51-RCBC" to "59-RCBC"), copies of which were delivered to GOYU, RCBC, and
MICO. However, because these endorsements do not bear the signature of any officer of GOYU, the trial court, as well as
the Court of Appeals, concluded that the endorsements are defective.

We do not quite agree.

It is settled that a mortgagor and a mortgagee have separated and distinct insurable interests in the same mortgaged
property, such that each one of them may insure the same property for his own sole benefit. There is no question that GOYU
could insure the mortgaged property for its own exclusive benefit. In the present case, although it appears that GOYU
obtained the subject insurance policies naming itself as the sole payee, the intentions of the parties as shown by their
contemporaneous acts, must be given due consideration in order to better serve the interest of justice and equity.

It is to be noted that nine endorsement documents were prepared by Alchester in favor of RCBC. The Court is in a quandary
how Alchester could arrive at the idea of endorsing any specific insurance policy in favor of any particular beneficiary or
payee other than the insured had not such named payee or beneficiary been specifically disclosed by the insured itself. It is
also significant that GOYU voluntarily and purposely took the insurance policies from MICO, a sister company of RCBC, and
not just from any other insurance company. Alchester would not have found out that the subject pieces of property were
mortgaged to RCBC had not such information been voluntarily disclosed by GOYU itself. Had it not been for GOYU,
Alchester would not have known of GOYU's intention of obtaining insurance coverage in compliance with its undertaking in
the mortgage contracts with RCBC, and verily, Alchester would not have endorsed the policies to RCBC had it not been so
directed by GOYU.

On equitable principles, particularly on the ground of estoppel, the Court is constrained to rule in favor of mortgagor RCBC.
The basis and purpose of the doctrine was explained in Philippine National Bank vs. Court of Appeals (94 SCRA 357 [1979]),
to wit:

The doctrine of estoppel is based upon the grounds of public, policy, fair dealing, good faith and justice, and
its purpose is to forbid one to speak against his own act, representations, or commitments to the injury of
one to whom they were directed and who reasonably relied thereon. The doctrine of estoppel springs from
equitable principles and the equities in the case. It is designed to aid the law in the administration of justice
where without its aid injustice might result. It has been applied by this Court wherever and whenever special
circumstances of a case so demand.

(p. 368.)

Evelyn Lozada of Alchester testified that upon instructions of Mr. Go, through a certain Mr. Yam, she prepared in
quadruplicate on February 11, 1992 the nine endorsement documents for GOYU's nine insurance policies in favor of RCBC.
The original copies of each of these nine endorsement documents were sent to GOYU, and the others were sent to RCBC
and MICO, while the fourth copies were detained for Alchester's file (tsn, February 23, pp. 7-8). GOYU has not denied having
received from Alchester the originals of these documents.

RCBC, in good faith, relied upon the endorsement documents sent to it as this was only pursuant to the stipulation in the
mortgage contracts. We find such reliance to be justified under the circumstances of the case. GOYU failed to seasonably
repudiate the authority of the person or persons who prepared such endorsements. Over and above this, GOYU continued,
in the meantime, to enjoy the benefits of the credit facilities extended to it by RCBC. After the occurrence of the loss insure
against, it was too late for GOYU to disown the endorsements for any imagined or contrived lack of authority of Alchester to
prepare and issue said endorsements. If there had not been actually an implied ratification of said endorsements by virtue of
GOYU's inaction in this case, GOYU is at the very least estopped from assailing their operative effects. To permit GOYU to
capitalize on its non-confirmation of these endorsements while it continued to enjoy the benefits of the credit facilities of
RCBC which believed in good faith that there was due endorsement pursuant to their mortgage contracts, is to countenance
grave contravention of public policy, fair dealing, good faith, and justice. Such an unjust situation, the Court cannot sanction.
Under the peculiar circumstances obtaining in this case, the Court is bound to recognize RCBC's right to the proceeds of the
insurance polices if not for the actual endorsement of the policies, at least on the basis of the equitable principle of estoppel.

GOYU cannot seek relief under Section 53 of the Insurance Code which provides that the proceeds of insurance shall
exclusively apply to the interest of the person in whose name or for whose benefit it is made. The peculiarity of the
circumstances obtaining in the instant case presents a justification to take exception to the strict application of said provision,
it having been sufficiently established that it was the intention of the parties to designate RCBC as the party for whose benefit
the insurance policies were taken out. Consider thus the following:

1. It is undisputed that the insured pieces of property were the subject of mortgage contracts entered into between RCBC
and GOYU in consideration of and for securing GOYU's credit facilities from RCBC. The mortgage contracts contained
common provisions whereby GOYU, as mortgagor, undertook to have the mortgaged property properly covered against any
loss by an insurance company acceptable to RCBC.

2. GOYU voluntarily procured insurance policies to cover the mortgaged property from MICO, no less than a sister company
of RCBC and definitely an acceptable insurance company to RCBC.

3. Endorsement documents were prepared by MICO's underwriter, Alchester Insurance Agency, Inc., and copies thereof were
sent to GOYU, MICO, and RCBC. GOYU did not assail, until of late, the validity of said endorsements.

4. GOYU continued until the occurrence of the fire, to enjoy the benefits of the credit facilities extended by RCBC which was
conditioned upon the endorsement of the insurance policies to be taken by GOYU to cover the mortgaged properties.

This Court can not over stress the fact that upon receiving its copies of the endorsement documents prepared by Alchester,
GOYU, despite the absence of its written conformity thereto, obviously considered said endorsement to be sufficient
compliance with its obligation under the mortgage contracts since RCBC accordingly continued to extend the benefits of its
credits facilities and GOYU continued to benefit therefrom. Just as plain too is the intention of the parties to constitute RCBC
as the beneficiary of the various insurance policies obtained by GOYU. The intention of the parties will have to be given full
force and effect particular case. The insurance proceeds may, therefore, be exclusively applied to RCBC, which under the
factual circumstances of the case, is truly the person or entity for whose benefit the polices were clearly intended.

Moreover, the law's evident intention to protect the interests of the mortgage upon the mortgaged property is expressed in
Article 2127 of the Civil Code which states:

Art. 2127. The mortgage extends to the natural accessions, to the improvements, growing fruits, and the
rents or income not yet received when the obligation becomes due, and to the amount of the indemnity
granted or owing to the proprietor from the insurers of the property mortgaged, or in virtue of expropriation
for public use, with the declarations, amplifications and limitations established by law, whether the estate
remains in the possession of the mortgagor, or it passes into the hands of a third person.

Significantly, the Court notes that out of the 10 insurance policies subject of this case, only 8 of them appear to have been
subject of the endorsements prepared and delivered by Alchester for and upon instructions of GOYU as shown below:

INSURANCE POLICY PARTICULARS ENDORSEMENT

a. Policy Number F-114-07795 None


Issue Date March 18, 1992
Expiry Date April 5, 1993
Amount P9,646,224.92

b. Policy Number ACIA/F-174-07660 Exhibit "1-Malayan"


Issue Date January 18, 1992
Expiry Date February 9, 1993
Amount P4,307,217.54

c. Policy Number ACIA/F-114-07661 Exhibit "2-Malayan"


Issue Date January 18, 1992
Expiry Date February 15, 1993
Amount P6,603,586.43

d. Policy Number ACIA/F-114-07662 Exhibit "3-Malayan"


Issue Date January 18, 1992
Expiry Date (not legible)
Amount P6,603,586.43

e. Policy Number ACIA/F-114-07663 Exhibit "4-Malayan"


Issue Date January 18, 1992
Expiry Date February 9, 1993
Amount P9,457,972.76

f. Policy Number ACIA/F-114-07623 Exhibit "7-Malayan"


Issue Date January 13, 1992
Expiry Date January 13, 1993
Amount P24,750,000.00

g. Policy Number ACIA/F-174-07223 Exhibit "6-Malayan"


Issue Date May 29, 1991
Expiry Date June 27, 1992
Amount P6,000,000.00

h. Policy Number CI/F-128-03341 None


Issue Date May 3, 1991
Expiry Date May 3, 1992
Amount P10,000,000.00

i. Policy Number F-114-07402 Exhibit "8-Malayan"


Issue Date September 16, 1991
Expiry Date October 19, 1992
Amount P32,252,125.20

j. Policy Number F-114-07525 Exhibit "9-Malayan"


Issue Date November 20, 1991
Expiry Date December 5, 1992
Amount P6,603,586.43

(pp. 456-457, Record; Folder of Exhibits for MICO.)

Policy Number F-114-07795 [(a) above] has not been endorsed. This fact was admitted by MICO's witness, Atty. Farolan
(tsn, February 16, 1994, p. 25). Likewise, the record shows no endorsement for Policy Number CI/F-128-03341 [(h) above].
Also, one of the endorsement documents, Exhibit "5-Malayan", refers to a certain insurance policy number ACIA-F-07066,
which is not among the insurance policies involved in the complaint.

The proceeds of the 8 insurance policies endorsed to RCBC aggregate to P89,974,488.36. Being excessively payable to
RCBC by reason of the endorsement by Alchester to RCBC, which we already ruled to have the force and effect of an
endorsement by GOYU itself, these 8 policies can not be attached by GOYU's other creditors up to the extent of the GOYU's
outstanding obligation in RCBC's favor. Section 53 of the Insurance Code ordains that the insurance proceeds of the
endorsed policies shall be applied exclusively to the proper interest of the person for whose benefit it was made. In this case,
to the extent of GOYU's obligation with RCBC, the interest of GOYU in the subject policies had been transferred to RCBC
effective as of the time of the endorsement. These policies may no longer be attached by the other creditors of GOYU, like
Alfredo Sebastian in the present G.R. No. 128834, which may nonetheless forthwith be dismissed for being moot and
academic in view of the results reached herein. Only the two other policies amounting to P19,646,224.92 may be validly
attached, garnished, and levied upon by GOYU's other creditors. To the extent of GOYU's outstanding obligation with RCBC,
all the rest of the other insurance policies above-listed which were endorsed to RCBC, are, therefore, to be released from
attachment, garnishment, and levy by the other creditors of GOYU.

This brings us to the next issue to be resolved, which is, the extent of GOYU's outstanding obligation with RCBC which the
proceeds of the 8 insurance policies will discharge and liquidate, or put differently, the actual amount of GOYU's liability to
RCBC.

The Court of Appeals simply echoed the declaration of the trial court finding that GOYU's total obligation to RCBC was only
P68,785,060.04 as of April 27, 1992, thus sanctioning the trial court's exclusion of Promissory Note No. 421-92 (renewal of
Promissory Note No. 908-91) and Promissory Note No. 420-92 (renewal of Promissory Note No. 952-91) on the ground that
their execution is highly questionable for not only are these dated after the fire, but also because the signatures of either
GOYU or any its representative are conspicuously absent. Accordingly, the Court of Appeals speculated thusly:

. . . Hence, this Court is inclined to conclude that said promissory notes were pre-signed by plaintiff in bank
terms, as averred by plaintiff, in contemplation of the speedy grant of future loans, for the same practice of
procedure has always been adopted in its previous dealings with the bank.

(Rollo, pp. 181-182.)

The fact that the promissory notes bear dates posterior to the fire does not necessarily mean that the documents are
spurious, for it is presumed that the ordinary course of business had been followed (Metropolitan Bank and Trust Company
vs. Quilts and All, Inc., 22 SCRA 486 [1993]). The obligor and not the holder of the negotiable instrument has the burden of
proof of showing that he no longer owes the obligee any amount (Travel-On, Inc. vs. Court of Appeals, 210 SCRA 351
[1992]).

Even casting aside the presumption of regularity of private transactions, receipt of the loan amounting to P121,966,058.67
(Exhibits 1-29, RCBC) was admitted by GOYU as indicated in the testimony of Go Song Hiap when he answered the queries
of the trial court.

ATTY. NATIVIDAD

Q: But insofar as the amount stated in Exhibits 1 to 29-RCBC, you received all the
amounts stated therein?

A: Yes, sir, I received the amount.


COURT

He is asking if he received all the amounts stated in Exhibits 1 to 29-RCBC?

WITNESS:

Yes, Your Honor, I received all the amounts.

COURT

Indicated in the Promissory Notes?

WITNESS

A. The promissory Notes they did not give to me but the amount I asked which is correct,
Your Honor.

COURT

Q Your mean to say the amounts indicated in Exhibits 1 to 29-RCBC is correct?

A Yes, Your Honor.

(tsn, Jan. 14, 1994, p. 26.)

Furthermore, aside from its judicial admission of having received all the proceeds of the 29 promissory notes as hereinabove
quotes, GOYU also offered and admitted to RCBC that is obligation be fixed at P116,301,992.60 as shown in its letter date
March 9, 1993, which pertinently reads:

We wish to inform you, therefore that we are ready and willing to pay the current past due account of this
company in the amount of P116,301,992.60 as of 21 January 1993, specified in pars. 15, p. 10, and 18, p.
13 of your affidavits of Third Party Claims in the Urban case at Makati, Metro Manila and in the Zamboanga
case at Zamboanga city, respectively, less the total of P8,851,519.71 paid from the Seaboard and Equitable
insurance companies and other legitimate deductions. We accept and confirm this amount of
P116,301,992.60 as stated as true and correct.

(Exhibit BB.)

The Court of Appeals erred in placing much significance on the fact that the excluded promissory notes are dated after the
fire. It failed to consider that said notes had for their origin transactions consummated prior to the fire. Thus, careful attention
must be paid to the fact that Promissory Notes No. 420-92 and 421-92 are mere renewalsof Promissory Notes No. 908-91
and 952-91, loans already availed of by GOYU.

The two courts below erred in failing to see that the promissory notes which they ruled should be excluded for bearing dates
which are after that of the fire, are mere renewals of previous ones. The proceeds of the loan represented by these
promissory notes were admittedly received by GOYU. There is ample factual and legal basis for giving GOYU's judicial
admission of liability in the amount of P116,301,992.60 full force and effect.

It should, however, be quickly added that whatever amount RCBC may have recovered from the other insurers of the
mortgage property will, nonetheless, have to be applied as payment against GOYU's obligation. But, contrary to the lower
courts' findings, payments effected by GOYU prior to January 21, 1993 should no longer be deducted. Such payments had
obviously been duly considered by GOYU, in its aforequoted letter date March 9, 1993, wherein it admitted that its past due
account totaled P116,301,992.60 as of January 21, 1993.

The net obligation of GOYU, after deductions, is thus reduced to P107,246,887.90 as of January 21, 1993, to wit:

Total Obligation as admitted by GOYU


as of January 21, 1993: P116,301,992.60
Broken down as follows:

Principal 1 Interest

Regular 80,535,946.32
FDU 27,548,025.17
____________
Total 108,083,971.49 8,218,021.11 2

LESS:

1) Proceeds from
Seaboard Eastern
Insurance Company 6,095,145.81

2) Proceeds from
Equitable Insurance
Company 2,756,373.00

3) Payment from
foreign department
negotiation: 203,584.89
___________

9,055,104.70 3
================
NET AMOUNT as of January 21, 1993 P107,246,887.90

The need for the payment of interest due the principal amount of the obligation, which is the cost of money to RCBC, the
primary end and the ultimate reason for RCBC's existence and being, was duly recognized by the trial court when it ruled
favorably on RCBC's counterclaim, ordering GOYU "to pay its loan obligation with RCBC in the amount of P68,785,069.04,
as of April 27, 1992, with interest thereon at the rate stipulated in the respective promissory notes (without surcharges and
penalties) per computation, pp. 14-A, 14-B 14-C" (Record, p. 479). Inexplicably, the Court of Appeals, without even laying
down the factual or legal justification for its ruling, modified the trial court's ruling and ordered GOYU "to pay the principal
amount of P68,785,069.04 without any interest, surcharges and penalties" (Rollo, p. 200).

It is to be noted in this regard that even the trial court hedgingly and with much uncertainty deleted the payment ofadditional
interest, penalties, and charges, in this manner:

Regarding defendant RCBC's commitment not to charge additional interest, penalties and surcharges, the
same does not require that it be embodied in a document or some form of writing to be binding and
enforceable. The principle is well known that generally a verbal agreement or contract is no less binding and
effective than a written one. And the existence of such a verbal agreement has been amply established by
the evidence in this case. In any event, regardless of the existence of such verbal agreement, it would still
be unjust and inequitable for defendant RCBC to charge the plaintiff with surcharges and penalties
considering the latter's pitiful situation. (Emphasis supplied).

(Record, p. 476)

The essence or rationale for the payment of interest or cost of money is separate and distinct from that of surcharges and
penalties. What may justify a court in not allowing the creditor to charge surcharges and penalties despite express stipulation
therefor in a valid agreement, may not equally justify non-payment of interest. The charging of interest for loans forms a very
essential and fundamental element of the banking business, which may truly be considered to be at the very core of its
existence or being. It is inconceivable for a bank to grant loans for which it will not charge any interest at all. We fail to find
justification for the Court of Appeal's outright deletion of the payment of interest as agreed upon in the respective promissory
notes. This constitutes gross error.

For the computation of the interest due to be paid to RCBC, the following rules of thumb laid down by this Court inEastern
Shipping Lines, Inc. vs. Court of Appeals (234 SCRA 78 [1994]), shall apply, to wit:
I. When an obligation, regardless of its source, i.e., law, contracts, quasi-contracts, delicts or quasi-delicts is breached, the
contravenor can be held liable for damages. The provisions under Title XVIII on "Damages" of the Civil Code govern in
determining the measure of recoverable damages.

II. With regard particularly to an award of interest in the concept of actual and compensatory damages, the rate of interest, as
well as the actual thereof, is imposed, as follows:

1. When the obligation is breached, and it consists in the payment of a sum of money, i.e., a loan or
forbearance of money, the interest due should be that which may have been stipulated in writing.
Furthermore, the interest due shall itself earn legal interest from the time it is judicially demanded. In the
absence of stipulation, the rate of interest shall be 12% per annum to be computed from default,i.e., from
judicial or extrajudicial demand under and subject to the provisions of Article 1169 of the Civil Code.

2. When an obligation, not constituting a loan or forbearance of money, is breached, an interest on the
amount of damages awarded may be imposed at the discretion of the court at the rate of 6% per annum. No
interest, however, shall be adjudged on unliquidated claims or damages except when or until the demand
can be established with reasonable certainty. Accordingly, where the demand is established with reasonable
certainty, the interest shall begin to run from the time the claim is made judicially or extrajudicially (Art. 1169,
Civil Code) but when such certainty cannot be so reasonably established at the time the demand is made,
the interest shall begin to run only from the date of the judgment of the court is made (at which time the
quantification of damages may be deemed to have been reasonably ascertained). The actual base for the
computation of legal interest shall, in any case, be on the amount finally adjudged.

3. When the judgment of the court awarding a sum of money becomes final and executory, the rate of legal
interest, whether the case falls under paragraph 1 or paragraph 2, above, shall be 12% per annum from
such finality until its satisfaction, this interim period being deemed to be by then an equivalent to a
forbearance of credit.

(pp. 95-97).

There being written stipulations as to the rate of interest owing on each specific promissory note as summarized and
tabulated by the trial court in its decision (pp. 470 and 471, Record) such agreed interest rates must be followed. This is very
clear from paragraph II, sub-paragraph 1 quoted above.

On the issue of payment of surcharges and penalties, we partly agree that GOYU's pitiful situation must be taken into
account. We do not agree, however, that payment of any amount as surcharges and penalties should altogether be deleted.
Even assuming that RCBC, through its responsible officers, herein petitioners Eli Lao and Uy Chun Bing, may have relayed
its assurance for assistance to GOYU immediately after the occurrence of the fire, we cannot accept the lower courts' finding
that RCBC had thereby ipso facto effectively waived collection of any additional interests, surcharges, and penalties from
GOYU. Assurances of assistance are one thing, but waiver of additional interests, surcharges, and penalties is another.

Surcharges and penalties agreed to be paid by the debtor in case of default partake of the nature of liquidated damages,
covered by Section 4, Chapter 3, Title XVIII of the Civil Code. Article 2227 thereof provides:

Art. 2227. Liquidated damages, whether intended as a indemnity or penalty, shall be equitably reduced if
they are iniquitous and unconscionable.

In exercising this vested power to determine what is iniquitous and unconscionable, the Court must consider the
circumstances of each case. It should be stressed that the Court will not make any sweeping ruling that surcharges and
penalties imposed by banks for non-payment of the loans extended by them are generally iniquitous and unconscionable.
What may be iniquitous and unconscionable in one case, may be totally just and equitable in another. This provision of law
will have to be applied to the established facts of any given case. Given the circumstance under which GOYU found itself
after the occurrence of the fire, the Court rules the surcharges rates ranging anywhere from 9% to 27%, plus the penalty
charges of 36%, to be definitely iniquitous and unconscionable. The Court tempers these rates to 2% and 3%, respectively.
Furthermore, in the light of GOYU's offer to pay the amount of P116,301,992.60 to RCBC as March 1993 (See: Exhibit "BB"),
which RCBC refused, we find it more in keeping with justice and equity for RCBC not to charge additional interest,
surcharges, and penalties from that time onward.

Given the factual milieu hereover, we rule that it was error to hold MICO liable in damages for denying or withholding the
proceeds of the insurance claim to GOYU.
Firstly, by virtue of the mortgage contracts as well as the endorsements of the insurance policies, RCBC has the right to
claim the insurance proceeds, in substitution of the property lost in the fire. Having assigned its rights, GOYU lost its standing
as the beneficiary of the said insurance policies.

Secondly, for an insurance company to be held liable for unreasonably delaying and withholding payment of insurance
proceeds, the delay must be wanton, oppressive, or malevolent (Zenith Insurance Corporation vs. CA. 185 SCRA 403
[1990]). It is generally agreed, however, that an insurer may in good faith and honesty entertain a difference of opinion as to
its liability. Accordingly, the statutory penalty for vexatious refusal of an insurer to pay a claim should not be inflicted unless
the evidence and circumstances show that such refusal was willful and without reasonable cause as the facts appear to a
reasonable and prudent man (Bufallo Ins. Co. vs. Bommarito [CCA 8th] 42 F [2d] 53, 70 ALR 1211; Phoenix Ins. Co. vs. Clay,
101 Ga. 331, 28 SE 853, 65 Am St. Rep 307; Kusnetsky vs. Security Ins. Co., 313 Mo. 143, 281 SW 47, 45 ALR 189). The
case at bar does not show that MICO wantonly and in bad faith delayed the release of the proceeds. The problem in the
determination of who is the actual beneficiary of the insurance policies, aggravated by the claim of various creditors who
wanted to partake of the insurance proceeds, not to mention the importance of the endorsement to RCBC, to our mind, and
as now borne out by the outcome herein, justified MICO in withholding payment to GOYU.

In adjudging RCBC liable in damages to GOYU, the Court of Appeals said that RCBC cannot avail itself of two simultaneous
remedies in enforcing the claim of an unpaid creditor, one for specific performance and the other for foreclosure. In doing so,
said the appellate court, the second action is deemed barred, RCBC having split a single cause of action (Rollo, pp. 195-
199). The Court of Appeals was too accommodating in giving due consideration to this argument of GOYU, for the
foreclosure suit is still pending appeal before the same Court of Appeals in CA G.R. CV No. 46247, the case having been
elevated by RCBC.

In finding that the foreclosure suit cannot prosper, the Fifteenth Division of the Court of Appeals pre-empted the resolution of
said foreclosure case which is not before it. This is plain reversible error if not grave abuse of discretion.

As held in Pea vs. Court of Appeals (245 SCRA 691 [1995]):

It should have been enough, nonetheless, for the appellate court to merely set aside the questioned
ordered of the trial court for having been issued by the latter with grave abuse of discretion. In likewise
enjoining permanently herein petitioner "from entering in and interfering with the use or occupation and
enjoyment of petitioner's (now private respondent) residential house and compound," the appellate court in
effect, precipitately resolved with finality the case for injunction that was yet to be heard on the merits by the
lower court. Elevated to the appellate court, it might be stressed, were mere incidents of the principal case
still pending with the trial court. In Municipality of Bian, Laguna vs. Court of Appeals, 219 SCRA 69, we
ruled that the Court of Appeals would have "no jurisdiction in a certiorari proceeding involving an incident in
a case to rule on the merits of the main case itself which was not on appeal before it.

(pp. 701-702.)

Anent the right of RCBC to intervene in Civil Case No. 1073, before the Zamboanga Regional Trial Court, since it has been
determined that RCBC has the right to the insurance proceeds, the subject matter of intervention is rendered moot and
academic. Respondent Sebastian must, however, yield to the preferential right of RCBC over the MICO insurance policies. It
is basic and fundamental that the first mortgagee has superior rights over junior mortgagees or attaching creditors (Alpha
Insurance & Surety Co. vs. Reyes, 106 SCRA 274 [1981]; Sun Life Assurance Co. of Canada vs. Gonzales Diaz, 52 Phil. 271
[1928]).

WHEREFORE, the petitions are hereby GRANTED and the decision and resolution of December 16, 1996 and April 3, 1997
in CA-G.R. CV No. 46162 are hereby REVERSED and SET ASIDE, and a new one entered:

1. Dismissing the Complaint of private respondent GOYU in Civil Case No. 93-65442 before Branch 3 of the
Manila Trial Court for lack of merit;

2. Ordering Malayan Insurance Company, Inc. to deliver to Rizal Commercial Banking Corporation the
proceeds of the insurance policies in the amount of P51,862,390.94 (per report of adjuster Toplis & Harding
(Far East), Inc., Exhibits "2" and "2-1"), less the amount of P50,505,594.60 (per O.R. No. 3649285);

3. Ordering the Clerk of Court to release the amount of P50,505,594.60 including the interests earned to
Rizal Commercial Banking Corporation;
4. Ordering Goyu & Sons, Inc. to pay its loan obligation with Rizal Commercial Banking Corporation in the
principal amount of P107,246,887.90, with interest at the respective rates stipulated in each promissory
note from January 21, 1993 until finality of this judgment, and surcharges at 2% and penalties at 3% from
January 21, 1993 to March 9, 1993, minus payments made by Malayan Insurance Company, Inc. and the
proceeds of the amount deposited with the trial court and its earned interest. The total amount due RCBC at
the time of the finality of this judgment shall earn interest at the legal rate of 12% in lieu of all other
stipulated interests and charges until fully paid.

The petition of Rizal Commercial Banking Corporation against the respondent Court in CA-GR CV 48376 is DISMISSED for
being moot and academic in view of the results herein arrived at. Respondent Sebastian's right as attaching creditor must
yield to the preferential rights of Rizal Commercial Banking Corporation over the Malayan insurance policies as first
mortgagee.

SO ORDERED.

Regalado, Puno, Mendoza and Martinez, JJ., concur.

G.R. No. 147839 June 8, 2006

GAISANO CAGAYAN, INC. Petitioner,


vs.
INSURANCE COMPANY OF NORTH AMERICA, Respondent.

DECISION

AUSTRIA-MARTINEZ, J.:

Before the Court is a petition for review on certiorari of the Decision 1 dated October 11, 2000 of the Court of Appeals (CA) in
CA-G.R. CV No. 61848 which set aside the Decision dated August 31, 1998 of the Regional Trial Court, Branch 138, Makati
(RTC) in Civil Case No. 92-322 and upheld the causes of action for damages of Insurance Company of North America
(respondent) against Gaisano Cagayan, Inc. (petitioner); and the CA Resolution dated April 11, 2001 which denied
petitioner's motion for reconsideration.

The factual background of the case is as follows:


Intercapitol Marketing Corporation (IMC) is the maker of Wrangler Blue Jeans. Levi Strauss (Phils.) Inc. (LSPI) is the local
distributor of products bearing trademarks owned by Levi Strauss & Co.. IMC and LSPI separately obtained from respondent
fire insurance policies with book debt endorsements. The insurance policies provide for coverage on "book debts in
connection with ready-made clothing materials which have been sold or delivered to various customers and dealers of the
Insured anywhere in the Philippines."2 The policies defined book debts as the "unpaid account still appearing in the Book of
Account of the Insured 45 days after the time of the loss covered under this Policy." 3 The policies also provide for the
following conditions:

1. Warranted that the Company shall not be liable for any unpaid account in respect of the merchandise sold and
delivered by the Insured which are outstanding at the date of loss for a period in excess of six (6) months from the
date of the covering invoice or actual delivery of the merchandise whichever shall first occur.

2. Warranted that the Insured shall submit to the Company within twelve (12) days after the close of every calendar
month all amount shown in their books of accounts as unpaid and thus become receivable item from their customers
and dealers. x x x4

xxxx

Petitioner is a customer and dealer of the products of IMC and LSPI. On February 25, 1991, the Gaisano Superstore
Complex in Cagayan de Oro City, owned by petitioner, was consumed by fire. Included in the items lost or destroyed in the
fire were stocks of ready-made clothing materials sold and delivered by IMC and LSPI.

On February 4, 1992, respondent filed a complaint for damages against petitioner. It alleges that IMC and LSPI filed with
respondent their claims under their respective fire insurance policies with book debt endorsements; that as of February 25,
1991, the unpaid accounts of petitioner on the sale and delivery of ready-made clothing materials with IMC
was P2,119,205.00 while with LSPI it was P535,613.00; that respondent paid the claims of IMC and LSPI and, by virtue
thereof, respondent was subrogated to their rights against petitioner; that respondent made several demands for payment
upon petitioner but these went unheeded. 5

In its Answer with Counter Claim dated July 4, 1995, petitioner contends that it could not be held liable because the property
covered by the insurance policies were destroyed due to fortuities event or force majeure; that respondent's right of
subrogation has no basis inasmuch as there was no breach of contract committed by it since the loss was due to fire which it
could not prevent or foresee; that IMC and LSPI never communicated to it that they insured their properties; that it never
consented to paying the claim of the insured. 6

At the pre-trial conference the parties failed to arrive at an amicable settlement. 7 Thus, trial on the merits ensued.

On August 31, 1998, the RTC rendered its decision dismissing respondent's complaint. 8 It held that the fire was purely
accidental; that the cause of the fire was not attributable to the negligence of the petitioner; that it has not been established
that petitioner is the debtor of IMC and LSPI; that since the sales invoices state that "it is further agreed that merely for
purpose of securing the payment of purchase price, the above-described merchandise remains the property of the vendor
until the purchase price is fully paid", IMC and LSPI retained ownership of the delivered goods and must bear the loss.

Dissatisfied, petitioner appealed to the CA.9 On October 11, 2000, the CA rendered its decision setting aside the decision of
the RTC. The dispositive portion of the decision reads:

WHEREFORE, in view of the foregoing, the appealed decision is REVERSED and SET ASIDE and a new one is entered
ordering defendant-appellee Gaisano Cagayan, Inc. to pay:

1. the amount of P2,119,205.60 representing the amount paid by the plaintiff-appellant to the insured Inter Capitol
Marketing Corporation, plus legal interest from the time of demand until fully paid;

2. the amount of P535,613.00 representing the amount paid by the plaintiff-appellant to the insured Levi Strauss
Phil., Inc., plus legal interest from the time of demand until fully paid.

With costs against the defendant-appellee.

SO ORDERED.10
The CA held that the sales invoices are proofs of sale, being detailed statements of the nature, quantity and cost of the thing
sold; that loss of the goods in the fire must be borne by petitioner since the proviso contained in the sales invoices is an
exception under Article 1504 (1) of the Civil Code, to the general rule that if the thing is lost by a fortuitous event, the risk is
borne by the owner of the thing at the time the loss under the principle of res perit domino; that petitioner's obligation to IMC
and LSPI is not the delivery of the lost goods but the payment of its unpaid account and as such the obligation to pay is not
extinguished, even if the fire is considered a fortuitous event; that by subrogation, the insurer has the right to go against
petitioner; that, being a fire insurance with book debt endorsements, what was insured was the vendor's interest as a
creditor.11

Petitioner filed a motion for reconsideration12 but it was denied by the CA in its Resolution dated April 11, 2001. 13

Hence, the present petition for review on certiorari anchored on the following Assignment of Errors:

THE COURT OF APPEALS ERRED IN HOLDING THAT THE INSURANCE IN THE INSTANT CASE WAS ONE OVER
CREDIT.

THE COURT OF APPEALS ERRED IN HOLDING THAT ALL RISK OVER THE SUBJECT GOODS IN THE INSTANT CASE
HAD TRANSFERRED TO PETITIONER UPON DELIVERY THEREOF.

THE COURT OF APPEALS ERRED IN HOLDING THAT THERE WAS AUTOMATIC SUBROGATION UNDER ART. 2207 OF
THE CIVIL CODE IN FAVOR OF RESPONDENT.14

Anent the first error, petitioner contends that the insurance in the present case cannot be deemed to be over credit since an
insurance "on credit" belies not only the nature of fire insurance but the express terms of the policies; that it was not credit
that was insured since respondent paid on the occasion of the loss of the insured goods to fire and not because of the non-
payment by petitioner of any obligation; that, even if the insurance is deemed as one over credit, there was no loss as the
accounts were not yet due since no prior demands were made by IMC and LSPI against petitioner for payment of the debt
and such demands came from respondent only after it had already paid IMC and LSPI under the fire insurance policies. 15

As to the second error, petitioner avers that despite delivery of the goods, petitioner-buyer IMC and LSPI assumed the risk of
loss when they secured fire insurance policies over the goods.

Concerning the third ground, petitioner submits that there is no subrogation in favor of respondent as no valid insurance
could be maintained thereon by IMC and LSPI since all risk had transferred to petitioner upon delivery of the goods; that
petitioner was not privy to the insurance contract or the payment between respondent and its insured nor was its consent or
approval ever secured; that this lack of privity forecloses any real interest on the part of respondent in the obligation to pay,
limiting its interest to keeping the insured goods safe from fire.

For its part, respondent counters that while ownership over the ready- made clothing materials was transferred upon delivery
to petitioner, IMC and LSPI have insurable interest over said goods as creditors who stand to suffer direct pecuniary loss
from its destruction by fire; that petitioner is liable for loss of the ready-made clothing materials since it failed to overcome the
presumption of liability under Article 126516 of the Civil Code; that the fire was caused through petitioner's negligence in failing
to provide stringent measures of caution, care and maintenance on its property because electric wires do not usually short
circuit unless there are defects in their installation or when there is lack of proper maintenance and supervision of the
property; that petitioner is guilty of gross and evident bad faith in refusing to pay respondent's valid claim and should be liable
to respondent for contracted lawyer's fees, litigation expenses and cost of suit. 17

As a general rule, in petitions for review, the jurisdiction of this Court in cases brought before it from the CA is limited to
reviewing questions of law which involves no examination of the probative value of the evidence presented by the litigants or
any of them.18 The Supreme Court is not a trier of facts; it is not its function to analyze or weigh evidence all over
again.19 Accordingly, findings of fact of the appellate court are generally conclusive on the Supreme Court. 20

Nevertheless, jurisprudence has recognized several exceptions in which factual issues may be resolved by this Court, such
as: (1) when the findings are grounded entirely on speculation, surmises or conjectures; (2) when the inference made is
manifestly mistaken, absurd or impossible; (3) when there is grave abuse of discretion; (4) when the judgment is based on a
misapprehension of facts; (5) when the findings of facts are conflicting; (6) when in making its findings the CA went beyond
the issues of the case, or its findings are contrary to the admissions of both the appellant and the appellee; (7) when the
findings are contrary to the trial court; (8) when the findings are conclusions without citation of specific evidence on which
they are based; (9) when the facts set forth in the petition as well as in the petitioner's main and reply briefs are not disputed
by the respondent; (10) when the findings of fact are premised on the supposed absence of evidence and contradicted by the
evidence on record; and (11) when the CA manifestly overlooked certain relevant facts not disputed by the parties, which, if
properly considered, would justify a different conclusion. 21 Exceptions (4), (5), (7), and (11) apply to the present petition.

At issue is the proper interpretation of the questioned insurance policy. Petitioner claims that the CA erred in construing a fire
insurance policy on book debts as one covering the unpaid accounts of IMC and LSPI since such insurance applies to loss of
the ready-made clothing materials sold and delivered to petitioner.

The Court disagrees with petitioner's stand.

It is well-settled that when the words of a contract are plain and readily understood, there is no room for construction. 22 In this
case, the questioned insurance policies provide coverage for "book debts in connection with ready-made clothing materials
which have been sold or delivered to various customers and dealers of the Insured anywhere in the Philippines." 23 ; and
defined book debts as the "unpaid account still appearing in the Book of Account of the Insured 45 days after the time of the
loss covered under this Policy."24 Nowhere is it provided in the questioned insurance policies that the subject of the insurance
is the goods sold and delivered to the customers and dealers of the insured.

Indeed, when the terms of the agreement are clear and explicit that they do not justify an attempt to read into it any alleged
intention of the parties, the terms are to be understood literally just as they appear on the face of the contract. 25 Thus, what
were insured against were the accounts of IMC and LSPI with petitioner which remained unpaid 45 days after the loss
through fire, and not the loss or destruction of the goods delivered.

Petitioner argues that IMC bears the risk of loss because it expressly reserved ownership of the goods by stipulating in the
sales invoices that "[i]t is further agreed that merely for purpose of securing the payment of the purchase price the above
described merchandise remains the property of the vendor until the purchase price thereof is fully paid." 26

The Court is not persuaded.

The present case clearly falls under paragraph (1), Article 1504 of the Civil Code:

ART. 1504. Unless otherwise agreed, the goods remain at the seller's risk until the ownership therein is transferred to the
buyer, but when the ownership therein is transferred to the buyer the goods are at the buyer's risk whether actual delivery
has been made or not, except that:

(1) Where delivery of the goods has been made to the buyer or to a bailee for the buyer, in pursuance of the contract and the
ownership in the goods has been retained by the seller merely to secure performance by the buyer of his obligations under
the contract, the goods are at the buyer's risk from the time of such delivery; (Emphasis supplied)

xxxx

Thus, when the seller retains ownership only to insure that the buyer will pay its debt, the risk of loss is borne by the
buyer.27 Accordingly, petitioner bears the risk of loss of the goods delivered.

IMC and LSPI did not lose complete interest over the goods. They have an insurable interest until full payment of the value of
the delivered goods. Unlike the civil law concept of res perit domino, where ownership is the basis for consideration of who
bears the risk of loss, in property insurance, one's interest is not determined by concept of title, but whether insured has
substantial economic interest in the property. 28

Section 13 of our Insurance Code defines insurable interest as "every interest in property, whether real or personal, or any
relation thereto, or liability in respect thereof, of such nature that a contemplated peril might directly damnify the insured."
Parenthetically, under Section 14 of the same Code, an insurable interest in property may consist in: (a) an existing interest;
(b) an inchoate interest founded on existing interest; or (c) an expectancy, coupled with an existing interest in that out of
which the expectancy arises.

Therefore, an insurable interest in property does not necessarily imply a property interest in, or a lien upon, or possession of,
the subject matter of the insurance, and neither the title nor a beneficial interest is requisite to the existence of such an
interest, it is sufficient that the insured is so situated with reference to the property that he would be liable to loss should it be
injured or destroyed by the peril against which it is insured. 29 Anyone has an insurable interest in property who derives a
benefit from its existence or would suffer loss from its destruction. 30Indeed, a vendor or seller retains an insurable interest in
the property sold so long as he has any interest therein, in other words, so long as he would suffer by its destruction, as
where he has a vendor's lien.31 In this case, the insurable interest of IMC and LSPI pertain to the unpaid accounts appearing
in their Books of Account 45 days after the time of the loss covered by the policies.

The next question is: Is petitioner liable for the unpaid accounts?

Petitioner's argument that it is not liable because the fire is a fortuitous event under Article 1174 32 of the Civil Code is
misplaced. As held earlier, petitioner bears the loss under Article 1504 (1) of the Civil Code.

Moreover, it must be stressed that the insurance in this case is not for loss of goods by fire but for petitioner's accounts with
IMC and LSPI that remained unpaid 45 days after the fire. Accordingly, petitioner's obligation is for the payment of money. As
correctly stated by the CA, where the obligation consists in the payment of money, the failure of the debtor to make the
payment even by reason of a fortuitous event shall not relieve him of his liability. 33 The rationale for this is that the rule that an
obligor should be held exempt from liability when the loss occurs thru a fortuitous event only holds true when the obligation
consists in the delivery of a determinate thing and there is no stipulation holding him liable even in case of fortuitous event. It
does not apply when the obligation is pecuniary in nature. 34

Under Article 1263 of the Civil Code, "[i]n an obligation to deliver a generic thing, the loss or destruction of anything of the
same kind does not extinguish the obligation." If the obligation is generic in the sense that the object thereof is designated
merely by its class or genus without any particular designation or physical segregation from all others of the same class, the
loss or destruction of anything of the same kind even without the debtor's fault and before he has incurred in delay will not
have the effect of extinguishing the obligation.35This rule is based on the principle that the genus of a thing can never perish.
Genus nunquan perit.36 An obligation to pay money is generic; therefore, it is not excused by fortuitous loss of any specific
property of the debtor.37

Thus, whether fire is a fortuitous event or petitioner was negligent are matters immaterial to this case. What is relevant here
is whether it has been established that petitioner has outstanding accounts with IMC and LSPI.

With respect to IMC, the respondent has adequately established its claim. Exhibits "C" to "C-22" 38 show that petitioner has an
outstanding account with IMC in the amount of P2,119,205.00. Exhibit "E"39 is the check voucher evidencing payment to IMC.
Exhibit "F"40 is the subrogation receipt executed by IMC in favor of respondent upon receipt of the insurance proceeds. All
these documents have been properly identified, presented and marked as exhibits in court. The subrogation receipt, by itself,
is sufficient to establish not only the relationship of respondent as insurer and IMC as the insured, but also the amount paid
to settle the insurance claim. The right of subrogation accrues simply upon payment by the insurance company of the
insurance claim.41Respondent's action against petitioner is squarely sanctioned by Article 2207 of the Civil Code which
provides:

Art. 2207. If the plaintiff's property has been insured, and he has received indemnity from the insurance company for the
injury or loss arising out of the wrong or breach of contract complained of, the insurance company shall be subrogated to the
rights of the insured against the wrongdoer or the person who has violated the contract. x x x

Petitioner failed to refute respondent's evidence.

As to LSPI, respondent failed to present sufficient evidence to prove its cause of action. No evidentiary weight can be given
to Exhibit "F Levi Strauss",42 a letter dated April 23, 1991 from petitioner's General Manager, Stephen S. Gaisano, Jr., since it
is not an admission of petitioner's unpaid account with LSPI. It only confirms the loss of Levi's products in the amount
of P535,613.00 in the fire that razed petitioner's building on February 25, 1991.

Moreover, there is no proof of full settlement of the insurance claim of LSPI; no subrogation receipt was offered in evidence.
Thus, there is no evidence that respondent has been subrogated to any right which LSPI may have against petitioner. Failure
to substantiate the claim of subrogation is fatal to petitioner's case for recovery of the amount of P535,613.00.

WHEREFORE, the petition is partly GRANTED. The assailed Decision dated October 11, 2000 and Resolution dated April
11, 2001 of the Court of Appeals in CA-G.R. CV No. 61848 are AFFIRMED with the MODIFICATIONthat the order to pay the
amount of P535,613.00 to respondent is DELETED for lack of factual basis.

No pronouncement as to costs.

SO ORDERED.
IV. Devices for ascertaining and controlling risk and loss

G.R. No. 113899 October 13, 1999

GREAT PACIFIC LIFE ASSURANCE CORP., petitioner,


vs.
COURT OF APPEALS AND MEDARDA V. LEUTERIO, respondents.

QUISUMBING, J.:

This petition for review, under Rule 45 of the Rules of Court, assails the Decision 1 dated May 17, 1993, of the Court of
Appeals and its Resolution 2 dated January 4, 1994 in CA-G.R. CV No. 18341. The appellate court affirmed in toto the
judgment of the Misamis Oriental Regional Trial Court, Branch 18, in an insurance claim filed by private respondent
against Great Pacific Life Assurance Co. The dispositive portion of the trial court's decision reads:

WHEREFORE, judgment is rendered adjudging the defendant GREAT PACIFIC LIFE ASSURANCE
CORPORATION as insurer under its Group policy No. G-1907, in relation to Certification B-18558 liable and
ordered to pay to the DEVELOPMENT BANK OF THE PHILIPPINES as creditor of the insured Dr. Wilfredo
Leuterio, the amount of EIGHTY SIX THOUSAND TWO HUNDRED PESOS (P86,200.00); dismissing the
claims for damages, attorney's fees and litigation expenses in the complaint and counterclaim, with costs
against the defendant and dismissing the complaint in respect to the plaintiffs, other than the widow-
beneficiary, for lack of cause of action. 3

The facts, as found by the Court of Appeals, are as follows:

A contract of group life insurance was executed between petitioner Great Pacific Life Assurance Corporation (hereinafter
Grepalife) and Development Bank of the Philippines (hereinafter DBP). Grepalife agreed to insure the lives of eligible
housing loan mortgagors of DBP.

On November 11, 1983, Dr. Wilfredo Leuterio, a physician and a housing debtor of DBP applied for membership in the group
life insurance plan. In an application form, Dr. Leuterio answered questions concerning his health condition as follows:

7. Have you ever had, or consulted, a physician for a heart condition, high blood pressure,
cancer, diabetes, lung; kidney or stomach disorder or any other physical impairment?

Answer: No. If so give details _____________.

8. Are you now, to the best of your knowledge, in good health?

Answer: [x] Yes [ ] NO. 4

On November 15, 1983, Grepalife issued Certificate No. B-18558, as insurance coverage of Dr. Leuterio, to the extent of his
DBP mortgage indebtedness amounting to eighty-six thousand, two hundred (P86,200.00) pesos. 1wphi1.nt

On August 6, 1984, Dr. Leuterio died due to "massive cerebral hemorrhage." Consequently, DBP submitted a death claim to
Grepalife. Grepalife denied the claim alleging that Dr. Leuterio was not physically healthy when he applied for an insurance
coverage on November 15, 1983. Grepalife insisted that Dr. Leuterio did not disclose he had been suffering from
hypertension, which caused his death. Allegedly, such non-disclosure constituted concealment that justified the denial of the
claim.

On October 20, 1986, the widow of the late Dr. Leuterio, respondent Medarda V. Leuterio, filed a complaint with the Regional
Trial Court of Misamis Oriental, Branch 18, against Grepalife for "Specific Performance with Damages." 5 During the trial, Dr.
Hernando Mejia, who issued the death certificate, was called to testify. Dr. Mejia's findings, based partly from the
information given by the respondent widow, stated that Dr. Leuterio complained of headaches presumably due to high
blood pressure. The inference was not conclusive because Dr. Leuterio was not autopsied, hence, other causes were
not ruled out.

On February 22, 1988, the trial court rendered a decision in favor of respondent widow and against Grepalife. On May 17,
1993, the Court of Appeals sustained the trial court's decision. Hence, the present petition. Petitioners interposed the
following assigned errors:

1. THE LOWER COURT ERRED IN HOLDING DEFENDANT-APPELLANT LIABLE TO


THE DEVELOPMENT BANK OF THE PHILIPPINES (DBP) WHICH IS NOT A PARTY TO
THE CASE FOR PAYMENT OF THE PROCEEDS OF A MORTGAGE REDEMPTION
INSURANCE ON THE LIFE OF PLAINTIFF'S HUSBAND WILFREDO LEUTERIO ONE
OF ITS LOAN BORROWERS, INSTEAD OF DISMISSING THE CASE AGAINST
DEFENDANT-APPELLANT [Petitioner Grepalife] FOR LACK OF CAUSE OF ACTION.

2. THE LOWER COURT ERRED IN NOT DISMISSING THE CASE FOR WANT OF
JURISDICTION OVER THE SUBJECT OR NATURE OF THE ACTION AND OVER THE
PERSON OF THE DEFENDANT.
3. THE LOWER COURT ERRED IN ORDERING DEFENDANT-APPELLANT TO PAY TO
DBP THE AMOUNT OF P86,200.00 IN THE ABSENCE OF ANY EVIDENCE TO SHOW
HOW MUCH WAS THE ACTUAL AMOUNT PAYABLE TO DBP IN ACCORDANCE WITH
ITS GROUP INSURANCE CONTRACT WITH DEFENDANT-APPELLANT.

4. THE LOWER COURT ERRED IN HOLDING THAT THERE WAS NO CONCEALMENT


OF MATERIAL INFORMATION ON THE PART OF WILFREDO LEUTERIO IN HIS
APPLICATION FOR MEMBERSHIP IN THE GROUP LIFE INSURANCE PLAN BETWEEN
DEFENDANT-APPELLANT OF THE INSURANCE CLAIM ARISING FROM THE DEATH
OF WILFREDO LEUTERIO. 6

Synthesized below are the assigned errors for our resolution:

1. Whether the Court of Appeals erred in holding petitioner liable to DBP as beneficiary in
a group life insurance contract from a complaint filed by the widow of the
decedent/mortgagor?

2. Whether the Court of Appeals erred in not finding that Dr. Leuterio concealed that he
had hypertension, which would vitiate the insurance contract?

3. Whether the Court of Appeals erred in holding Grepalife liable in the amount of eighty
six thousand, two hundred (P86,200.00) pesos without proof of the actual outstanding
mortgage payable by the mortgagor to DBP.

Petitioner alleges that the complaint was instituted by the widow of Dr. Leuterio, not the real party in interest, hence the trial
court acquired no jurisdiction over the case. It argues that when the Court of Appeals affirmed the trial court's judgment,
Grepalife was held liable to pay the proceeds of insurance contract in favor of DBP, the indispensable party who was not
joined in the suit.

To resolve the issue, we must consider the insurable interest in mortgaged properties and the parties to this type of contract.
The rationale of a group insurance policy of mortgagors, otherwise known as the "mortgage redemption insurance," is a
device for the protection of both the mortgagee and the mortgagor. On the part of the mortgagee, it has to enter into such
form of contract so that in the event of the unexpected demise of the mortgagor during the subsistence of the mortgage
contract, the proceeds from such insurance will be applied to the payment of the mortgage debt, thereby relieving the heirs of
the mortgagor from paying the obligation. 7 In a similar vein, ample protection is given to the mortgagor under such a
concept so that in the event of death; the mortgage obligation will be extinguished by the application of the insurance
proceeds to the mortgage indebtedness. 8 Consequently, where the mortgagor pays the insurance premium under the
group insurance policy, making the loss payable to the mortgagee, the insurance is on the mortgagor's interest, and
the mortgagor continues to be a party to the contract. In this type of policy insurance, the mortgagee is simply an
appointee of the insurance fund, such loss-payable clause does not make the mortgagee a party to the contract. 9

Sec. 8 of the Insurance Code provides:

Unless the policy provides, where a mortgagor of property effects insurance in his own name providing that
the loss shall be payable to the mortgagee, or assigns a policy of insurance to a mortgagee, the insurance
is deemed to be upon the interest of the mortgagor, who does not cease to be a party to the original
contract, and any act of his, prior to the loss, which would otherwise avoid the insurance, will have the same
effect, although the property is in the hands of the mortgagee, but any act which, under the contract of
insurance, is to be performed by the mortgagor, may be performed by the mortgagee therein named, with
the same effect as if it had been performed by the mortgagor.

The insured private respondent did not cede to the mortgagee all his rights or interests in the insurance, the policy stating
that: "In the event of the debtor's death before his indebtedness with the Creditor [DBP] shall have been fully paid, an amount
to pay the outstanding indebtedness shall first be paid to the creditor and the balance of sum assured, if there is any, shall
then be paid to the beneficiary/ies designated by the debtor." 10 When DBP submitted the insurance claim against
petitioner, the latter denied payment thereof, interposing the defense of concealment committed by the insured.
Thereafter, DBP collected the debt from the mortgagor and took the necessary action of foreclosure on the residential
lot of private respondent. 11 In Gonzales La O vs. Yek Tong Lin Fire & Marine Ins. Co. 12 we held:

Insured, being the person with whom the contract was made, is primarily the proper person to bring suit
thereon. * * * Subject to some exceptions, insured may thus sue, although the policy is taken wholly or in
part for the benefit of another person named or unnamed, and although it is expressly made payable to
another as his interest may appear or otherwise. * * * Although a policy issued to a mortgagor is taken out
for the benefit of the mortgagee and is made payable to him, yet the mortgagor may sue thereon in his own
name, especially where the mortgagee's interest is less than the full amount recoverable under the policy, *
* *.

And in volume 33, page 82, of the same work, we read the following:

Insured may be regarded as the real party in interest, although he has assigned the policy for the purpose
of collection, or has assigned as collateral security any judgment he may obtain. 13

And since a policy of insurance upon life or health may pass by transfer, will or succession to any person, whether he has an
insurable interest or not, and such person may recover it whatever the insured might have recovered, 14 the widow of the
decedent Dr. Leuterio may file the suit against the insurer, Grepalife.

The second assigned error refers to an alleged concealment that the petitioner interposed as its defense to annul the
insurance contract. Petitioner contends that Dr. Leuterio failed to disclose that he had hypertension, which might have
caused his death. Concealment exists where the assured had knowledge of a fact material to the risk, and honesty, good
faith, and fair dealing requires that he should communicate it to the assured, but he designedly and intentionally withholds
the same. 15

Petitioner merely relied on the testimony of the attending physician, Dr. Hernando Mejia, as supported by the information
given by the widow of the decedent. Grepalife asserts that Dr. Mejia's technical diagnosis of the cause of death of Dr.
Leuterio was a duly documented hospital record, and that the widow's declaration that her husband had "possible
hypertension several years ago" should not be considered as hearsay, but as part of res gestae.

On the contrary the medical findings were not conclusive because Dr. Mejia did not conduct an autopsy on the body of the
decedent. As the attending physician, Dr. Mejia stated that he had no knowledge of Dr. Leuterio's any previous hospital
confinement. 16 Dr. Leuterio's death certificate stated that hypertension was only "the possible cause of death." The
private respondent's statement, as to the medical history of her husband, was due to her unreliable recollection of
events. Hence, the statement of the physician was properly considered by the trial court as hearsay.

The question of whether there was concealment was aptly answered by the appellate court, thus:

The insured, Dr. Leuterio, had answered in his insurance application that he was in good health and that he
had not consulted a doctor or any of the enumerated ailments, including hypertension; when he died the
attending physician had certified in the death certificate that the former died of cerebral hemorrhage,
probably secondary to hypertension. From this report, the appellant insurance company refused to pay the
insurance claim. Appellant alleged that the insured had concealed the fact that he had hypertension.

Contrary to appellant's allegations, there was no sufficient proof that the insured had suffered from
hypertension. Aside from the statement of the insured's widow who was not even sure if the medicines
taken by Dr. Leuterio were for hypertension, the appellant had not proven nor produced any witness who
could attest to Dr. Leuterio's medical history . . .

xxx xxx xxx

Appellant insurance company had failed to establish that there was concealment made by the insured,
hence, it cannot refuse payment of the claim. 17

The fraudulent intent on the part of the insured must be established to entitle the insurer to rescind the
contract.18 Misrepresentation as a defense of the insurer to avoid liability is an affirmative defense and the duty to
establish such defense by satisfactory and convincing evidence rests upon the insurer. 19 In the case at bar, the
petitioner failed to clearly and satisfactorily establish its defense, and is therefore liable to pay the proceeds of the
insurance. 1wphi1.nt

And that brings us to the last point in the review of the case at bar. Petitioner claims that there was no evidence as to the
amount of Dr. Leuterio's outstanding indebtedness to DBP at the time of the mortgagor's death. Hence, for private
respondent's failure to establish the same, the action for specific performance should be dismissed. Petitioner's claim is
without merit. A life insurance policy is a valued policy. 20 Unless the interest of a person insured is susceptible of exact
pecuniary measurement, the measure of indemnity under a policy of insurance upon life or health is the sum fixed in
the policy. 21 The mortgagor paid the premium according to the coverage of his insurance, which states that:

The policy states that upon receipt of due proof of the Debtor's death during the terms of this insurance, a
death benefit in the amount of P86,200.00 shall be paid.

In the event of the debtor's death before his indebtedness with the creditor shall have been fully paid, an
amount to pay the outstanding indebtedness shall first be paid to the Creditor and the balance of the Sum
Assured, if there is any shall then be paid to the beneficiary/ies designated by the debtor." 22 (Emphasis
omitted)

However, we noted that the Court of Appeals' decision was promulgated on May 17, 1993. In private respondent's
memorandum, she states that DBP foreclosed in 1995 their residential lot, in satisfaction of mortgagor's outstanding loan.
Considering this supervening event, the insurance proceeds shall inure to the benefit of the heirs of the deceased person or
his beneficiaries. Equity dictates that DBP should not unjustly enrich itself at the expense of another (Nemo cum alterius
detrimenio protest). Hence, it cannot collect the insurance proceeds, after it already foreclosed on the mortgage. The
proceeds now rightly belong to Dr. Leuterio's heirs represented by his widow, herein private respondent Medarda Leuterio.

WHEREFORE, the petition is hereby DENIED. The Decision and Resolution of the Court of Appeals in CA-G.R. CV 18341 is
AFFIRMED with MODIFICATION that the petitioner is ORDERED to pay the insurance proceeds amounting to Eighty-six
thousand, two hundred (P86,200.00) pesos to the heirs of the insured, Dr. Wilfredo Leuterio (deceased), upon presentation
of proof of prior settlement of mortgagor's indebtedness to Development Bank of the Philippines. Costs against petitioner. 1wphi1.nt

SO ORDERED.

Mendoza, Buena and De Leon, Jr., JJ., concu

G.R. No. 105135 June 22, 1995

SUNLIFE ASSURANCE COMPANY OF CANADA, petitioner,


vs.
The Hon. COURT OF APPEALS and Spouses ROLANDO and BERNARDA BACANI, respondents.

QUIASON, J.:

This is a petition for review for certiorari under Rule 45 of the Revised Rules of Court to reverse and set aside the Decision
dated February 21, 1992 of the Court of Appeals in CA-G.R. CV No. 29068, and its Resolution dated April 22, 1992, denying
reconsideration thereof.

We grant the petition.


I

On April 15, 1986, Robert John B. Bacani procured a life insurance contract for himself from petitioner. He was issued Policy
No. 3-903-766-X valued at P100,000.00, with double indemnity in case of accidental death. The designated beneficiary was
his mother, respondent Bernarda Bacani.

On June 26, 1987, the insured died in a plane crash. Respondent Bernarda Bacani filed a claim with petitioner, seeking the
benefits of the insurance policy taken by her son. Petitioner conducted an investigation and its findings prompted it to reject
the claim.

In its letter, petitioner informed respondent Bernarda Bacani, that the insured did not disclose material facts relevant to the
issuance of the policy, thus rendering the contract of insurance voidable. A check representing the total premiums paid in the
amount of P10,172.00 was attached to said letter.

Petitioner claimed that the insured gave false statements in his application when he answered the following questions:

5. Within the past 5 years have you:

a) consulted any doctor or other health practitioner?

b) submitted to:

EGG?
X-rays?
blood tests?
other tests?

c) attended or been admitted to any hospital or other medical facility?

6. Have you ever had or sought advice for:

xxx xxx xxx

b) urine, kidney or bladder disorder? (Rollo, p. 53)

The deceased answered question No. 5(a) in the affirmative but limited his answer to a consultation with a certain Dr.
Reinaldo D. Raymundo of the Chinese General Hospital on February 1986, for cough and flu complications. The other
questions were answered in the negative (Rollo, p. 53).

Petitioner discovered that two weeks prior to his application for insurance, the insured was examined and confined at the
Lung Center of the Philippines, where he was diagnosed for renal failure. During his confinement, the deceased was
subjected to urinalysis, ultra-sonography and hematology tests.

On November 17, 1988, respondent Bernarda Bacani and her husband, respondent Rolando Bacani, filed an action for
specific performance against petitioner with the Regional Trial Court, Branch 191, Valenzuela, Metro Manila. Petitioner filed
its answer with counterclaim and a list of exhibits consisting of medical records furnished by the Lung Center of the
Philippines.

On January 14, 1990, private respondents filed a "Proposed Stipulation with Prayer for Summary Judgment" where they
manifested that they "have no evidence to refute the documentary evidence of concealment/misrepresentation by the
decedent of his health condition (Rollo, p. 62).

Petitioner filed its Request for Admissions relative to the authenticity and due execution of several documents as well as
allegations regarding the health of the insured. Private respondents failed to oppose said request or reply thereto, thereby
rendering an admission of the matters alleged.

Petitioner then moved for a summary judgment and the trial court decided in favor of private respondents. The dispositive
portion of the decision is reproduced as follows:
WHEREFORE, judgment is hereby rendered in favor of the plaintiffs and against the defendant,
condemning the latter to pay the former the amount of One Hundred Thousand Pesos (P100,000.00) the
face value of insured's Insurance Policy No. 3903766, and the Accidental Death Benefit in the amount of
One Hundred Thousand Pesos (P100,000.00) and further sum of P5,000.00 in the concept of reasonable
attorney's fees and costs of suit.

Defendant's counterclaim is hereby Dismissed (Rollo, pp. 43-44).

In ruling for private respondents, the trial court concluded that the facts concealed by the insured were made in good faith
and under a belief that they need not be disclosed. Moreover, it held that the health history of the insured was immaterial
since the insurance policy was "non-medical".

Petitioner appealed to the Court of Appeals, which affirmed the decision of the trial court. The appellate court ruled that
petitioner cannot avoid its obligation by claiming concealment because the cause of death was unrelated to the facts
concealed by the insured. It also sustained the finding of the trial court that matters relating to the health history of the
insured were irrelevant since petitioner waived the medical examination prior to the approval and issuance of the insurance
policy. Moreover, the appellate court agreed with the trial court that the policy was "non-medical" (Rollo, pp. 4-5).

Petitioner's motion for reconsideration was denied; hence, this petition.

II

We reverse the decision of the Court of Appeals.

The rule that factual findings of the lower court and the appellate court are binding on this Court is not absolute and admits of
exceptions, such as when the judgment is based on a misappreciation of the facts (Geronimo v. Court of Appeals, 224 SCRA
494 [1993]).

In weighing the evidence presented, the trial court concluded that indeed there was concealment and misrepresentation,
however, the same was made in "good faith" and the facts concealed or misrepresented were irrelevant since the policy was
"non-medical". We disagree.

Section 26 of The Insurance Code is explicit in requiring a party to a contract of insurance to communicate to the other, in
good faith, all facts within his knowledge which are material to the contract and as to which he makes no warranty, and which
the other has no means of ascertaining. Said Section provides:

A neglect to communicate that which a party knows and ought to communicate, is called concealment.

Materiality is to be determined not by the event, but solely by the probable and reasonable influence of the facts upon the
party to whom communication is due, in forming his estimate of the disadvantages of the proposed contract or in making his
inquiries (The Insurance Code, Sec. 31).

The terms of the contract are clear. The insured is specifically required to disclose to the insurer matters relating to his
health.

The information which the insured failed to disclose were material and relevant to the approval and issuance of the insurance
policy. The matters concealed would have definitely affected petitioner's action on his application, either by approving it with
the corresponding adjustment for a higher premium or rejecting the same. Moreover, a disclosure may have warranted a
medical examination of the insured by petitioner in order for it to reasonably assess the risk involved in accepting the
application.

In Vda. de Canilang v. Court of Appeals, 223 SCRA 443 (1993), we held that materiality of the information withheld does not
depend on the state of mind of the insured. Neither does it depend on the actual or physical events which ensue.

Thus, "goad faith" is no defense in concealment. The insured's failure to disclose the fact that he was hospitalized for two
weeks prior to filing his application for insurance, raises grave doubts about his bonafides. It appears that such concealment
was deliberate on his part.

The argument, that petitioner's waiver of the medical examination of the insured debunks the materiality of the facts
concealed, is untenable. We reiterate our ruling in Saturnino v. Philippine American Life Insurance Company, 7 SCRA 316
(1963), that " . . . the waiver of a medical examination [in a non-medical insurance contract] renders even more material the
information required of the applicant concerning previous condition of health and diseases suffered, for such information
necessarily constitutes an important factor which the insurer takes into consideration in deciding whether to issue the policy
or not . . . "

Moreover, such argument of private respondents would make Section 27 of the Insurance Code, which allows the injured
party to rescind a contract of insurance where there is concealment, ineffective (See Vda. de Canilang v. Court of
Appeals, supra).

Anent the finding that the facts concealed had no bearing to the cause of death of the insured, it is well settled that the
insured need not die of the disease he had failed to disclose to the insurer. It is sufficient that his non-disclosure misled the
insurer in forming his estimates of the risks of the proposed insurance policy or in making inquiries (Henson v. The Philippine
American Life Insurance Co., 56 O.G. No. 48 [1960]).

We, therefore, rule that petitioner properly exercised its right to rescind the contract of insurance by reason of the
concealment employed by the insured. It must be emphasized that rescission was exercised within the two-year
contestability period as recognized in Section 48 of The Insurance Code.

WHEREFORE, the petition is GRANTED and the Decision of the Court of Appeals is REVERSED and SET ASIDE.

SO ORDERED.

Padilla, Davide, Jr., Bellosillo and Kapunan, JJ., concur.

G.R. No. 125678 March 18, 2002

PHILAMCARE HEALTH SYSTEMS, INC., petitioner,


vs.
COURT OF APPEALS and JULITA TRINOS, respondents.

YNARES-SANTIAGO, J.:

Ernani Trinos, deceased husband of respondent Julita Trinos, applied for a health care coverage with petitioner Philamcare
Health Systems, Inc. In the standard application form, he answered no to the following question:

Have you or any of your family members ever consulted or been treated for high blood pressure, heart trouble,
diabetes, cancer, liver disease, asthma or peptic ulcer? (If Yes, give details). 1
The application was approved for a period of one year from March 1, 1988 to March 1, 1989. Accordingly, he was issued
Health Care Agreement No. P010194. Under the agreement, respondents husband was entitled to avail of hospitalization
benefits, whether ordinary or emergency, listed therein. He was also entitled to avail of "out-patient benefits" such as annual
physical examinations, preventive health care and other out-patient services.

Upon the termination of the agreement, the same was extended for another year from March 1, 1989 to March 1, 1990, then
from March 1, 1990 to June 1, 1990. The amount of coverage was increased to a maximum sum of P75,000.00 per
disability.2

During the period of his coverage, Ernani suffered a heart attack and was confined at the Manila Medical Center (MMC) for
one month beginning March 9, 1990. While her husband was in the hospital, respondent tried to claim the benefits under the
health care agreement. However, petitioner denied her claim saying that the Health Care Agreement was void. According to
petitioner, there was a concealment regarding Ernanis medical history. Doctors at the MMC allegedly discovered at the time
of Ernanis confinement that he was hypertensive, diabetic and asthmatic, contrary to his answer in the application form.
Thus, respondent paid the hospitalization expenses herself, amounting to about P76,000.00.

After her husband was discharged from the MMC, he was attended by a physical therapist at home. Later, he was admitted
at the Chinese General Hospital. Due to financial difficulties, however, respondent brought her husband home again. In the
morning of April 13, 1990, Ernani had fever and was feeling very weak. Respondent was constrained to bring him back to the
Chinese General Hospital where he died on the same day.

On July 24, 1990, respondent instituted with the Regional Trial Court of Manila, Branch 44, an action for damages against
petitioner and its president, Dr. Benito Reverente, which was docketed as Civil Case No. 90-53795. She asked for
reimbursement of her expenses plus moral damages and attorneys fees. After trial, the lower court ruled against
petitioners, viz:

WHEREFORE, in view of the forgoing, the Court renders judgment in favor of the plaintiff Julita Trinos, ordering:

1. Defendants to pay and reimburse the medical and hospital coverage of the late Ernani Trinos in the amount of
P76,000.00 plus interest, until the amount is fully paid to plaintiff who paid the same;

2. Defendants to pay the reduced amount of moral damages of P10,000.00 to plaintiff;

3. Defendants to pay the reduced amount of P10,000.00 as exemplary damages to plaintiff;

4. Defendants to pay attorneys fees of P20,000.00, plus costs of suit.

SO ORDERED.3

On appeal, the Court of Appeals affirmed the decision of the trial court but deleted all awards for damages and absolved
petitioner Reverente.4 Petitioners motion for reconsideration was denied.5 Hence, petitioner brought the instant petition for
review, raising the primary argument that a health care agreement is not an insurance contract; hence the "incontestability
clause" under the Insurance Code6 does not apply. 1wphi1.nt

Petitioner argues that the agreement grants "living benefits," such as medical check-ups and hospitalization which a member
may immediately enjoy so long as he is alive upon effectivity of the agreement until its expiration one-year thereafter.
Petitioner also points out that only medical and hospitalization benefits are given under the agreement without any
indemnification, unlike in an insurance contract where the insured is indemnified for his loss. Moreover, since Health Care
Agreements are only for a period of one year, as compared to insurance contracts which last longer, 7 petitioner argues that
the incontestability clause does not apply, as the same requires an effectivity period of at least two years. Petitioner further
argues that it is not an insurance company, which is governed by the Insurance Commission, but a Health Maintenance
Organization under the authority of the Department of Health.

Section 2 (1) of the Insurance Code defines a contract of insurance as an agreement whereby one undertakes for a
consideration to indemnify another against loss, damage or liability arising from an unknown or contingent event. An
insurance contract exists where the following elements concur:

1. The insured has an insurable interest;

2. The insured is subject to a risk of loss by the happening of the designated peril;
3. The insurer assumes the risk;

4. Such assumption of risk is part of a general scheme to distribute actual losses among a large group of persons
bearing a similar risk; and

5. In consideration of the insurers promise, the insured pays a premium. 8

Section 3 of the Insurance Code states that any contingent or unknown event, whether past or future, which may damnify a
person having an insurable interest against him, may be insured against. Every person has an insurable interest in the life
and health of himself. Section 10 provides:

Every person has an insurable interest in the life and health:

(1) of himself, of his spouse and of his children;

(2) of any person on whom he depends wholly or in part for education or support, or in whom he has a pecuniary
interest;

(3) of any person under a legal obligation to him for the payment of money, respecting property or service, of which
death or illness might delay or prevent the performance; and

(4) of any person upon whose life any estate or interest vested in him depends.

In the case at bar, the insurable interest of respondents husband in obtaining the health care agreement was his own health.
The health care agreement was in the nature of non-life insurance, which is primarily a contract of indemnity. 9 Once the
member incurs hospital, medical or any other expense arising from sickness, injury or other stipulated contingent, the health
care provider must pay for the same to the extent agreed upon under the contract.

Petitioner argues that respondents husband concealed a material fact in his application. It appears that in the application for
health coverage, petitioners required respondents husband to sign an express authorization for any person, organization or
entity that has any record or knowledge of his health to furnish any and all information relative to any hospitalization,
consultation, treatment or any other medical advice or examination. 10 Specifically, the Health Care Agreement signed by
respondents husband states:

We hereby declare and agree that all statement and answers contained herein and in any addendum annexed to
this application are full, complete and true and bind all parties in interest under the Agreement herein applied for,
that there shall be no contract of health care coverage unless and until an Agreement is issued on this application
and the full Membership Fee according to the mode of payment applied for is actually paid during the lifetime and
good health of proposed Members; that no information acquired by any Representative of PhilamCare shall be
binding upon PhilamCare unless set out in writing in the application;that any physician is, by these presents,
expressly authorized to disclose or give testimony at anytime relative to any information acquired by him in his
professional capacity upon any question affecting the eligibility for health care coverage of the Proposed
Members and that the acceptance of any Agreement issued on this application shall be a ratification of any
correction in or addition to this application as stated in the space for Home Office Endorsement. 11 (Underscoring
ours)

In addition to the above condition, petitioner additionally required the applicant for authorization to inquire about the
applicants medical history, thus:

I hereby authorize any person, organization, or entity that has any record or knowledge of my health and/or that of
__________ to give to the PhilamCare Health Systems, Inc. any and all information relative to any hospitalization,
consultation, treatment or any other medical advice or examination. This authorization is in connection with the
application for health care coverage only. A photographic copy of this authorization shall be as valid as the
original.12 (Underscoring ours)

Petitioner cannot rely on the stipulation regarding "Invalidation of agreement" which reads:

Failure to disclose or misrepresentation of any material information by the member in the application or medical
examination, whether intentional or unintentional, shall automatically invalidate the Agreement from the very
beginning and liability of Philamcare shall be limited to return of all Membership Fees paid. An undisclosed or
misrepresented information is deemed material if its revelation would have resulted in the declination of the applicant
by Philamcare or the assessment of a higher Membership Fee for the benefit or benefits applied for. 13

The answer assailed by petitioner was in response to the question relating to the medical history of the applicant. This largely
depends on opinion rather than fact, especially coming from respondents husband who was not a medical doctor. Where
matters of opinion or judgment are called for, answers made in good faith and without intent to deceive will not avoid a policy
even though they are untrue.14 Thus,

(A)lthough false, a representation of the expectation, intention, belief, opinion, or judgment of the insured will not
avoid the policy if there is no actual fraud in inducing the acceptance of the risk, or its acceptance at a lower rate of
premium, and this is likewise the rule although the statement is material to the risk, if the statement is obviously of
the foregoing character, since in such case the insurer is not justified in relying upon such statement, but is obligated
to make further inquiry. There is a clear distinction between such a case and one in which the insured is fraudulently
and intentionally states to be true, as a matter of expectation or belief, that which he then knows, to be actually
untrue, or the impossibility of which is shown by the facts within his knowledge, since in such case the intent to
deceive the insurer is obvious and amounts to actual fraud. 15 (Underscoring ours)

The fraudulent intent on the part of the insured must be established to warrant rescission of the insurance
contract.16 Concealment as a defense for the health care provider or insurer to avoid liability is an affirmative defense and the
duty to establish such defense by satisfactory and convincing evidence rests upon the provider or insurer. In any case, with
or without the authority to investigate, petitioner is liable for claims made under the contract. Having assumed a responsibility
under the agreement, petitioner is bound to answer the same to the extent agreed upon. In the end, the liability of the health
care provider attaches once the member is hospitalized for the disease or injury covered by the agreement or whenever he
avails of the covered benefits which he has prepaid.

Under Section 27 of the Insurance Code, "a concealment entitles the injured party to rescind a contract of insurance." The
right to rescind should be exercised previous to the commencement of an action on the contract. 17 In this case, no rescission
was made. Besides, the cancellation of health care agreements as in insurance policies require the concurrence of the
following conditions:

1. Prior notice of cancellation to insured;

2. Notice must be based on the occurrence after effective date of the policy of one or more of the grounds mentioned;

3. Must be in writing, mailed or delivered to the insured at the address shown in the policy;

4. Must state the grounds relied upon provided in Section 64 of the Insurance Code and upon request of insured, to furnish
facts on which cancellation is based.18

None of the above pre-conditions was fulfilled in this case. When the terms of insurance contract contain limitations on
liability, courts should construe them in such a way as to preclude the insurer from non-compliance with his
obligation.19 Being a contract of adhesion, the terms of an insurance contract are to be construed strictly against the party
which prepared the contract the insurer. 20 By reason of the exclusive control of the insurance company over the terms and
phraseology of the insurance contract, ambiguity must be strictly interpreted against the insurer and liberally in favor of the
insured, especially to avoid forfeiture.21 This is equally applicable to Health Care Agreements. The phraseology used in
medical or hospital service contracts, such as the one at bar, must be liberally construed in favor of the subscriber, and if
doubtful or reasonably susceptible of two interpretations the construction conferring coverage is to be adopted, and
exclusionary clauses of doubtful import should be strictly construed against the provider. 22

Anent the incontestability of the membership of respondents husband, we quote with approval the following findings of the
trial court:

(U)nder the title Claim procedures of expenses, the defendant Philamcare Health Systems Inc. had twelve months
from the date of issuance of the Agreement within which to contest the membership of the patient if he had previous
ailment of asthma, and six months from the issuance of the agreement if the patient was sick of diabetes or
hypertension. The periods having expired, the defense of concealment or misrepresentation no longer lie. 23

Finally, petitioner alleges that respondent was not the legal wife of the deceased member considering that at the time of their
marriage, the deceased was previously married to another woman who was still alive. The health care agreement is in the
nature of a contract of indemnity. Hence, payment should be made to the party who incurred the expenses. It is not
controverted that respondent paid all the hospital and medical expenses. She is therefore entitled to reimbursement. The
records adequately prove the expenses incurred by respondent for the deceaseds hospitalization, medication and the
professional fees of the attending physicians.24

WHEREFORE, in view of the foregoing, the petition is DENIED. The assailed decision of the Court of Appeals dated
December 14, 1995 is AFFIRMED.

SO ORDERED.

Davide, Jr., C.J., Puno, and Kapunan, JJ., concur.

G.R. No. 92492 June 17, 1993

THELMA VDA. DE CANILANG, petitioner,


vs.
HON. COURT OF APPEALS and GREAT PACIFIC LIFE ASSURANCE CORPORATION, respondents.

Simeon C. Sato for petitioner.

FELICIANO, J.:

On 18 June 1982, Jaime Canilang consulted Dr. Wilfredo B. Claudio and was diagnosed as suffering from "sinus
tachycardia." The doctor prescribed the following fro him: Trazepam, a tranquilizer; and Aptin, a beta-blocker drug. Mr.
Canilang consulted the same doctor again on 3 August 1982 and this time was found to have "acute bronchitis."
On next day, 4 August 1982, Jaime Canilang applied for a "non-medical" insurance policy with respondent Great Pacific Life
Assurance Company ("Great Pacific") naming his wife, Thelma Canilang, as his beneficiary. 1 Jaime Canilang was issued
ordinary life insurance Policy No. 345163, with the face value of P19,700, effective as of 9 August 1982.

On 5 August 1983, Jaime Canilang died of "congestive heart failure," "anemia," and "chronic anemia." 2 Petitioner, widow
and beneficiary of the insured, filed a claim with Great Pacific which the insurer denied on 5 December 1983 upon the
ground that the insured had concealed material information from it.

Petitioner then filed a complaint against Great Pacific with the Insurance Commission for recovery of the insurance
proceeds. During the hearing called by the Insurance Commissioner, petitioner testified that she was not aware of any
serious illness suffered by her late husband 3 and that, as far as she knew, her husband had died because of a kidney
disorder. 4 A deposition given by Dr. Wilfredo Claudio was presented by petitioner. There Dr. Claudio stated that he was
the family physician of the deceased Jaime Canilang 5 and that he had previously treated him for "sinus tachycardia"
and "acute bronchitis." 6 Great Pacific for its part presented Dr. Esperanza Quismorio, a physician
and a medical underwriter working for Great Pacific. 7 She testified that the deceased's insurance application had been
approved on the basis of his medical declaration. 8 She explained that as a rule, medical examinations are required
only in cases where the applicant has indicated in his application for insurance coverage that he has previously
undergone medical consultation and hospitalization. 9

In a decision dated 5 November 1985, Insurance Commissioner Armando Ansaldo ordered Great Pacific to pay P19,700
plus legal interest and P2,000.00 as attorney's fees after holding that:

1. the ailment of Jaime Canilang was not so serious that, even if it had been disclosed, it would not have
affected Great Pacific's decision to insure him;

2. Great Pacific had waived its right to inquire into the health condition of the applicant by the issuance of
the policy despite the lack of answers to "some of the pertinent questions" in the insurance application;

3. there was no intentional concealment on the part of the insured Jaime Canilang as he had thought that
he was merely suffering from a minor ailment and simple cold; 10 and

4. Batas Pambansa Blg. 847 which voids an insurance contract, whether or not concealment was
intentionally made, was not applicable to Canilang's case as that law became effective only on 1 June
1985.

On appeal by Great Pacific, the Court of Appeals reversed and set aside the decision of the Insurance Commissioner and
dismissed Thelma Canilang's complaint and Great Pacific's counterclaim. The Court of Appealed found that the use of the
word "intentionally" by the Insurance Commissioner in defining and resolving the issue agreed upon by the parties at pre-trial
before the Insurance Commissioner was not supported by the evidence; that the issue agreed upon by the parties had been
whether the deceased insured, Jaime Canilang, made a material concealment as the state of his health at the time of the
filing of insurance application, justifying respondent's denial of the claim. The Court of Appeals also found that the failure of
Jaime Canilang to disclose previous medical consultation and treatment constituted material information which should have
been communicated to Great Pacific to enable the latter to make proper inquiries. The Court of Appeals finally held that
the Ng Gan Zee case which had involved misrepresentation was not applicable in respect of the case at bar which
involves concealment.

Petitioner Thelma Canilang is now before this Court on a Petition for Review on Certiorari alleging that:

1. . . . the Honorable Court of Appeals, speaking with due respect, erred in not holding that the issue in the
case agreed upon between the parties before the Insurance Commission is whether or not Jaime Canilang
"intentionally" made material concealment in stating his state of health;

2. . . . at any rate, the non-disclosure of certain facts about his previous health conditions does not amount
to fraud and private respondent is deemed to have waived inquiry thereto. 11

The medical declaration which was set out in the application for insurance executed by Jaime Canilang read as follows:

MEDICAL DECLARATION
I hereby declare that:

(1) I have not been confined in any hospital, sanitarium or infirmary, nor receive any medical or surgical
advice/attention within the last five (5) years.

(2) I have never been treated nor consulted a physician for a heart condition, high blood pressure, cancer,
diabetes, lung, kidney, stomach disorder, or any other physical impairment.

(3) I am, to the best of my knowledge, in good health.

EXCEPTIONS:

________________________________________________________________________________

GENERAL DECLARATION

I hereby declare that all the foregoing answers and statements are complete, true and correct. I
hereby agree that if there be any fraud or misrepresentation in the above statements material to the risk,
the INSURANCE COMPANY upon discovery within two (2) years from the effective date of insurance shall
have the right to declare such insurance null and void. That the liabilities of the Company under the said
Policy/TA/Certificate shall accrue and begin only from the date of commencement of risk stated in the
Policy/TA/Certificate, provided that the first premium is paid and the Policy/TA/Certificate is delivered to,
and accepted by me in person, when I am in actual good health.

Signed at Manila his 4th day of August, 1992.

Illegi
ble





Sign
atur
e of
Appl
icant
. 12

We note that in addition to the negative statements made by Mr. Canilang in paragraph 1 and 2 of the medical declaration,
he failed to disclose in the appropriate space, under the caption "Exceptions," that he had twice consulted Dr. Wilfredo B.
Claudio who had found him to be suffering from "sinus tachycardia" and "acute bronchitis."

The relevant statutory provisions as they stood at the time Great Pacific issued the contract of insurance and at the time
Jaime Canilang died, are set out in P.D. No. 1460, also known as the Insurance Code of 1978, which went into effect on 11
June 1978. These provisions read as follows:

Sec. 26. A neglect to communicate that which a party knows and ought to communicate, is called a
concealment.

xxx xxx xxx

Sec. 28. Each party to a contract of insurance must communicate to the other, in good faith, all factorswithin
his knowledge which are material to the contract and as to which he makes no warranty, and which the
other has not the means of ascertaining. (Emphasis supplied)

Under the foregoing provisions, the information concealed must be information which the concealing party knew and "ought
to [have] communicate[d]," that is to say, information which was "material to the contract." The test of materiality is contained
in Section 31 of the Insurance Code of 1978 which reads:
Sec. 31. Materially is to be determined not by the event, but solely by the probable and reasonable
influence of the facts upon the party to whom the communication is due, in forming his estimate of the
disadvantages of the proposed contract, or in making his inquiries. (Emphasis supplied)

"Sinus tachycardia" is considered present "when the heart rate exceeds 100 beats per minute."
13
The symptoms of this
condition include pounding in the chest and sometimes faintness and weakness of the person affected. The following
elaboration was offered by Great Pacific and set out by the Court of Appeals in its Decision:

Sinus tachycardia is defined as sinus-initiated; heart rate faster than 100 beats per minute. (Harrison' s
Principles of Internal Medicine, 8th ed. [1978], p. 1193.) It is, among others, a common reaction to heart
disease, including myocardial infarction, and heart failure per se. (Henry J.L. Marriot,
M.D.,Electrocardiography, 6th ed., [1977], p. 127.) The medication prescribed by Dr. Claudio for treatment
of Canilang's ailment on June 18, 1982, indicates the condition that said physician was trying to manage.
Thus, he prescribed Trazepam, (Philippine Index of Medical Specialties (PIMS), Vol. 14, No. 3, Dec. 1985,
p. 112) which is anti-anxiety, anti-convulsant, muscle-relaxant; and Aptin, (Idem, p. 36) a cardiac drug, for
palpitations and nervous heart. Such treatment could have been a very material information to the insurer in
determining the action to be take on Canilang's application for life insurance coverage. 14

We agree with the Court of Appeals that the information which Jaime Canilang failed to disclose was material to the ability of
Great Pacific to estimate the probable risk he presented as a subject of life insurance. Had Canilang disclosed his visits to
his doctor, the diagnosis made and medicines prescribed by such doctor, in the insurance application, it may be reasonably
assumed that Great Pacific would have made further inquiries and would have probably refused to issue a non-medical
insurance policy or, at the very least, required a higher premium for the same coverage. 15 The materiality of the information
withheld by Great Pacific did not depend upon the state of mind of Jaime Canilang. A man's state of mind or subjective
belief is not capable of proof in our judicial process, except through proof of external acts or failure to act from which
inferences as to his subjective belief may be reasonably drawn. Neither does materiality depend upon the actual or
physical events which ensue. Materiality relates rather to the "probable and reasonable influence of the facts" upon the
party to whom the communication should have been made, in assessing the risk involved in making or omitting to
make further inquiries and in accepting the application for insurance; that "probable and reasonable influence of the
facts" concealed must, of course, be determined objectively, by the judge ultimately.

The insurance Great Pacific applied for was a "non-medical" insurance policy. In Saturnino v. Philippine-American Life
Insurance Company, 16 this Court held that:

. . . if anything, the waiver of medical examination [in a non-medical insurance contract] renders even more
material the information required of the applicant concerning previous condition of health and diseases
suffered, for such information necessarily constitutes an important factor which the insurer takes into
consideration in deciding whether to issue the policy or not . . . . 17 (Emphasis supplied)

The Insurance Commissioner had also ruled that the failure of Great Pacific to convey certain information to the insurer was
not "intentional" in nature, for the reason that Jaime Canilang believed that he was suffering from minor ailment like a
common cold. Section 27 of the Insurance Code of 1978 as it existed from 1974 up to 1985, that is, throughout the time
range material for present purposes, provided that:

Sec. 27. A concealment entitles the injured party to rescind a contract of insurance.

The preceding statute, Act No. 2427, as it stood from 1914 up to 1974, had provided:

Sec. 26. A concealment, whether intentional or unintentional, entitles the injured party to rescind a contract
of insurance. (Emphasis supplied)

Upon the other hand, in 1985, the Insurance Code of 1978 was amended by
B.P. Blg. 874. This subsequent statute modified Section 27 of the Insurance Code of 1978 so as to read as follows:

Sec. 27. A concealment whether intentional or unintentional entitles the injured party to rescind a contract of
insurance. (Emphasis supplied)

The unspoken theory of the Insurance Commissioner appears to have been that by deleting the phrase "intentional or
unintentional," the Insurance Code of 1978 (prior to its amendment by B.P. Blg. 874) intended to limit the kinds of
concealment which generate a right to rescind on the part of the injured party to "intentional concealments." This argument is
not persuasive. As a simple matter of grammar, it may be noted that "intentional" and "unintentional" cancel each other out.
The net result therefore of the phrase "whether intentional or unitentional" is precisely to leave unqualified the term
"concealment." Thus, Section 27 of the Insurance Code of 1978 is properly read as referring to "any concealment" without
regard to whether such concealment is intentional or unintentional. The phrase "whether intentional or unintentional" was in
fact superfluous. The deletion of the phrase "whether intentional or unintentional" could not have had the effect of imposing
an affirmative requirement that a concealment must be intentional if it is to entitle the injured party to rescind a contract of
insurance. The restoration in 1985 by B.P. Blg. 874 of the phrase "whether intentional or unintentional" merely underscored
the fact that all throughout (from 1914 to 1985), the statute did not require proof that concealment must be "intentional" in
order to authorize rescission by the injured party.

In any case, in the case at bar, the nature of the facts not conveyed to the insurer was such that the failure to
communicate must have been intentional rather than merely inadvertent. For Jaime Canilang could not have been unaware
that his heart beat would at times rise to high and alarming levels and that he had consulted a doctor twice in the two (2)
months before applying for non-medical insurance. Indeed, the last medical consultation took place just the day before the
insurance application was filed. In all probability, Jaime Canilang went to visit his doctor precisely because of the discomfort
and concern brought about by his experiencing "sinus tachycardia."

We find it difficult to take seriously the argument that Great Pacific had waived inquiry into the concealment by issuing the
insurance policy notwithstanding Canilang's failure to set out answers to some of the questions in the insurance application.
Such failure precisely constituted concealment on the part of Canilang. Petitioner's argument, if accepted, would obviously
erase Section 27 from the Insurance Code of 1978.

It remains only to note that the Court of Appeals finding that the parties had not agreed in the pretrial before the Insurance
Commission that the relevant issue was whether or not Jaime Canilang had intentionally concealed material information from
the insurer, was supported by the evidence of record, i.e., the Pre-trial Order itself dated 17 October 1984 and the Minutes of
the Pre-trial Conference dated 15 October 1984, which "readily shows that the word "intentional" does not appear in the
statement or definition of the issue in the said Order and Minutes." 18

WHEREFORE, the Petition for Review is DENIED for lack of merit and the Decision of the Court of Appeals dated 16
October 1989 in C.A.-G.R. SP No. 08696 is hereby AFFIRMED. No pronouncement as to the costs.

SO ORDERED.

Bidin, Davide, Jr., Romero and Melo, JJ., concur.

G.R. No. 48049 June 29, 1989

EMILIO TAN, JUANITO TAN, ALBERTO TAN and ARTURO TAN, petitioners,
vs.
THE COURT OF APPEALS and THE PHILIPPINE AMERICAN LIFE INSURANCE COMPANY, respondents.

O.F. Santos & P.C. Nolasco for petitioners.

Ferry, De la Rosa and Associates for private respondent.


GUTIERREZ, JR., J.:

This is a petition for review on certiorari of the Court of Appeals' decision affirming the decision of the Insurance
Commissioner which dismissed the petitioners' complaint against respondent Philippine American Life Insurance Company
for the recovery of the proceeds from their late father's policy. The facts of the case as found by the Court of Appeals are:

Petitioners appeal from the Decision of the Insurance Commissioner dismissing herein petitioners'
complaint against respondent Philippine American Life Insurance Company for the recovery of the proceeds
of Policy No. 1082467 in the amount of P 80,000.00.

On September 23,1973, Tan Lee Siong, father of herein petitioners, applied for life insurance in the amount
of P 80,000.00 with respondent company. Said application was approved and Policy No. 1082467 was
issued effective November 6,1973, with petitioners the beneficiaries thereof (Exhibit A).

On April 26,1975, Tan Lee Siong died of hepatoma (Exhibit B). Petitioners then filed with respondent
company their claim for the proceeds of the life insurance policy. However, in a letter dated September 11,
1975, respondent company denied petitioners' claim and rescinded the policy by reason of the alleged
misrepresentation and concealment of material facts made by the deceased Tan Lee Siong in his
application for insurance (Exhibit 3). The premiums paid on the policy were thereupon refunded .

Alleging that respondent company's refusal to pay them the proceeds of the policy was unjustified and
unreasonable, petitioners filed on November 27, 1975, a complaint against the former with the Office of the
Insurance Commissioner, docketed as I.C. Case No. 218.

After hearing the evidence of both parties, the Insurance Commissioner rendered judgment on August 9,
1977, dismissing petitioners' complaint. (Rollo, pp. 91-92)

The Court of Appeals dismissed ' the petitioners' appeal from the Insurance Commissioner's decision for lack of merit

Hence, this petition.

The petitioners raise the following issues in their assignment of errors, to wit:

A. The conclusion in law of respondent Court that respondent insurer has the right to rescind the policy
contract when insured is already dead is not in accordance with existing law and applicable jurisprudence.

B. The conclusion in law of respondent Court that respondent insurer may be allowed to avoid the policy on
grounds of concealment by the deceased assured, is contrary to the provisions of the policy contract itself,
as well as, of applicable legal provisions and established jurisprudence.

C. The inference of respondent Court that respondent insurer was misled in issuing the policy are manifestly
mistaken and contrary to admitted evidence. (Rollo, p. 7)

The petitioners contend that the respondent company no longer had the right to rescind the contract of insurance as
rescission must allegedly be done during the lifetime of the insured within two years and prior to the commencement of
action.

The contention is without merit.

The pertinent section in the Insurance Code provides:

Section 48. Whenever a right to rescind a contract of insurance is given to the insurer by any provision of
this chapter, such right must be exercised previous to the commencement of an action on the contract.

After a policy of life insurance made payable on the death of the insured shall have been in force during the
lifetime of the insured for a period of two years from the date of its issue or of its last reinstatement, the
insurer cannot prove that the policy is void ab initio or is rescindable by reason of the fraudulent
concealment or misrepresentation of the insured or his agent.
According to the petitioners, the Insurance Law was amended and the second paragraph of Section 48 added to prevent the
insurance company from exercising a right to rescind after the death of the insured.

The so-called "incontestability clause" precludes the insurer from raising the defenses of false representations or
concealment of material facts insofar as health and previous diseases are concerned if the insurance has been in force for at
least two years during the insured's lifetime. The phrase "during the lifetime" found in Section 48 simply means that the policy
is no longer considered in force after the insured has died. The key phrase in the second paragraph of Section 48 is "for a
period of two years."

As noted by the Court of Appeals, to wit:

The policy was issued on November 6,1973 and the insured died on April 26,1975. The policy was thus in
force for a period of only one year and five months. Considering that the insured died before the two-year
period had lapsed, respondent company is not, therefore, barred from proving that the policy is void ab
initio by reason of the insured's fraudulent concealment or misrepresentation. Moreover, respondent
company rescinded the contract of insurance and refunded the premiums paid on September 11, 1975,
previous to the commencement of this action on November 27,1975. (Rollo, pp. 99-100)

xxx xxx xxx

The petitioners contend that there could have been no concealment or misrepresentation by their late father because Tan
Lee Siong did not have to buy insurance. He was only pressured by insistent salesmen to do so. The petitioners state:

Here then is a case of an assured whose application was submitted because of repeated visits and
solicitations by the insurer's agent. Assured did not knock at the door of the insurer to buy insurance. He
was the object of solicitations and visits.

Assured was a man of means. He could have obtained a bigger insurance, not just P 80,000.00. If his
purpose were to misrepresent and to conceal his ailments in anticipation of death during the two-year
period, he certainly could have gotten a bigger insurance. He did not.

Insurer Philamlife could have presented as witness its Medical Examiner Dr. Urbano Guinto. It was he who
accomplished the application, Part II, medical. Philamlife did not.

Philamlife could have put to the witness stand its Agent Bienvenido S. Guinto, a relative to Dr. Guinto, Again
Philamlife did not. (pp. 138139, Rollo)

xxx xxx xxx

This Honorable Supreme Court has had occasion to denounce the pressure and practice indulged in by
agents in selling insurance. At one time or another most of us have been subjected to that pressure, that
practice. This court took judicial cognizance of the whirlwind pressure of insurance selling-especially of the
agent's practice of 'supplying the information, preparing and answering the application, submitting the
application to their companies, concluding the transactions and otherwisesmoothing out all difficulties.

We call attention to what this Honorable Court said in Insular Life v. Feliciano, et al., 73 Phil. 201; at page 205:

It is of common knowledge that the selling of insurance today is subjected to the whirlwind pressureof
modern salesmanship.

Insurance companies send detailed instructions to their agents to solicit and procure applications.

These agents are to be found all over the length and breadth of the land. They are stimulated to more active
efforts by contests and by the keen competition offered by the other rival insurance companies.

They supply all the information, prepare and answer the applications, submit the applications to their
companies, conclude the transactions, and otherwise smooth out all difficulties.

The agents in short do what the company set them out to do.
The Insular Life case was decided some forty years ago when the pressure of insurance salesmanship was
not overwhelming as it is now; when the population of this country was less than one-fourth of what it is
now; when the insurance companies competing with one another could be counted by the fingers. (pp. 140-
142, Rollo)

xxx xxx xxx

In the face of all the above, it would be unjust if, having been subjected to the whirlwind pressure of
insurance salesmanship this Court itself has long denounced, the assured who dies within the two-year
period, should stand charged of fraudulent concealment and misrepresentation." (p. 142, Rollo)

The legislative answer to the arguments posed by the petitioners is the "incontestability clause" added by the second
paragraph of Section 48.

The insurer has two years from the date of issuance of the insurance contract or of its last reinstatement within which to
contest the policy, whether or not, the insured still lives within such period. After two years, the defenses of concealment or
misrepresentation, no matter how patent or well founded, no longer lie. Congress felt this was a sufficient answer to the
various tactics employed by insurance companies to avoid liability. The petitioners' interpretation would give rise to the
incongruous situation where the beneficiaries of an insured who dies right after taking out and paying for a life insurance
policy, would be allowed to collect on the policy even if the insured fraudulently concealed material facts.

The petitioners argue that no evidence was presented to show that the medical terms were explained in a layman's language
to the insured. They state that the insurer should have presented its two medical field examiners as witnesses. Moreover, the
petitioners allege that the policy intends that the medical examination must be conducted before its issuance otherwise the
insurer "waives whatever imperfection by ratification."

We agree with the Court of Appeals which ruled:

On the other hand, petitioners argue that no evidence was presented by respondent company to show that
the questions appearing in Part II of the application for insurance were asked, explained to and understood
by the deceased so as to prove concealment on his part. The same is not well taken. The deceased, by
affixing his signature on the application form, affirmed the correctness of all the entries and answers
appearing therein. It is but to be expected that he, a businessman, would not have affixed his signature on
the application form unless he clearly understood its significance. For, the presumption is that a person
intends the ordinary consequence of his voluntary act and takes ordinary care of his concerns. [Sec. 5(c)
and (d), Rule 131, Rules of Court].

The evidence for respondent company shows that on September 19,1972, the deceased was examined by
Dr. Victoriano Lim and was found to be diabetic and hypertensive; that by January, 1973, the deceased was
complaining of progressive weight loss and abdominal pain and was diagnosed to be suffering from
hepatoma, (t.s.n. August 23, 1976, pp. 8-10; Exhibit 2). Another physician, Dr. Wenceslao Vitug, testified
that the deceased came to see him on December 14, 1973 for consolation and claimed to have been
diabetic for five years. (t.s.n., Aug. 23,1976, p. 5; Exhibit 6) Because of the concealment made by the
deceased of his consultations and treatments for hypertension, diabetes and liver disorders, respondent
company was thus misled into accepting the risk and approving his application as medically standard
(Exhibit 5- C) and dispensing with further medical investigation and examination (Exhibit 5-A). For as long
as no adverse medical history is revealed in the application form, an applicant for insurance is presumed to
be healthy and physically fit and no further medical investigation or examination is conducted by respondent
company. (t.s.n., April 8,1976, pp. 6-8). (Rollo, pp. 96-98)

There is no strong showing that we should apply the "fine print" or "contract of adhesion" rule in this case. (Sweet Lines, Inc.
v. Teves, 83 SCRA 361 [1978]). The petitioners cite:

It is a matter of common knowledge that large amounts of money are collected from ignorant persons by
companies and associations which adopt high sounding titles and print the amount of benefits they agree to
pay in large black-faced type, following such undertakings by fine print conditions which destroy the
substance of the promise. All provisions, conditions, or exceptions which in any way tend to work a
forfeiture of the policy should be construed most strongly against those for whose benefit they are inserted,
and most favorably toward those against whom they are meant to operate. (Trinidad v. Orient Protective
Assurance Assn., 67 Phil. 184)
There is no showing that the questions in the application form for insurance regarding the insured's medical history are in
smaller print than the rest of the printed form or that they are designed in such a way as to conceal from the applicant their
importance. If a warning in bold red letters or a boxed warning similar to that required for cigarette advertisements by the
Surgeon General of the United States is necessary, that is for Congress or the Insurance Commission to provide as
protection against high pressure insurance salesmanship. We are limited in this petition to ascertaining whether or not the
respondent Court of Appeals committed reversible error. It is the petitioners' burden to show that the factual findings of the
respondent court are not based on substantial evidence or that its conclusions are contrary to applicable law and
jurisprudence. They have failed to discharge that burden.

WHEREFORE, the petition is hereby DENIED for lack of merit. The questioned decision of the Court of Appeals is
AFFIRMED.

SO ORDERED.

Fernan, (C.J., Chairman), Bidin and Cortes, JJ., concur.

Feliciano, took no part.

G.R. No. 151890 June 20, 2006

PRUDENTIAL GUARANTEE and ASSURANCE INC., petitioner,


vs.
TRANS-ASIA SHIPPING LINES, INC., Respondent.

x- - - - - - - - - - - - - - - - - - - - - - - - - x

G.R. No. 151991 June 20, 2006


TRANS-ASIA SHIPPING LINES, INC., petitioner,
vs.
PRUDENTIAL GUARANTEE and ASSURANCE INC., Respondent.

DECISION

CHICO-NAZARIO, J:

This is a consolidation of two separate Petitions for Review on Certiorari filed by petitioner Prudential Guarantee and
Assurance, Inc. (PRUDENTIAL) in G.R. No. 151890 and Trans-Asia Shipping Lines, Inc. (TRANS-ASIA) in G.R. No. 151991,
assailing the Decision1 dated 6 November 2001 of the Court of Appeals in CA G.R. CV No. 68278, which reversed the
Judgment2 dated 6 June 2000 of the Regional Trial Court (RTC), Branch 13, Cebu City in Civil Case No. CEB-20709. The 29
January 2002 Resolution3 of the Court of Appeals, denying PRUDENTIALs Motion for Reconsideration and TRANS-ASIAs
Partial Motion for Reconsideration of the 6 November 2001 Decision, is likewise sought to be annulled and set aside.

The Facts

The material antecedents as found by the court a quo and adopted by the appellate court are as follows:

Plaintiff [TRANS-ASIA] is the owner of the vessel M/V Asia Korea. In consideration of payment of premiums, defendant
[PRUDENTIAL] insured M/V Asia Korea for loss/damage of the hull and machinery arising from perils, inter alia, of fire and
explosion for the sum of P40 Million, beginning [from] the period [of] July 1, 1993 up to July 1, 1994. This is evidenced by
Marine Policy No. MH93/1363 (Exhibits "A" to "A-11"). On October 25, 1993, while the policy was in force, a fire broke out
while [M/V Asia Korea was] undergoing repairs at the port of Cebu. On October 26, 1993 plaintiff [TRANS-ASIA] filed its
notice of claim for damage sustained by the vessel. This is evidenced by a letter/formal claim of even date (Exhibit "B").
Plaintiff [TRANS-ASIA] reserved its right to subsequently notify defendant [PRUDENTIAL] as to the full amount of the claim
upon final survey and determination by average adjuster Richard Hogg International (Phil.) of the damage sustained by
reason of fire. An adjusters report on the fire in question was submitted by Richard Hogg International together with the U-
Marine Surveyor Report (Exhibits "4" to "4-115").

On May 29, 1995[,] plaintiff [TRANS-ASIA] executed a document denominated "Loan and Trust receipt", a portion of which
read (sic):

"Received from Prudential Guarantee and Assurance, Inc., the sum of PESOS THREE MILLION ONLY (P3,000,000.00) as a
loan without interest under Policy No. MH 93/1353 [sic], repayable only in the event and to the extent that any net recovery is
made by Trans-Asia Shipping Corporation, from any person or persons, corporation or corporations, or other parties, on
account of loss by any casualty for which they may be liable occasioned by the 25 October 1993: Fire on Board." (Exhibit "4")

In a letter dated 21 April 1997 defendant [PRUDENTIAL] denied plaintiffs claim (Exhibit "5"). The letter reads:

"After a careful review and evaluation of your claim arising from the above-captioned incident, it has been ascertained that
you are in breach of policy conditions, among them "WARRANTED VESSEL CLASSED AND CLASS MAINTAINED".
Accordingly, we regret to advise that your claim is not compensable and hereby DENIED."

This was followed by defendants letter dated 21 July 1997 requesting the return or payment of the P3,000,000.00 within a
period of ten (10) days from receipt of the letter (Exhibit "6").4

Following this development, on 13 August 1997, TRANS-ASIA filed a Complaint 5 for Sum of Money against PRUDENTIAL
with the RTC of Cebu City, docketed as Civil Case No. CEB-20709, wherein TRANS-ASIA sought the amount of
P8,395,072.26 from PRUDENTIAL, alleging that the same represents the balance of the indemnity due upon the insurance
policy in the total amount of P11,395,072.26. TRANS-ASIA similarly sought interest at 42% per annum citing Section 243 6 of
Presidential Decreee No. 1460, otherwise known as the "Insurance Code," as amended.

In its Answer,7 PRUDENTIAL denied the material allegations of the Complaint and interposed the defense that TRANS-ASIA
breached insurance policy conditions, in particular: "WARRANTED VESSEL CLASSED AND CLASS MAINTAINED."
PRUDENTIAL further alleged that it acted as facts and law require and incurred no liability to TRANS-ASIA; that TRANS-
ASIA has no cause of action; and, that its claim has been effectively waived and/or abandoned, or it is estopped from
pursuing the same. By way of a counterclaim, PRUDENTIAL sought a refund of P3,000,000.00, which it allegedly advanced
to TRANS-ASIA by way of a loan without interest and without prejudice to the final evaluation of the claim, including the
amounts of P500,000.00, for survey fees and P200,000.00, representing attorneys fees.
The Ruling of the Trial Court

On 6 June 2000, the court a quo rendered Judgment 8 finding for (therein defendant) PRUDENTIAL. It ruled that a
determination of the parties liabilities hinged on whether TRANS-ASIA violated and breached the policy conditions on
WARRANTED VESSEL CLASSED AND CLASS MAINTAINED. It interpreted the provision to mean that TRANS-ASIA is
required to maintain the vessel at a certain class at all times pertinent during the life of the policy. According to the court a
quo, TRANS-ASIA failed to prove compliance of the terms of the warranty, the violation thereof entitled PRUDENTIAL, the
insured party, to rescind the contract.9

Further, citing Section 10710 of the Insurance Code, the court a quo ratiocinated that the concealment made by TRANS-ASIA
that the vessel was not adequately maintained to preserve its class was a material concealment sufficient to avoid the policy
and, thus, entitled the injured party to rescind the contract. The court a quo found merit in PRUDENTIALs contention that
there was nothing in the adjustment of the particular average submitted by the adjuster that would show that TRANS-ASIA
was not in breach of the policy. Ruling on the denominated loan and trust receipt, the court a quo said that in substance and
in form, the same is a receipt for a loan. It held that if TRANS-ASIA intended to receive the amount of P3,000,000.00 as
advance payment, it should have so clearly stated as such.

The court a quo did not award PRUDENTIALs claim for P500,000.00, representing expert survey fees on the ground of lack
of sufficient basis in support thereof. Neither did it award attorneys fees on the rationalization that the instant case does not
fall under the exceptions stated in Article 220811 of the Civil Code. However, the court a quo granted PRUDENTIALs
counterclaim stating that there is factual and legal basis for TRANS-ASIA to return the amount of P3,000,000.00 by way of
loan without interest.

The decretal portion of the Judgment of the RTC reads:

WHEREFORE, judgment is hereby rendered DISMISSING the complaint for its failure to prove a cause of action.

On defendants counterclaim, plaintiff is directed to return the sum of P3,000,000.00 representing the loan extended to it by
the defendant, within a period of ten (10) days from and after this judgment shall have become final and executory. 12

The Ruling of the Court of Appeals

On appeal by TRANS-ASIA, the Court of Appeals, in its assailed Decision of 6 November 2001, reversed the 6 June 2000
Judgment of the RTC.

On the issue of TRANS-ASIAs alleged breach of warranty of the policy condition CLASSED AND CLASS MAINTAINED, the
Court of Appeals ruled that PRUDENTIAL, as the party asserting the non-compensability of the loss had the burden of proof
to show that TRANS-ASIA breached the warranty, which burden it failed to discharge. PRUDENTIAL cannot rely on the lack
of certification to the effect that TRANS-ASIA was CLASSED AND CLASS MAINTAINED as its sole basis for reaching the
conclusion that the warranty was breached. The Court of Appeals opined that the lack of a certification does not necessarily
mean that the warranty was breached by TRANS-ASIA. Instead, the Court of Appeals considered PRUDENTIALs admission
that at the time the insurance contract was entered into between the parties, the vessel was properly classed by Bureau
Veritas, a classification society recognized by the industry. The Court of Appeals similarly gave weight to the fact that it was
the responsibility of Richards Hogg International (Phils.) Inc., the average adjuster hired by PRUDENTIAL, to secure a copy
of such certification to support its conclusion that mere absence of a certification does not warrant denial of TRANS-ASIAs
claim under the insurance policy.

In the same token, the Court of Appeals found the subject warranty allegedly breached by TRANS-ASIA to be a rider which,
while contained in the policy, was inserted by PRUDENTIAL without the intervention of TRANS-ASIA. As such, it partakes of
a nature of a contract dadhesion which should be construed against PRUDENTIAL, the party which drafted the contract.
Likewise, according to the Court of Appeals, PRUDENTIALs renewal of the insurance policy from noon of 1 July 1994 to
noon of 1 July 1995, and then again, until noon of 1 July 1996 must be deemed a waiver by PRUDENTIAL of any breach of
warranty committed by TRANS-ASIA.

Further, the Court of Appeals, contrary to the ruling of the court a quo, interpreted the transaction between PRUDENTIAL and
TRANS-ASIA as one of subrogation, instead of a loan. The Court of Appeals concluded that TRANS-ASIA has no obligation
to pay back the amount of P3,000.000.00 to PRUDENTIAL based on its finding that the aforesaid amount was
PRUDENTIALs partial payment to TRANS-ASIAs claim under the policy. Finally, the Court of Appeals denied TRANS-ASIAs
prayer for attorneys fees, but held TRANS-ASIA entitled to double interest on the policy for the duration of the delay of
payment of the unpaid balance, citing Section 24413 of the Insurance Code.
Finding for therein appellant TRANS-ASIA, the Court of Appeals ruled in this wise:

WHEREFORE, the foregoing consideration, We find for Appellant. The instant appeal is ALLOWED and the Judgment
appealed from REVERSED. The P3,000,000.00 initially paid by appellee Prudential Guarantee Assurance Incorporated to
appellant Trans-Asia and covered by a "Loan and Trust Receipt" dated 29 May 1995 is HELD to be in partial settlement of
the loss suffered by appellant and covered by Marine Policy No. MH93/1363 issued by appellee. Further, appellee is hereby
ORDERED to pay appellant the additional amount of P8,395,072.26 representing the balance of the loss suffered by the
latter as recommended by the average adjuster Richard Hogg International (Philippines) in its Report, with double interest
starting from the time Richard Hoggs Survey Report was completed, or on 13 August 1996, until the same is fully paid.

All other claims and counterclaims are hereby DISMISSED.

All costs against appellee.14

Not satisfied with the judgment, PRUDENTIAL and TRANS-ASIA filed a Motion for Reconsideration and Partial Motion for
Reconsideration thereon, respectively, which motions were denied by the Court of Appeals in the Resolution dated 29
January 2002.

The Issues

Aggrieved, PRUDENTIAL filed before this Court a Petition for Review, docketed as G.R. No. 151890, relying on the following
grounds, viz:

I.

THE AWARD IS GROSSLY UNCONSCIONABLE.

II.

THE COURT OF APPEALS ERRED IN HOLDING THAT THERE WAS NO VIOLATION BY TRANS-ASIA OF A MATERIAL
WARRANTY, NAMELY, WARRANTY CLAUSE NO. 5, OF THE INSURANCE POLICY.

III.

THE COURT OF APPEALS ERRED IN HOLDING THAT PRUDENTIAL, AS INSURER HAD THE BURDEN OF PROVING
THAT THE ASSURED, TRANS-ASIA, VIOLATED A MATERIAL WARRANTY.

IV.

THE COURT OF APPEALS ERRED IN HOLDING THAT THE WARRANTY CLAUSE EMBODIED IN THE INSURANCE
POLICY CONTRACT WAS A MERE RIDER.

V.

THE COURT OF APPEALS ERRED IN HOLDING THAT THE ALLEGED RENEWALS OF THE POLICY CONSTITUTED A
WAIVER ON THE PART OF PRUDENTIAL OF THE BREACH OF THE WARRANTY BY TRANS-ASIA.

VI.

THE COURT OF APPEALS ERRED IN HOLDING THAT THE "LOAN AND TRUST RECEIPT" EXECUTED BY TRANS-ASIA
IS AN ADVANCE ON THE POLICY, THUS CONSTITUTING PARTIAL PAYMENT THEREOF.

VII.

THE COURT OF APPEALS ERRED IN HOLDING THAT THE ACCEPTANCE BY PRUDENTIAL OF THE FINDINGS OF
RICHARDS HOGG IS INDICATIVE OF A WAIVER ON THE PART OF PRUDENTIAL OF ANY VIOLATION BY TRANS-ASIA
OF THE WARRANTY.
VIII.

THE COURT OF APPEALS ERRRED (sic) IN REVERSING THE TRIAL COURT, IN FINDING THAT PRUDENTIAL
"UNJUSTIFIABLY REFUSED" TO PAY THE CLAIM AND IN ORDERING PRUDENTIAL TO PAY TRANS-ASIA P8,395,072.26
PLUS DOUBLE INTEREST FROM 13 AUGUST 1996, UNTIL [THE] SAME IS FULLY PAID. 15

Similarly, TRANS-ASIA, disagreeing in the ruling of the Court of Appeals filed a Petition for Review docketed as G.R. No.
151991, raising the following grounds for the allowance of the petition, to wit:

I.

THE HONORABLE COURT OF APPEALS ERRED IN NOT AWARDING ATTORNEYS FEES TO PETITIONER TRANS-
ASIA ON THE GROUND THAT SUCH CAN ONLY BE AWARDED IN THE CASES ENUMERATED IN ARTICLE 2208 OF
THE CIVIL CODE, AND THERE BEING NO BAD FAITH ON THE PART OF RESPONDENT PRUDENTIAL IN DENYING
HEREIN PETITIONER TRANS-ASIAS INSURANCE CLAIM.

II.

THE "DOUBLE INTEREST" REFERRED TO IN THE DECISION DATED 06 NOVEMBER 2001 SHOULD BE CONSTRUED
TO MEAN DOUBLE INTEREST BASED ON THE LEGAL INTEREST OF 12%, OR INTEREST AT THE RATE OF 24% PER
ANNUM.16

In our Resolution of 2 December 2002, we granted TRANS-ASIAs Motion for Consolidation 17 of G.R. Nos. 151890 and
151991;18 hence, the instant consolidated petitions.

In sum, for our main resolution are: (1) the liability, if any, of PRUDENTIAL to TRANS-ASIA arising from the subject insurance
contract; (2) the liability, if any, of TRANS-ASIA to PRUDENTIAL arising from the transaction between the parties as
evidenced by a document denominated as "Loan and Trust Receipt," dated 29 May 1995; and (3) the amount of interest to
be imposed on the liability, if any, of either or both parties.

Ruling of the Court

Prefatorily, it must be emphasized that in a petition for review, only questions of law, and not questions of fact, may be
raised.19 This rule may be disregarded only when the findings of fact of the Court of Appeals are contrary to the findings and
conclusions of the trial court, or are not supported by the evidence on record. 20 In the case at bar, we find an incongruence
between the findings of fact of the Court of Appeals and the court a quo, thus, in our determination of the issues, we are
constrained to assess the evidence adduced by the parties to make appropriate findings of facts as are necessary.

I.

A. PRUDENTIAL failed to establish that TRANS-ASIA violated and breached the policy condition on WARRANTED
VESSEL CLASSED AND CLASS MAINTAINED, as contained in the subject insurance contract.

In resisting the claim of TRANS-ASIA, PRUDENTIAL posits that TRANS-ASIA violated an express and material warranty in
the subject insurance contract, i.e., Marine Insurance Policy No. MH93/1363, specifically Warranty Clause No. 5 thereof,
which stipulates that the insured vessel, "M/V ASIA KOREA" is required to be CLASSED AND CLASS MAINTAINED.
According to PRUDENTIAL, on 25 October 1993, or at the time of the occurrence of the fire, "M/V ASIA KOREA" was in
violation of the warranty as it was not CLASSED AND CLASS MAINTAINED. PRUDENTIAL submits that Warranty Clause
No. 5 was a condition precedent to the recovery of TRANS-ASIA under the policy, the violation of which entitled
PRUDENTIAL to rescind the contract under Sec. 7421 of the Insurance Code.

The warranty condition CLASSED AND CLASS MAINTAINED was explained by PRUDENTIALs Senior Manager of the
Marine and Aviation Division, Lucio Fernandez. The pertinent portions of his testimony on direct examination is reproduced
hereunder, viz:

ATTY. LIM

Q Please tell the court, Mr. Witness, the result of the evaluation of this claim, what final action was taken?
A It was eventually determined that there was a breach of the policy condition, and basically there is a breach of policy
warranty condition and on that basis the claim was denied.

Q To refer you (sic) the "policy warranty condition," I am showing to you a policy here marked as Exhibits "1", "1-A" series,
please point to the warranty in the policy which you said was breached or violated by the plaintiff which constituted your basis
for denying the claim as you testified.

A Warranted Vessel Classed and Class Maintained.

ATTY. LIM

Witness pointing, Your Honor, to that portion in Exhibit "1-A" which is the second page of the policy below the printed words:
"Clauses, Endorsements, Special Conditions and Warranties," below this are several typewritten clauses and the witness
pointed out in particular the clause reading: "Warranted Vessel Classed and Class Maintained."

COURT

Q Will you explain that particular phrase?

A Yes, a warranty is a condition that has to be complied with by the insured. When we say a class warranty, it must be
entered in the classification society.

COURT

Slowly.

WITNESS

(continued)

A A classification society is an organization which sets certain standards for a vessel to maintain in order to maintain their
membership in the classification society. So, if they failed to meet that standard, they are considered not members of that
class, and thus breaching the warranty, that requires them to maintain membership or to maintain their class on that
classification society. And it is not sufficient that the member of this classification society at the time of a loss, their
membership must be continuous for the whole length of the policy such that during the effectivity of the policy, their
classification is suspended, and then thereafter, they get reinstated, that again still a breach of the warranty that they
maintained their class (sic). Our maintaining team membership in the classification society thereby maintaining the standards
of the vessel (sic).

ATTY. LIM

Q Can you mention some classification societies that you know?

A Well we have the Bureau Veritas, American Bureau of Shipping, D&V Local Classification Society, The Philippine
Registration of Ships Society, China Classification, NKK and Company Classification Society, and many others, we have
among others, there are over 20 worldwide. 22

At the outset, it must be emphasized that the party which alleges a fact as a matter of defense has the burden of proving it.
PRUDENTIAL, as the party which asserted the claim that TRANS-ASIA breached the warranty in the policy, has the burden
of evidence to establish the same. Hence, on the part of PRUDENTIAL lies the initiative to show proof in support of its
defense; otherwise, failing to establish the same, it remains self-serving. Clearly, if no evidence on the alleged breach of
TRANS-ASIA of the subject warranty is shown, a fortiori, TRANS-ASIA would be successful in claiming on the policy. It
follows that PRUDENTIAL bears the burden of evidence to establish the fact of breach.

In our rule on evidence, TRANS-ASIA, as the plaintiff below, necessarily has the burden of proof to show proof of loss, and
the coverage thereof, in the subject insurance policy. However, in the course of trial in a civil case, once plaintiff makes out a
prima facie case in his favor, the duty or the burden of evidence shifts to defendant to controvert plaintiffs prima facie case,
otherwise, a verdict must be returned in favor of plaintiff.23 TRANS-ASIA was able to establish proof of loss and the coverage
of the loss, i.e., 25 October 1993: Fire on Board. Thereafter, the burden of evidence shifted to PRUDENTIAL to counter
TRANS-ASIAs case, and to prove its special and affirmative defense that TRANS-ASIA was in violation of the particular
condition on CLASSED AND CLASS MAINTAINED.

We sustain the findings of the Court of Appeals that PRUDENTIAL was not successful in discharging the burden of evidence
that TRANS-ASIA breached the subject policy condition on CLASSED AND CLASS MAINTAINED.

Foremost, PRUDENTIAL, through the Senior Manager of its Marine and Aviation Division, Lucio Fernandez, made a
categorical admission that at the time of the procurement of the insurance contract in July 1993, TRANS-ASIAs vessel, "M/V
Asia Korea" was properly classed by Bureau Veritas, thus:

Q Kindly examine the records particularly the policy, please tell us if you know whether M/V Asia Korea was classed at the
time (sic) policy was procured perthe (sic) insurance was procured that Exhibit "1" on 1st July 1993 (sic).

WITNESS

A I recall that they were classed.

ATTY. LIM

Q With what classification society?

A I believe with Bureau Veritas.24

As found by the Court of Appeals and as supported by the records, Bureau Veritas is a classification society recognized in
the marine industry. As it is undisputed that TRANS-ASIA was properly classed at the time the contract of insurance was
entered into, thus, it becomes incumbent upon PRUDENTIAL to show evidence that the status of TRANS-ASIA as being
properly CLASSED by Bureau Veritas had shifted in violation of the warranty. Unfortunately, PRUDENTIAL failed to support
the allegation.

We are in accord with the ruling of the Court of Appeals that the lack of a certification in PRUDENTIALs records to the effect
that TRANS-ASIAs "M/V Asia Korea" was CLASSED AND CLASS MAINTAINED at the time of the occurrence of the fire
cannot be tantamount to the conclusion that TRANS-ASIA in fact breached the warranty contained in the policy. With more
reason must we sustain the findings of the Court of Appeals on the ground that as admitted by PRUDENTIAL, it was likewise
the responsibility of the average adjuster, Richards Hogg International (Phils.), Inc., to secure a copy of such certification,
and the alleged breach of TRANS-ASIA cannot be gleaned from the average adjusters survey report, or adjustment of
particular average per "M/V Asia Korea" of the 25 October 1993 fire on board.

We are not unmindful of the clear language of Sec. 74 of the Insurance Code which provides that, "the violation of a material
warranty, or other material provision of a policy on the part of either party thereto, entitles the other to rescind." It is generally
accepted that "[a] warranty is a statement or promise set forth in the policy, or by reference incorporated therein, the untruth
or non-fulfillment of which in any respect, and without reference to whether the insurer was in fact prejudiced by such untruth
or non-fulfillment, renders the policy voidable by the insurer." 25 However, it is similarly indubitable that for the breach of a
warranty to avoid a policy, the same must be duly shown by the party alleging the same. We cannot sustain an allegation that
is unfounded. Consequently, PRUDENTIAL, not having shown that TRANS-ASIA breached the warranty condition,
CLASSED AND CLASS MAINTAINED, it remains that TRANS-ASIA must be allowed to recover its rightful claims on the
policy.

B. Assuming arguendo that TRANS-ASIA violated the policy condition on WARRANTED VESSEL CLASSED AND CLASS
MAINTAINED, PRUDENTIAL made a valid waiver of the same.

The Court of Appeals, in reversing the Judgment of the RTC which held that TRANS-ASIA breached the warranty provision
on CLASSED AND CLASS MAINTAINED, underscored that PRUDENTIAL can be deemed to have made a valid waiver of
TRANS-ASIAs breach of warranty as alleged, ratiocinating, thus:

Third, after the loss, Prudential renewed the insurance policy of Trans-Asia for two (2) consecutive years, from noon of 01
July 1994 to noon of 01 July 1995, and then again until noon of 01 July 1996. This renewal is deemed a waiver of any breach
of warranty.26

PRUDENTIAL finds fault with the ruling of the appellate court when it ruled that the renewal policies are deemed a waiver of
TRANS-ASIAs alleged breach, averring herein that the subsequent policies, designated as MH94/1595 and MH95/1788
show that they were issued only on 1 July 1994 and 3 July 1995, respectively, prior to the time it made a request to TRANS-
ASIA that it be furnished a copy of the certification specifying that the insured vessel "M/V Asia Korea" was CLASSED AND
CLASS MAINTAINED. PRUDENTIAL posits that it came to know of the breach by TRANS-ASIA of the subject warranty
clause only on 21 April 1997. On even date, PRUDENTIAL sent TRANS-ASIA a letter of denial, advising the latter that their
claim is not compensable. In fine, PRUDENTIAL would have this Court believe that the issuance of the renewal policies
cannot be a waiver because they were issued without knowledge of the alleged breach of warranty committed by TRANS-
ASIA.27

We are not impressed. We do not find that the Court of Appeals was in error when it held that PRUDENTIAL, in renewing
TRANS-ASIAs insurance policy for two consecutive years after the loss covered by Policy No. MH93/1363, was considered
to have waived TRANS-ASIAs breach of the subject warranty, if any. Breach of a warranty or of a condition renders the
contract defeasible at the option of the insurer; but if he so elects, he may waive his privilege and power to rescind by the
mere expression of an intention so to do. In that event his liability under the policy continues as before. 28 There can be no
clearer intention of the waiver of the alleged breach than the renewal of the policy insurance granted by PRUDENTIAL to
TRANS-ASIA in MH94/1595 and MH95/1788, issued in the years 1994 and 1995, respectively.

To our mind, the argument is made even more credulous by PRUDENTIALs lack of proof to support its allegation that the
renewals of the policies were taken only after a request was made to TRANS-ASIA to furnish them a copy of the certificate
attesting that "M/V Asia Korea" was CLASSED AND CLASS MAINTAINED. Notwithstanding PRUDENTIALs claim that no
certification was issued to that effect, it renewed the policy, thereby, evidencing an intention to waive TRANS-ASIAs alleged
breach. Clearly, by granting the renewal policies twice and successively after the loss, the intent was to benefit the insured,
TRANS-ASIA, as well as to waive compliance of the warranty.

The foregoing finding renders a determination of whether the subject warranty is a rider, moot, as raised by the
PRUDENTIAL in its assignment of errors. Whether it is a rider will not effectively alter the result for the reasons that: (1)
PRUDENTIAL was not able to discharge the burden of evidence to show that TRANS-ASIA committed a breach, thereof; and
(2) assuming arguendo the commission of a breach by TRANS-ASIA, the same was shown to have been waived by
PRUDENTIAL.

II.

A. The amount of P3,000,000.00 granted by PRUDENTIAL to TRANS- ASIA via a transaction between the parties evidenced
by a document denominated as "Loan and Trust Receipt," dated 29 May 1995 constituted partial payment on the policy.

It is undisputed that TRANS-ASIA received from PRUDENTIAL the amount of P3,000,000.00. The same was evidenced by a
transaction receipt denominated as a "Loan and Trust Receipt," dated 29 May 1995, reproduced hereunder:

LOAN AND TRUST RECEIPT

Claim File No. MH-93-025 May 29, 1995


P3,000,000.00
Check No. PCIB066755

Received FROM PRUDENTIAL GUARANTEE AND ASSURANCE INC., the sum of PESOS THREE MILLION ONLY
(P3,000,000.00) as a loan without interest, under Policy No. MH93/1353, repayable only in the event and to the extent that
any net recovery is made by TRANS ASIA SHIPPING CORP., from any person or persons, corporation or corporations, or
other parties, on account of loss by any casualty for which they may be liable, occasioned by the 25 October 1993: Fire on
Board.

As security for such repayment, we hereby pledge to PRUDENTIAL GUARANTEE AND ASSURANCE INC. whatever
recovery we may make and deliver to it all documents necessary to prove our interest in said property. We also hereby agree
to promptly prosecute suit against such persons, corporation or corporations through whose negligence the aforesaid loss
was caused or who may otherwise be responsible therefore, with all due diligence, in our own name, but at the expense of
and under the exclusive direction and control of PRUDENTIAL GUARANTEE AND ASSURANCE INC.

TRANS-ASIA SHIPPING CORPORATION29

PRUDENTIAL largely contends that the "Loan and Trust Receipt" executed by the parties evidenced a loan of P3,000,000.00
which it granted to TRANS-ASIA, and not an advance payment on the policy or a partial payment for the loss. It further
submits that it is a customary practice for insurance companies in this country to extend loans gratuitously as part of good
business dealing with their assured, in order to afford their assured the chance to continue business without embarrassment
while awaiting outcome of the settlement of their claims.30According to PRUDENTIAL, the "Trust and Loan Agreement" did
not subrogate to it whatever rights and/or actions TRANS-ASIA may have against third persons, and it cannot by no means
be taken that by virtue thereof, PRUDENTIAL was granted irrevocable power of attorney by TRANS-ASIA, as the sole power
to prosecute lies solely with the latter.

The Court of Appeals held that the real character of the transaction between the parties as evidenced by the "Loan and Trust
Receipt" is that of an advance payment by PRUDENTIAL of TRANS-ASIAs claim on the insurance, thus:

The Philippine Insurance Code (PD 1460 as amended) was derived from the old Insurance Law Act No. 2427 of the
Philippine Legislature during the American Regime. The Insurance Act was lifted verbatim from the law of California, except
Chapter V thereof, which was taken largely from the insurance law of New York. Therefore, ruling case law in that jurisdiction
is to Us persuasive in interpreting provisions of our own Insurance Code. In addition, the application of the adopted statute
should correspond in fundamental points with the application in its country of origin x x x.

xxxx

Likewise, it is settled in that jurisdiction that the (sic) notwithstanding recitals in the Loan Receipt that the money was
intended as a loan does not detract from its real character as payment of claim, thus:

"The receipt of money by the insured employers from a surety company for losses on account of forgery of drafts by an
employee where no provision or repayment of the money was made except upon condition that it be recovered from other
parties and neither interest nor security for the asserted debts was provided for, the money constituted the payment of a
liability and not a mere loan, notwithstanding recitals in the written receipt that the money was intended as a mere loan."

What is clear from the wordings of the so-called "Loan and Trust Receipt Agreement" is that appellant is obligated to hand
over to appellee "whatever recovery (Trans Asia) may make and deliver to (Prudential) all documents necessary to prove its
interest in the said property." For all intents and purposes therefore, the money receipted is payment under the policy, with
Prudential having the right of subrogation to whatever net recovery Trans-Asia may obtain from third parties resulting from
the fire. In the law on insurance, subrogation is an equitable assignment to the insurer of all remedies which the insured may
have against third person whose negligence or wrongful act caused the loss covered by the insurance policy, which is
created as the legal effect of payment by the insurer as an assignee in equity. The loss in the first instance is that of the
insured but after reimbursement or compensation, it becomes the loss of the insurer. It has been referred to as the doctrine
of substitution and rests on the principle that substantial justice should be attained regardless of form, that is, its basis is the
doing of complete, essential, and perfect justice between all the parties without regard to form. 31

We agree. Notwithstanding its designation, the tenor of the "Loan and Trust Receipt" evidences that the real nature of the
transaction between the parties was that the amount of P3,000,000.00 was not intended as a loan whereby TRANS-ASIA is
obligated to pay PRUDENTIAL, but rather, the same was a partial payment or an advance on the policy of the claims due to
TRANS-ASIA.

First, the amount of P3,000,000.00 constitutes an advance payment to TRANS-ASIA by PRUDENTIAL, subrogating the
former to the extent of "any net recovery made by TRANS ASIA SHIPPING CORP., from any person or persons, corporation
or corporations, or other parties, on account of loss by any casualty for which they may be liable, occasioned by the 25
October 1993: Fire on Board." 32

Second, we find that per the "Loan and Trust Receipt," even as TRANS-ASIA agreed to "promptly prosecute suit against
such persons, corporation or corporations through whose negligence the aforesaid loss was caused or who may otherwise
be responsible therefore, with all due diligence" in its name, the prosecution of the claims against such third persons are to
be carried on "at the expense of and under the exclusive direction and control of PRUDENTIAL GUARANTEE AND
ASSURANCE INC."33 The clear import of the phrase "at the expense of and under the exclusive direction and control" as
used in the "Loan and Trust Receipt" grants solely to PRUDENTIAL the power to prosecute, even as the same is carried in
the name of TRANS-ASIA, thereby making TRANS-ASIA merely an agent of PRUDENTIAL, the principal, in the prosecution
of the suit against parties who may have occasioned the loss.

Third, per the subject "Loan and Trust Receipt," the obligation of TRANS-ASIA to repay PRUDENTIAL is highly speculative
and contingent, i.e., only in the event and to the extent that any net recovery is made by TRANS-ASIA from any person on
account of loss occasioned by the fire of 25 October 1993. The transaction, therefore, was made to benefit TRANS-ASIA,
such that, if no recovery from third parties is made, PRUDENTIAL cannot be repaid the amount. Verily, we do not think that
this is constitutive of a loan.34 The liberality in the tenor of the "Loan and Trust Receipt" in favor of TRANS-ASIA leads to the
conclusion that the amount of P3,000,000.00 was a form of an advance payment on TRANS-ASIAs claim on MH93/1353.
III.

A. PRUDENTIAL is directed to pay TRANS-ASIA the amount of P8,395,072.26, representing the balance of the loss suffered
by TRANS-ASIA and covered by Marine Policy No. MH93/1363.

Our foregoing discussion supports the conclusion that TRANS-ASIA is entitled to the unpaid claims covered by Marine Policy
No. MH93/1363, or a total amount of P8,395,072.26.

B. Likewise, PRUDENTIAL is directed to pay TRANS-ASIA, damages in the form of attorneys fees equivalent to 10% of
P8,395,072.26.

The Court of Appeals denied the grant of attorneys fees. It held that attorneys fees cannot be awarded absent a showing of
bad faith on the part of PRUDENTIAL in rejecting TRANS-ASIAs claim, notwithstanding that the rejection was erroneous.
According to the Court of Appeals, attorneys fees can be awarded only in the cases enumerated in Article 2208 of the Civil
Code which finds no application in the instant case.

We disagree. Sec. 244 of the Insurance Code grants damages consisting of attorneys fees and other expenses incurred by
the insured after a finding by the Insurance Commissioner or the Court, as the case may be, of an unreasonable denial or
withholding of the payment of the claims due. Moreover, the law imposes an interest of twice the ceiling prescribed by the
Monetary Board on the amount of the claim due the insured from the date following the time prescribed in Section 242 35 or in
Section 243,36 as the case may be, until the claim is fully satisfied. Finally, Section 244 considers the failure to pay the claims
within the time prescribed in Sections 242 or 243, when applicable, as prima facie evidence of unreasonable delay in
payment.

To the mind of this Court, Section 244 does not require a showing of bad faith in order that attorneys fees be granted. As
earlier stated, under Section 244, a prima facie evidence of unreasonable delay in payment of the claim is created by failure
of the insurer to pay the claim within the time fixed in both Sections 242 and 243 of the Insurance Code. As established in
Section 244, by reason of the delay and the consequent filing of the suit by the insured, the insurers shall be adjudged to pay
damages which shall consist of attorneys fees and other expenses incurred by the insured. 37

Section 244 reads:

In case of any litigation for the enforcement of any policy or contract of insurance, it shall be the duty of the Commissioner or
the Court, as the case may be, to make a finding as to whether the payment of the claim of the insured has been
unreasonably denied or withheld; and in the affirmative case, the insurance company shall be adjudged to pay damages
which shall consist of attorneys fees and other expenses incurred by the insured person by reason of such unreasonable
denial or withholding of payment plus interest of twice the ceiling prescribed by the Monetary Board of the amount of the
claim due the insured, from the date following the time prescribed in section two hundred forty-two or in section two hundred
forty-three, as the case may be, until the claim is fully satisfied; Provided, That the failure to pay any such claim within the
time prescribed in said sections shall be considered prima facie evidence of unreasonable delay in payment.

Sections 243 and 244 of the Insurance Code apply when the court finds an unreasonable delay or refusal in the payment of
the insurance claims.

In the case at bar, the facts as found by the Court of Appeals, and confirmed by the records show that there was an
unreasonable delay by PRUDENTIAL in the payment of the unpaid balance of P8,395,072.26 to TRANS-ASIA. On 26
October 1993, a day after the occurrence of the fire in "M/V Asia Korea", TRANS-ASIA filed its notice of claim. On 13 August
1996, the adjuster, Richards Hogg International (Phils.), Inc., completed its survey report recommending the amount of
P11,395,072.26 as the total indemnity due to TRANS-ASIA. 38 On 21 April 1997, PRUDENTIAL, in a letter39 addressed to
TRANS-ASIA denied the latters claim for the amount of P8,395,072.26 representing the balance of the total indemnity. On
21 July 1997, PRUDENTIAL sent a second letter40 to TRANS-ASIA seeking a return of the amount of P3,000,000.00. On 13
August 1997, TRANS-ASIA was constrained to file a complaint for sum of money against PRUDENTIAL praying, inter alia,
for the sum of P8,395,072.26 representing the balance of the proceeds of the insurance claim.

As can be gleaned from the foregoing, there was an unreasonable delay on the part of PRUDENTIAL to pay TRANS-ASIA,
as in fact, it refuted the latters right to the insurance claims, from the time proof of loss was shown and the ascertainment of
the loss was made by the insurance adjuster. Evidently, PRUDENTIALs unreasonable delay in satisfying TRANS-ASIAs
unpaid claims compelled the latter to file a suit for collection.

Succinctly, an award equivalent to ten percent (10%) of the unpaid proceeds of the policy as attorneys fees to TRANS-ASIA
is reasonable under the circumstances, or otherwise stated, ten percent (10%) of P8,395,072.26. In the case of Cathay
Insurance, Co., Inc. v. Court of Appeals,41 where a finding of an unreasonable delay under Section 244 of the Insurance Code
was made by this Court, we grant an award of attorneys fees equivalent to ten percent (10%) of the total proceeds. We find
no reason to deviate from this judicial precedent in the case at bar.

C. Further, the aggregate amount (P8,395,072.26 plus 10% thereof as attorneys fees) shall be imposed double interest in
accordance with Section 244 of the Insurance Code.

Section 244 of the Insurance Code is categorical in imposing an interest twice the ceiling prescribed by the Monetary Board
due the insured, from the date following the time prescribed in Section 242 or in Section 243, as the case may be, until the
claim is fully satisfied. In the case at bar, we find Section 243 to be applicable as what is involved herein is a marine
insurance, clearly, a policy other than life insurance.

Section 243 is hereunder reproduced:

SEC. 243. The amount of any loss or damage for which an insurer may be liable, under any policy other than life insurance
policy, shall be paid within thirty days after proof of loss is received by the insurer and ascertainment of the loss or damage is
made either by agreement between the insured and the insurer or by arbitration; but if such ascertainment is not had or
made within sixty days after such receipt by the insurer of the proof of loss, then the loss or damage shall be paid within
ninety days after such receipt. Refusal or failure to pay the loss or damage within the time prescribed herein will entitle the
assured to collect interest on the proceeds of the policy for the duration of the delay at the rate of twice the ceiling prescribed
by the Monetary Board, unless such failure or refusal to pay is based on the ground that the claim is fraudulent.

As specified, the assured is entitled to interest on the proceeds for the duration of the delay at the rate of twice the ceiling
prescribed by the Monetary Board except when the failure or refusal of the insurer to pay was founded on the ground that the
claim is fraudulent.

D. The term "double interest" as used in the Decision of the Court of Appeals must be interpreted to mean 24% per annum.

PRUDENTIAL assails the award of interest, granted by the Court of Appeals, in favor of TRANS-ASIA in the assailed
Decision of 6 November 2001. It is PRUDENTIALs stance that the award is extortionate and grossly unsconscionable. In
support thereto, PRUDENTIAL makes a reference to TRANS-ASIAs prayer in the Complaint filed with the court a quo
wherein the latter sought, "interest double the prevailing rate of interest of 21% per annum now obtaining in the banking
business or plus 42% per annum pursuant to Article 243 of the Insurance Code x x x." 42

The contention fails to persuade. It is settled that an award of double interest is lawful and justified under Sections 243 and
244 of the Insurance Code.43 In Finman General Assurance Corporation v. Court of Appeals,44 this Court held that the
payment of 24% interest per annum is authorized by the Insurance Code. 45 There is no gainsaying that the term "double
interest" as used in Sections 243 and 244 can only be interpreted to mean twice 12% per annum or 24% per annum interest,
thus:

The term "ceiling prescribed by the Monetary Board" means the legal rate of interest of twelve per centum per annum (12%)
as prescribed by the Monetary Board in C.B. Circular No. 416, pursuant to P.D. No. 116, amending the Usury Law; so that
when Sections 242, 243 and 244 of the Insurance Code provide that the insurer shall be liable to pay interest "twice the
ceiling prescribed by the Monetary Board", it means twice 12% per annum or 24% per annum interest on the proceeds of the
insurance.46

E. The payment of double interest should be counted from 13 September 1996.

The Court of Appeals, in imposing double interest for the duration of the delay of the payment of the unpaid balance due
TRANS-ASIA, computed the same from 13 August 1996 until such time when the amount is fully paid. Although not raised by
the parties, we find the computation of the duration of the delay made by the appellate court to be patently erroneous.

To be sure, Section 243 imposes interest on the proceeds of the policy for the duration of the delay at the rate of twice the
ceiling prescribed by the Monetary Board. Significantly, Section 243 mandates the payment of any loss or damage for which
an insurer may be liable, under any policy other than life insurance policy, within thirty days after proof of loss is received by
the insurer and ascertainment of the loss or damage is made either by agreement between the insured and the insurer or by
arbitration. It is clear that under Section 243, the insurer has until the 30th day after proof of loss and ascertainment of the
loss or damage to pay its liability under the insurance, and only after such time can the insurer be held to be in delay, thereby
necessitating the imposition of double interest.
In the case at bar, it was not disputed that the survey report on the ascertainment of the loss was completed by the adjuster,
Richard Hoggs International (Phils.), Inc. on 13 August 1996. PRUDENTIAL had thirty days from 13 August 1996 within
which to pay its liability to TRANS-ASIA under the insurance policy, or until 13 September 1996. Therefore, the double
interest can begin to run from 13 September 1996 only.

IV.

A. An interest of 12% per annum is similarly imposed on the TOTAL amount of liability adjudged in section III herein,
computed from the time of finality of judgment until the full satisfaction thereof in conformity with this Courts ruling in Eastern
Shipping Lines, Inc. v. Court of Appeals.

This Court in Eastern Shipping Lines, Inc. v. Court of Appeals, 47 inscribed the rule of thumb48 in the application of interest to
be imposed on obligations, regardless of their source. Eastern emphasized beyond cavil that when the judgment of the court
awarding a sum of money becomes final and executory, the rate of legal interest, regardless of whether the obligation
involves a loan or forbearance of money, shall be 12% per annum from such finality until its satisfaction, this interim period
being deemed to be by then an equivalent to a forbearance 49 of credit.

We find application of the rule in the case at bar proper, thus, a rate of 12% per annum from the finality of judgment until the
full satisfaction thereof must be imposed on the total amount of liability adjudged to PRUDENTIAL. It is clear that the interim
period from the finality of judgment until the satisfaction of the same is deemed equivalent to a forbearance of credit, hence,
the imposition of the aforesaid interest.

Fallo

WHEREFORE, the Petition in G.R. No. 151890 is DENIED. However, the Petition in G.R. No. 151991 is GRANTED, thus, we
award the grant of attorneys fees and make a clarification that the term "double interest" as used in the 6 November 2001
Decision of the Court of Appeals in CA GR CV No. 68278 should be construed to mean interest at the rate of 24% per
annum, with a further clarification, that the same should be computed from 13 September 1996 until fully paid. The Decision
and Resolution of the Court of Appeals, in CA-G.R. CV No. 68278, dated 6 November 2001 and 29 January 2002,
respectively, are, thus, MODIFIED in the following manner, to wit:

1. PRUDENTIAL is DIRECTED to PAY TRANS-ASIA the amount of P8,395,072.26, representing the balance of the
loss suffered by TRANS-ASIA and covered by Marine Policy No. MH93/1363;

2. PRUDENTIAL is DIRECTED further to PAY TRANS-ASIA damages in the form of attorneys fees equivalent to
10% of the amount of P8,395,072.26;

3. The aggregate amount (P8,395,072.26 plus 10% thereof as attorneys fees) shall be imposed double interest at
the rate of 24% per annum to be computed from 13 September 1996 until fully paid; and

4. An interest of 12% per annum is similarly imposed on the TOTAL amount of liability adjudged as abovestated in
paragraphs (1), (2), and (3) herein, computed from the time of finality of judgment until the full satisfaction thereof.

No costs.

SO ORDERED.

V. The policy of insurance

G.R. No. L-38613 February 25, 1982

PACIFIC TIMBER EXPORT CORPORATION, petitioner,


vs.
THE HONORABLE COURT OF APPEALS and WORKMEN'S INSURANCE COMPANY, INC., respondents.

DE CASTRO, ** J.:
This petition seeks the review of the decision of the Court of Appeals reversing the decision of the Court of First Instance of
Manila in favor of petitioner and against private respondent which ordered the latter to pay the sum of Pll,042.04 with interest
at the rate of 12% interest from receipt of notice of loss on April 15, 1963 up to the complete payment, the sum of P3,000.00
as attorney's fees and the costs 1 thereby dismissing petitioner s complaint with costs. 2

The findings of the of fact of the Court of Appeals, which are generally binding upon this Court, Except as shall be indicated
in the discussion of the opinion of this Court the substantial correctness of still particular finding having been disputed,
thereby raising a question of law reviewable by this Court 3 are as follows:

March 19, l963, the plaintiff secured temporary insurance from the defendant for its exportation of 1,250,000
board feet of Philippine Lauan and Apitong logs to be shipped from the Diapitan. Bay, Quezon Province to
Okinawa and Tokyo, Japan. The defendant issued on said date Cover Note No. 1010, insuring the said
cargo of the plaintiff "Subject to the Terms and Conditions of the WORKMEN'S INSURANCE COMPANY,
INC. printed Marine Policy form as filed with and approved by the Office of the Insurance Commissioner
(Exhibit A).

The regular marine cargo policies were issued by the defendant in favor of the plaintiff on April 2, 1963. The
two marine policies bore the numbers 53 HO 1032 and 53 HO 1033 (Exhibits B and C, respectively). Policy
No. 53 H0 1033 (Exhibit B) was for 542 pieces of logs equivalent to 499,950 board feet. Policy No. 53 H0
1033 was for 853 pieces of logs equivalent to 695,548 board feet (Exhibit C). The total cargo insured under
the two marine policies accordingly consisted of 1,395 logs, or the equivalent of 1,195.498 bd. ft.

After the issuance of Cover Note No. 1010 (Exhibit A), but before the issuance of the two marine policies
Nos. 53 HO 1032 and 53 HO 1033, some of the logs intended to be exported were lost during loading
operations in the Diapitan Bay. The logs were to be loaded on the 'SS Woodlock' which docked about 500
meters from the shoreline of the Diapitan Bay. The logs were taken from the log pond of the plaintiff and
from which they were towed in rafts to the vessel. At about 10:00 o'clock a. m. on March 29, 1963, while the
logs were alongside the vessel, bad weather developed resulting in 75 pieces of logs which were rafted
together co break loose from each other. 45 pieces of logs were salvaged, but 30 pieces were verified to
have been lost or washed away as a result of the accident.

In a letter dated April 4, 1963, the plaintiff informed the defendant about the loss of 'appropriately 32 pieces of log's during
loading of the 'SS Woodlock'. The said letter (Exhibit F) reads as follows:

April 4, 1963

Workmen's Insurance Company, Inc. Manila, Philippines

Gentlemen:

This has reference to Insurance Cover Note No. 1010 for shipment of 1,250,000 bd. ft. Philippine Lauan
and Apitong Logs. We would like to inform you that we have received advance preliminary report from our
Office in Diapitan, Quezon that we have lost approximately 32 pieces of logs during loading of the SS
Woodlock.

We will send you an accurate report all the details including values as soon as same will be reported to us.

Thank you for your attention, we wish to remain.

Very respectfully yours,

PACIFIC TIMBER EXPORT CORPORATION

(Sgd.) EMMANUEL S. ATILANO Asst. General Manager.

Although dated April 4, 1963, the letter was received in the office of the defendant only on April 15, 1963, as
shown by the stamp impression appearing on the left bottom corner of said letter. The plaintiff subsequently
submitted a 'Claim Statement demanding payment of the loss under Policies Nos. 53 HO 1032 and 53 HO
1033, in the total amount of P19,286.79 (Exhibit G).
On July 17, 1963, the defendant requested the First Philippine Adjustment Corporation to inspect the loss
and assess the damage. The adjustment company submitted its 'Report on August 23, 1963 (Exhibit H). In
said report, the adjuster found that 'the loss of 30 pieces of logs is not covered by Policies Nos. 53 HO 1032
and 1033 inasmuch as said policies covered the actual number of logs loaded on board the 'SS Woodlock'
However, the loss of 30 pieces of logs is within the 1,250,000 bd. ft. covered by Cover Note 1010 insured
for $70,000.00.

On September 14, 1963, the adjustment company submitted a computation of the defendant's probable
liability on the loss sustained by the shipment, in the total amount of Pl1,042.04 (Exhibit 4).

On January 13, 1964, the defendant wrote the plaintiff denying the latter's claim, on the ground they
defendant's investigation revealed that the entire shipment of logs covered by the two marines policies No.
53 110 1032 and 713 HO 1033 were received in good order at their point of destination. It was further stated
that the said loss may be considered as covered under Cover Note No. 1010 because the said Note had
become 'null and void by virtue of the issuance of Marine Policy Nos. 53 HO 1032 and 1033'(Exhibit J-1).
The denial of the claim by the defendant was brought by the plaintiff to the attention of the Insurance
Commissioner by means of a letter dated March 21, 1964 (Exhibit K). In a reply letter dated March 30,
1964, Insurance Commissioner Francisco Y. Mandanas observed that 'it is only fair and equitable to
indemnify the insured under Cover Note No. 1010', and advised early settlement of the said marine loss and
salvage claim (Exhibit L).

On June 26, 1964, the defendant informed the Insurance Commissioner that, on advice of their attorneys,
the claim of the plaintiff is being denied on the ground that the cover note is null and void for lack of valuable
consideration (Exhibit M). 4

Petitioner assigned as errors of the Court of Appeals, the following:

THE COURT OF APPEALS ERRED IN HOLDING THAT THE COVER NOTE WAS NULL AND VOID FOR
LACK OF VALUABLE CONSIDERATION BECAUSE THE COURT DISREGARDED THE PROVEN FACTS
THAT PREMIUMS FOR THE COMPREHENSIVE INSURANCE COVERAGE THAT INCLUDED THE
COVER NOTE WAS PAID BY PETITIONER AND THAT INCLUDED THE COVER NOTE WAS PAID BY
PETITIONER AND THAT NO SEPARATE PREMIUMS ARE COLLECTED BY PRIVATE RESPONDENT ON
ALL ITS COVER NOTES.

II

THE COURT OF APPEALS ERRED IN HOLDING THAT PRIVATE RESPONDENT WAS RELEASED
FROM LIABILITY UNDER THE COVER NOTE DUE TO UNREASONABLE DELAY IN GIVING NOTICE OF
LOSS BECAUSE THE COURT DISREGARDED THE PROVEN FACT THAT PRIVATE RESPONDENT DID
NOT PROMPTLY AND SPECIFICALLY OBJECT TO THE CLAIM ON THE GROUND OF DELAY IN GIVING
NOTICE OF LOSS AND, CONSEQUENTLY, OBJECTIONS ON THAT GROUND ARE WAIVED UNDER
SECTION 84 OF THE INSURANCE ACT. 5

1. Petitioner contends that the Cover Note was issued with a consideration when, by express stipulation, the cover note is
made subject to the terms and conditions of the marine policies, and the payment of premiums is one of the terms of the
policies. From this undisputed fact, We uphold petitioner's submission that the Cover Note was not without consideration for
which the respondent court held the Cover Note as null and void, and denied recovery therefrom. The fact that no separate
premium was paid on the Cover Note before the loss insured against occurred, does not militate against the validity of
petitioner's contention, for no such premium could have been paid, since by the nature of the Cover Note, it did not contain,
as all Cover Notes do not contain particulars of the shipment that would serve as basis for the computation of the premiums.
As a logical consequence, no separate premiums are intended or required to be paid on a Cover Note. This is a fact admitted
by an official of respondent company, Juan Jose Camacho, in charge of issuing cover notes of the respondent company (p.
33, tsn, September 24, 1965).

At any rate, it is not disputed that petitioner paid in full all the premiums as called for by the statement issued by private
respondent after the issuance of the two regular marine insurance policies, thereby leaving no account unpaid by petitioner
due on the insurance coverage, which must be deemed to include the Cover Note. If the Note is to be treated as a separate
policy instead of integrating it to the regular policies subsequently issued, the purpose and function of the Cover Note would
be set at naught or rendered meaningless, for it is in a real sense a contract, not a mere application for insurance which is a
mere offer. 6

It may be true that the marine insurance policies issued were for logs no longer including those which had been lost during
loading operations. This had to be so because the risk insured against is not for loss during operations anymore, but for loss
during transit, the logs having already been safely placed aboard. This would make no difference, however, insofar as the
liability on the cover note is concerned, for the number or volume of logs lost can be determined independently as in fact it
had been so ascertained at the instance of private respondent itself when it sent its own adjuster to investigate and assess
the loss, after the issuance of the marine insurance policies.

The adjuster went as far as submitting his report to respondent, as well as its computation of respondent's liability on the
insurance coverage. This coverage could not have been no other than what was stipulated in the Cover Note, for no loss or
damage had to be assessed on the coverage arising from the marine insurance policies. For obvious reasons, it was not
necessary to ask petitioner to pay premium on the Cover Note, for the loss insured against having already occurred, the
more practical procedure is simply to deduct the premium from the amount due the petitioner on the Cover Note. The non-
payment of premium on the Cover Note is, therefore, no cause for the petitioner to lose what is due it as if there had been
payment of premium, for non-payment by it was not chargeable against its fault. Had all the logs been lost during the loading
operations, but after the issuance of the Cover Note, liability on the note would have already arisen even before payment of
premium. This is how the cover note as a "binder" should legally operate otherwise, it would serve no practical purpose in the
realm of commerce, and is supported by the doctrine that where a policy is delivered without requiring payment of the
premium, the presumption is that a credit was intended and policy is valid. 7

2. The defense of delay as raised by private respondent in resisting the claim cannot be sustained. The law requires this
ground of delay to be promptly and specifically asserted when a claim on the insurance agreement is made. The undisputed
facts show that instead of invoking the ground of delay in objecting to petitioner's claim of recovery on the cover note, it took
steps clearly indicative that this particular ground for objection to the claim was never in its mind. The nature of this specific
ground for resisting a claim places the insurer on duty to inquire when the loss took place, so that it could determine whether
delay would be a valid ground upon which to object to a claim against it.

As already stated earlier, private respondent's reaction upon receipt of the notice of loss, which was on April 15, 1963, was to
set in motion from July 1963 what would be necessary to determine the cause and extent of the loss, with a view to the
payment thereof on the insurance agreement. Thus it sent its adjuster to investigate and assess the loss in July, 1963. The
adjuster submitted his report on August 23, 1963 and its computation of respondent's liability on September 14, 1963. From
April 1963 to July, 1963, enough time was available for private respondent to determine if petitioner was guilty of delay in
communicating the loss to respondent company. In the proceedings that took place later in the Office of the Insurance
Commissioner, private respondent should then have raised this ground of delay to avoid liability. It did not do so. It must be
because it did not find any delay, as this Court fails to find a real and substantial sign thereof. But even on the assumption
that there was delay, this Court is satisfied and convinced that as expressly provided by law, waiver can successfully be
raised against private respondent. Thus Section 84 of the Insurance Act provides:

Section 84.Delay in the presentation to an insurer of notice or proof of loss is waived if caused by any act
of his or if he omits to take objection promptly and specifically upon that ground.

From what has been said, We find duly substantiated petitioner's assignments of error.
ACCORDINGLY, the appealed decision is set aside and the decision of the Court of First Instance is reinstated in toto with
the affirmance of this Court. No special pronouncement as to costs.

SO ORDERED.

Teehankee (Chairman), Makasiar, Fernandez Guerrero, Melencio-Herrera and Plana, JJ., concur.

G.R. No. 113899 October 13, 1999

GREAT PACIFIC LIFE ASSURANCE CORP., petitioner,


vs.
COURT OF APPEALS AND MEDARDA V. LEUTERIO, respondents.

QUISUMBING, J.:

This petition for review, under Rule 45 of the Rules of Court, assails the Decision 1 dated May 17, 1993, of the Court of
Appeals and its Resolution 2 dated January 4, 1994 in CA-G.R. CV No. 18341. The appellate court affirmed in toto the
judgment of the Misamis Oriental Regional Trial Court, Branch 18, in an insurance claim filed by private respondent
against Great Pacific Life Assurance Co. The dispositive portion of the trial court's decision reads:

WHEREFORE, judgment is rendered adjudging the defendant GREAT PACIFIC LIFE ASSURANCE
CORPORATION as insurer under its Group policy No. G-1907, in relation to Certification B-18558 liable and
ordered to pay to the DEVELOPMENT BANK OF THE PHILIPPINES as creditor of the insured Dr. Wilfredo
Leuterio, the amount of EIGHTY SIX THOUSAND TWO HUNDRED PESOS (P86,200.00); dismissing the
claims for damages, attorney's fees and litigation expenses in the complaint and counterclaim, with costs
against the defendant and dismissing the complaint in respect to the plaintiffs, other than the widow-
beneficiary, for lack of cause of action. 3

The facts, as found by the Court of Appeals, are as follows:

A contract of group life insurance was executed between petitioner Great Pacific Life Assurance Corporation (hereinafter
Grepalife) and Development Bank of the Philippines (hereinafter DBP). Grepalife agreed to insure the lives of eligible
housing loan mortgagors of DBP.

On November 11, 1983, Dr. Wilfredo Leuterio, a physician and a housing debtor of DBP applied for membership in the group
life insurance plan. In an application form, Dr. Leuterio answered questions concerning his health condition as follows:

7. Have you ever had, or consulted, a physician for a heart condition, high blood pressure,
cancer, diabetes, lung; kidney or stomach disorder or any other physical impairment?

Answer: No. If so give details _____________.

8. Are you now, to the best of your knowledge, in good health?

Answer: [x] Yes [ ] NO. 4

On November 15, 1983, Grepalife issued Certificate No. B-18558, as insurance coverage of Dr. Leuterio, to the extent of his
DBP mortgage indebtedness amounting to eighty-six thousand, two hundred (P86,200.00) pesos. 1wphi1.nt

On August 6, 1984, Dr. Leuterio died due to "massive cerebral hemorrhage." Consequently, DBP submitted a death claim to
Grepalife. Grepalife denied the claim alleging that Dr. Leuterio was not physically healthy when he applied for an insurance
coverage on November 15, 1983. Grepalife insisted that Dr. Leuterio did not disclose he had been suffering from
hypertension, which caused his death. Allegedly, such non-disclosure constituted concealment that justified the denial of the
claim.

On October 20, 1986, the widow of the late Dr. Leuterio, respondent Medarda V. Leuterio, filed a complaint with the Regional
Trial Court of Misamis Oriental, Branch 18, against Grepalife for "Specific Performance with Damages." 5 During the trial, Dr.
Hernando Mejia, who issued the death certificate, was called to testify. Dr. Mejia's findings, based partly from the
information given by the respondent widow, stated that Dr. Leuterio complained of headaches presumably due to high
blood pressure. The inference was not conclusive because Dr. Leuterio was not autopsied, hence, other causes were
not ruled out.

On February 22, 1988, the trial court rendered a decision in favor of respondent widow and against Grepalife. On May 17,
1993, the Court of Appeals sustained the trial court's decision. Hence, the present petition. Petitioners interposed the
following assigned errors:

1. THE LOWER COURT ERRED IN HOLDING DEFENDANT-APPELLANT LIABLE TO


THE DEVELOPMENT BANK OF THE PHILIPPINES (DBP) WHICH IS NOT A PARTY TO
THE CASE FOR PAYMENT OF THE PROCEEDS OF A MORTGAGE REDEMPTION
INSURANCE ON THE LIFE OF PLAINTIFF'S HUSBAND WILFREDO LEUTERIO ONE
OF ITS LOAN BORROWERS, INSTEAD OF DISMISSING THE CASE AGAINST
DEFENDANT-APPELLANT [Petitioner Grepalife] FOR LACK OF CAUSE OF ACTION.

2. THE LOWER COURT ERRED IN NOT DISMISSING THE CASE FOR WANT OF
JURISDICTION OVER THE SUBJECT OR NATURE OF THE ACTION AND OVER THE
PERSON OF THE DEFENDANT.
3. THE LOWER COURT ERRED IN ORDERING DEFENDANT-APPELLANT TO PAY TO
DBP THE AMOUNT OF P86,200.00 IN THE ABSENCE OF ANY EVIDENCE TO SHOW
HOW MUCH WAS THE ACTUAL AMOUNT PAYABLE TO DBP IN ACCORDANCE WITH
ITS GROUP INSURANCE CONTRACT WITH DEFENDANT-APPELLANT.

4. THE LOWER COURT ERRED IN HOLDING THAT THERE WAS NO CONCEALMENT


OF MATERIAL INFORMATION ON THE PART OF WILFREDO LEUTERIO IN HIS
APPLICATION FOR MEMBERSHIP IN THE GROUP LIFE INSURANCE PLAN BETWEEN
DEFENDANT-APPELLANT OF THE INSURANCE CLAIM ARISING FROM THE DEATH
OF WILFREDO LEUTERIO. 6

Synthesized below are the assigned errors for our resolution:

1. Whether the Court of Appeals erred in holding petitioner liable to DBP as beneficiary in
a group life insurance contract from a complaint filed by the widow of the
decedent/mortgagor?

2. Whether the Court of Appeals erred in not finding that Dr. Leuterio concealed that he
had hypertension, which would vitiate the insurance contract?

3. Whether the Court of Appeals erred in holding Grepalife liable in the amount of eighty
six thousand, two hundred (P86,200.00) pesos without proof of the actual outstanding
mortgage payable by the mortgagor to DBP.

Petitioner alleges that the complaint was instituted by the widow of Dr. Leuterio, not the real party in interest, hence the trial
court acquired no jurisdiction over the case. It argues that when the Court of Appeals affirmed the trial court's judgment,
Grepalife was held liable to pay the proceeds of insurance contract in favor of DBP, the indispensable party who was not
joined in the suit.

To resolve the issue, we must consider the insurable interest in mortgaged properties and the parties to this type of contract.
The rationale of a group insurance policy of mortgagors, otherwise known as the "mortgage redemption insurance," is a
device for the protection of both the mortgagee and the mortgagor. On the part of the mortgagee, it has to enter into such
form of contract so that in the event of the unexpected demise of the mortgagor during the subsistence of the mortgage
contract, the proceeds from such insurance will be applied to the payment of the mortgage debt, thereby relieving the heirs of
the mortgagor from paying the obligation. 7 In a similar vein, ample protection is given to the mortgagor under such a
concept so that in the event of death; the mortgage obligation will be extinguished by the application of the insurance
proceeds to the mortgage indebtedness. 8 Consequently, where the mortgagor pays the insurance premium under the
group insurance policy, making the loss payable to the mortgagee, the insurance is on the mortgagor's interest, and
the mortgagor continues to be a party to the contract. In this type of policy insurance, the mortgagee is simply an
appointee of the insurance fund, such loss-payable clause does not make the mortgagee a party to the contract. 9

Sec. 8 of the Insurance Code provides:

Unless the policy provides, where a mortgagor of property effects insurance in his own name providing that
the loss shall be payable to the mortgagee, or assigns a policy of insurance to a mortgagee, the insurance
is deemed to be upon the interest of the mortgagor, who does not cease to be a party to the original
contract, and any act of his, prior to the loss, which would otherwise avoid the insurance, will have the same
effect, although the property is in the hands of the mortgagee, but any act which, under the contract of
insurance, is to be performed by the mortgagor, may be performed by the mortgagee therein named, with
the same effect as if it had been performed by the mortgagor.

The insured private respondent did not cede to the mortgagee all his rights or interests in the insurance, the policy stating
that: "In the event of the debtor's death before his indebtedness with the Creditor [DBP] shall have been fully paid, an amount
to pay the outstanding indebtedness shall first be paid to the creditor and the balance of sum assured, if there is any, shall
then be paid to the beneficiary/ies designated by the debtor." 10 When DBP submitted the insurance claim against
petitioner, the latter denied payment thereof, interposing the defense of concealment committed by the insured.
Thereafter, DBP collected the debt from the mortgagor and took the necessary action of foreclosure on the residential
lot of private respondent. 11 In Gonzales La O vs. Yek Tong Lin Fire & Marine Ins. Co. 12 we held:

Insured, being the person with whom the contract was made, is primarily the proper person to bring suit
thereon. * * * Subject to some exceptions, insured may thus sue, although the policy is taken wholly or in
part for the benefit of another person named or unnamed, and although it is expressly made payable to
another as his interest may appear or otherwise. * * * Although a policy issued to a mortgagor is taken out
for the benefit of the mortgagee and is made payable to him, yet the mortgagor may sue thereon in his own
name, especially where the mortgagee's interest is less than the full amount recoverable under the policy, *
* *.

And in volume 33, page 82, of the same work, we read the following:

Insured may be regarded as the real party in interest, although he has assigned the policy for the purpose
of collection, or has assigned as collateral security any judgment he may obtain. 13

And since a policy of insurance upon life or health may pass by transfer, will or succession to any person, whether he has an
insurable interest or not, and such person may recover it whatever the insured might have recovered, 14 the widow of the
decedent Dr. Leuterio may file the suit against the insurer, Grepalife.

The second assigned error refers to an alleged concealment that the petitioner interposed as its defense to annul the
insurance contract. Petitioner contends that Dr. Leuterio failed to disclose that he had hypertension, which might have
caused his death. Concealment exists where the assured had knowledge of a fact material to the risk, and honesty, good
faith, and fair dealing requires that he should communicate it to the assured, but he designedly and intentionally withholds
the same. 15

Petitioner merely relied on the testimony of the attending physician, Dr. Hernando Mejia, as supported by the information
given by the widow of the decedent. Grepalife asserts that Dr. Mejia's technical diagnosis of the cause of death of Dr.
Leuterio was a duly documented hospital record, and that the widow's declaration that her husband had "possible
hypertension several years ago" should not be considered as hearsay, but as part of res gestae.

On the contrary the medical findings were not conclusive because Dr. Mejia did not conduct an autopsy on the body of the
decedent. As the attending physician, Dr. Mejia stated that he had no knowledge of Dr. Leuterio's any previous hospital
confinement. 16 Dr. Leuterio's death certificate stated that hypertension was only "the possible cause of death." The
private respondent's statement, as to the medical history of her husband, was due to her unreliable recollection of
events. Hence, the statement of the physician was properly considered by the trial court as hearsay.

The question of whether there was concealment was aptly answered by the appellate court, thus:

The insured, Dr. Leuterio, had answered in his insurance application that he was in good health and that he
had not consulted a doctor or any of the enumerated ailments, including hypertension; when he died the
attending physician had certified in the death certificate that the former died of cerebral hemorrhage,
probably secondary to hypertension. From this report, the appellant insurance company refused to pay the
insurance claim. Appellant alleged that the insured had concealed the fact that he had hypertension.

Contrary to appellant's allegations, there was no sufficient proof that the insured had suffered from
hypertension. Aside from the statement of the insured's widow who was not even sure if the medicines
taken by Dr. Leuterio were for hypertension, the appellant had not proven nor produced any witness who
could attest to Dr. Leuterio's medical history . . .

xxx xxx xxx

Appellant insurance company had failed to establish that there was concealment made by the insured,
hence, it cannot refuse payment of the claim. 17

The fraudulent intent on the part of the insured must be established to entitle the insurer to rescind the
contract.18 Misrepresentation as a defense of the insurer to avoid liability is an affirmative defense and the duty to
establish such defense by satisfactory and convincing evidence rests upon the insurer. 19 In the case at bar, the
petitioner failed to clearly and satisfactorily establish its defense, and is therefore liable to pay the proceeds of the
insurance. 1wphi1.nt

And that brings us to the last point in the review of the case at bar. Petitioner claims that there was no evidence as to the
amount of Dr. Leuterio's outstanding indebtedness to DBP at the time of the mortgagor's death. Hence, for private
respondent's failure to establish the same, the action for specific performance should be dismissed. Petitioner's claim is
without merit. A life insurance policy is a valued policy. 20 Unless the interest of a person insured is susceptible of exact
pecuniary measurement, the measure of indemnity under a policy of insurance upon life or health is the sum fixed in
the policy. 21 The mortgagor paid the premium according to the coverage of his insurance, which states that:

The policy states that upon receipt of due proof of the Debtor's death during the terms of this insurance, a
death benefit in the amount of P86,200.00 shall be paid.

In the event of the debtor's death before his indebtedness with the creditor shall have been fully paid, an
amount to pay the outstanding indebtedness shall first be paid to the Creditor and the balance of the Sum
Assured, if there is any shall then be paid to the beneficiary/ies designated by the debtor." 22 (Emphasis
omitted)

However, we noted that the Court of Appeals' decision was promulgated on May 17, 1993. In private respondent's
memorandum, she states that DBP foreclosed in 1995 their residential lot, in satisfaction of mortgagor's outstanding loan.
Considering this supervening event, the insurance proceeds shall inure to the benefit of the heirs of the deceased person or
his beneficiaries. Equity dictates that DBP should not unjustly enrich itself at the expense of another (Nemo cum alterius
detrimenio protest). Hence, it cannot collect the insurance proceeds, after it already foreclosed on the mortgage. The
proceeds now rightly belong to Dr. Leuterio's heirs represented by his widow, herein private respondent Medarda Leuterio.

WHEREFORE, the petition is hereby DENIED. The Decision and Resolution of the Court of Appeals in CA-G.R. CV 18341 is
AFFIRMED with MODIFICATION that the petitioner is ORDERED to pay the insurance proceeds amounting to Eighty-six
thousand, two hundred (P86,200.00) pesos to the heirs of the insured, Dr. Wilfredo Leuterio (deceased), upon presentation
of proof of prior settlement of mortgagor's indebtedness to Development Bank of the Philippines. Costs against petitioner. 1wphi1.nt

SO ORDERED.

G.R. No. 139776 August 1, 2002

PHILIPPINE AMERICAN LIFE AND GENERAL INSURANCE COMPANY, petitioner,


vs.
JUDGE LORE R. VALENCIA-BAGALACSA, Regional Trial Court of Libmanan, Camarines Sur, Branch 56, and
EDUARDO Z. LUMANIOG, CELSO Z. LUMANIOG and RUBEN Z. LUMANIOG, respondents.

DECISION
AUSTRIA-MARTINEZ, J.:

Before us is a petition for review on certiorari under Rule 45 of the Rules of Court. Petitioner Philippine American Life and
General Insurance Company prays that the decision of the Court of Appeals promulgated on April 30, 1999 be reversed and
set aside and that the Complaint filed against it by private respondents Eduardo Z. Lumaniog, Celso Z. Lumaniog and Ruben
Z. Lumaniog before the Regional Trial Court of Libmanan, Camarines Sur, docketed as Civil Case No. L-787 be ordered
dismissed on ground of prescription of action.

The facts of the case:

On June 20, 1995, private respondents, as legitimate children and forced heirs of their late father, Faustino Lumaniog, filed
with the aforesaid RTC, a complaint for recovery of sum of money against petitioner alleging that: their father was insured by
petitioner under Life Insurance Policy No. 1305486 with a face value of P50,000.00; their father died of "coronary
thrombosis" on November 25, 1980; on June 22, 1981, they claimed and continuously claimed for all the proceeds and
interests under the life insurance policy in the amount of P641,000.00, despite repeated demands for payment and/or
settlement of the claim due from petitioner, the last of which is on December 1, 1994, petitioner finally refused or disallowed
said claim on February 14, 1995;1 and so, they filed their complaint on June 20, 1995.

Petitioner filed an Answer with Counterclaim and Motion to Dismiss, contending that: the cause of action of private
respondents had prescribed and they are guilty of laches; it had denied private respondents claim in a letter dated March 12,
1982, signed by its then Assistant Vice President, Amado Dimalanta, on ground of concealment on the part of the deceased
insured Faustino when he asserted in his application for insurance coverage that he had not been treated for indication of
"chest pain, palpitation, high blood pressure, rheumatic fever, heart murmur, heart attack or other disorder of the heart or
blood vessel" when in fact he was a known hypertensive since 1974; private respondents sent a letter dated May 25,
19832 requesting for reconsideration of the denial; in a letter dated July 11, 1983, it reiterated its decision to deny the claim
for payment of the proceeds;3 more than ten (10) years later, or on December 1, 1994, it received a letter from Jose C. Claro,
a provincial board member of the province of Camarines Sur, reiterating the early request for reconsideration which it denied
in a letter dated February 14, 1995.4

Private respondents opposed the motion to dismiss.5

On June 7, 1996, the RTC issued an Order which reads:

"After a perusal of the motion to dismiss filed by defendants counsel and the objection submitted by plaintiffs counsel, the
Court finds that the matters treated in their respective pleadings are evidentiary in nature, hence, the necessity of a trial on
the merits.

"Set therefore the hearing in this case on August 1, 1996 at 8:30 a.m., considering that the calendar of the Court is already
filled up until the end of July. Notify parties and counsels.

"SO ORDERED."6

Petitioners motion for reconsideration was denied by the RTC in its Order dated December 12, 1997 upholding however in
the same Order the claim of private respondents counsel that the running of the 10-year period was "stopped" on May 25,
1983 when private respondents requested for a reconsideration of the denial and it was only on February 14, 1995 when
petitioner finally decided to deny their claim that the 10-year period began to run. 7

Petitioner filed a petition for certiorari (docketed as CA-G.R. SP No. 47885) under Rule 65 of the Rules of Court in the Court
of Appeals and after the comment of the private respondents and reply of petitioner, the appellate court rendered its
Decision, dated April 30, 1999, portions of which read as follows:

"Thus, this Court of the opinion and so holds that the prescriptive period to bring the present action commences to run only
on February 14, 1995 (Rollo, pp. 25-26), the date when the petitioner finally rejected the claim of private respondents and not
in 1983. The ten year period should instead be counted from the date of rejection by the insurer in this case February 14,
1995 since this is the time when the cause of action accrues.

"This fact was supported further by the letter of the petitioner to Atty. Claro dated December 20, 1994, stating that they were
reviewing the claim and shall advise Atty. Claro of their action regarding his request for reconsideration (Id., p. 53).
"In the case of Summit Guaranty and Insurance Co., Inc. Vs. De Guzman (151 SCRA 389, 397-398), citing the case of Eagle
Star Insurance Co., Ltd., et al. vs. Chia Yu, the Supreme Court held that:

The plaintiffs cause of action did not accrue until his claim was finally rejected by the insurance company. This is because,
before such final rejection, there was no real necessity for bringing suit.

"In the same case, the case of ACCFA vs. Alpha Insurance and Surety Co., was likewise cited where the Supreme Court
ruled in this wise:

Since a cause of action requires, as essential elements, not only a legal right of the plaintiff and a correlative of the
defendant but also an act or omission of the defendant in violation of said legal right, the cause of action does not accrue
until the party obligated refuses, expressly or impliedly, to comply with its duty.

"Hence, We find no grave abuse of discretion committed by the court a quo when it issued the Orders dated June 7, 1996
and dated December 12, 1997.

"WHEREFORE, the instant petition for certiorari with prayer for issuance of temporary restraining order and/or preliminary
injunction is DENIED DUE COURSE and is accordingly DISMISSED by this Court for lack of merit.

"Costs against the petitioner.

"SO ORDERED."8

Hence, the present petition for review. Petitioner posits the following issues:

"A. Whether or not the complaint filed by private respondents for payment of life insurance proceeds is already barred by
prescription of action.

"B. Whether or not an extrajudicial demand made after an action has prescribed shall cause the revival of the action." 9

Private respondents filed their Comment and petitioners, their Reply.

Before we determine whether the Court of Appeals had committed any reversible error, we must necessarily first ascertain
whether or not the RTC committed grave abuse of discretion in issuing the Orders dated June 7, 1996 and December 12,
1997.

Notably, the RTC was initially correct in issuing the Order dated June 7, 1996 when it set the case below for hearing as there
are matters in the respective pleadings of the parties "that are evidentiary in nature, hence the necessity of a trial on the
merits"10, in effect, denying the motion to dismiss, pursuant to the then prevailing Section 3, Rule 16, of the Rules of Court, to
wit:

"Sec. 3. Hearing and order. - After hearing the court may deny or grant the motion or allow amendment of pleading, or may
defer the hearing and determination of the motion until the trial if the ground alleged therein does not appear to be
indubitable."

before it was amended by the 1997 Rules of Civil Procedure, effective July 1, 1997. 11

It must be emphasized that petitioner had specifically alleged in the Answer that it had denied private respondents claim per
its letter dated July 11, 1983.12 Hence, due process demands that it be given the opportunity to prove that private
respondents had received said letter, dated July 11, 1983. Said letter is crucial to petitioners defense that the filing of the
complaint for recovery of sum of money in June, 1995 is beyond the 10-year prescriptive period 13.

It is for the above reason that the RTC committed a grave abuse of discretion when, in resolving the motion for
reconsideration of petitioner, it arbitrarily ruled in its Order dated December 12, 1997, that the period of ten (10) years had
not yet lapsed. It based its finding on a mere explanation of the private respondents counsel and not on evidence presented
by the parties as to the date when to reckon the prescriptive period. Portions of the Order dated December 12, 1997 read:

"A perusal of the record will likewise reveal that plaintiffs counsel explained that the running of the ten (10) year period was
stopped on May 25, 1983, upon demand of Celso Lomaniog for the compliance of the contract and reconsideration of the
decision. Counsel also wrote the President of the Company on December 1, 1994, asking for reconsideration. The letter was
answered by the Assistant Vice President of the Claims Department of Philamlife, with the advise that the company is
reviewing the claim. On February 14, 1995, Atty. Abis sent a letter to counsel, finally deciding the plaintiffs claim. Thus, the
period of prescription should commence to run only from February 14, 1995, when Atty. Abis finally decided plaintiffs claim. 1wphi1

"It is evident from the foregoing that the ten (10) year period for plaintiffs to claim the insurance proceeds has not yet
prescribed. The final determination denying the claim was made only on February 14, 1995. Hence, when the instant case
was filed on June 20, 1995, the ten year period has not yet lapsed. Moreover, defendants counsel failed to comply with the
requirements of the Rules in filing his motion for reconsideration." 14 (emphasis supplied)

The ruling of the RTC that the cause of action of private respondents had not prescribed, is arbitrary and patently erroneous
for not being founded on evidence on record, and therefore, the same is void. 15

Consequently, while the Court of Appeals did not err in upholding the June 7, 1986 Order of the RTC, it committed a
reversible error when it declared that the RTC did not commit any grave abuse of discretion in issuing the Order dated
December 12, 1997.

The appellate court should have granted the petition for certiorari assailing said Order of December 12, 1997. Certiorari is an
appropriate remedy to assail an interlocutory order (1) when the tribunal issued such order without or in excess of jurisdiction
or with grave abuse of discretion and (2) when the assailed interlocutory order is patently erroneous and the remedy of
appeal would not afford adequate and expeditious relief. 16 Said Order was issued with grave abuse of discretion for being
patently erroneous and arbitrary, thus, depriving petitioner of due process, as discussed earlier.

WHEREFORE, the petition is partly GRANTED. The assailed decision of the Court of Appeals dated April 30, 1999 insofar
only as it upheld the Order dated December 12, 1997 is REVERSED and SET ASIDE. A new judgment is entered reversing
and setting aside the Order dated December 12, 1997 of the Regional Trial Court of Libmanan, Camarines Sur (Branch 56)
and affirming its Order dated June 20, 1995. Said RTC is directed to proceed with dispatch with Civil Case No. L-787.

No costs.

SO ORDERED.

Davide, Jr., C.J., (Chairman), Vitug, Kapunan, and Ynares-Santiago, JJ., concur.

Вам также может понравиться